You are on page 1of 82

Section #2: View Explanations 1 Explanation for Correct Answer B. Choice (B) is correct.

The term "suffrage" refers to the right to vote. Because the structure of the sentence suggests that the term that fits the blank will be synonymous with "voting rights," the term "suffrage" fits perfectly. The sentence indicates that Black American men gained the right to vote before women in the United States did, but that "Black citizens had greater difficulty exercising their new voting rights.". Choice (A) is incorrect. "Restitution" is the act of returning something or repaying someone. The structure of the sentence suggests that the term that fits the blank will be synonymous with "voting rights"; there is no direct connection between voting rights and anything being returned or repaid, so the term "restitution" does not logically complete the sentence. Indeed, the sentence refers to rights granted to certain groups of people, not rights that were returned to those groups.. Choice (C) is incorrect. "Solace" is comfort or a source of relief. The structure of the sentence suggests that the term that fits the blank will be synonymous with "voting rights"; there is no direct connection between voting rights and comfort, so the term "solace" does not logically complete the sentence. There is no mention of comfort or relief.. Choice (D) is incorrect. In this context, the term "initiatives" refers to beginning steps or opening moves. Although the sentence indicates that women and Black citizens gained "new . . . rights," the sentence does not state that these rights were initiatives of any kind. Further, the structure of the sentence suggests that the term that fits the blank will be synonymous with "voting rights," and there is not necessarily a connection between voting rights and beginning steps.. Choice (E) is incorrect. A levy is a collected sum, similar to a tax. The structure of the sentence suggests that the term that fits the blank will be synonymous with "voting rights"; there is no direct connection between voting rights and collected sums, so the term "levies" does not logically complete the sentence. There is no mention of collected sums or taxes. 2 Explanation for Correct Answer B. Choice (B) is correct. "Demographic" means relating to the statistical characteristics of human populations. The structure of the sentence indicates that the part of the sentence after the colon defines the term that belongs in the blank. Because "statistical data about the size, growth, and distribution of human populations" is synonymous with "demographic information," the term "demographic" logically completes the sentence.. Choice (A) is incorrect. In this context, "classified" means secret or available only to authorized persons. The structure of the sentence indicates that the part of the sentence after the colon defines the term that belongs in the blank. Some marketing firms may rely on information that is available only to authorized persons, but "statistical data about the size, growth, and distribution of human populations" is not necessarily classified information. Rather than describing classified information, the part of the sentence after the colon is a definition of demographic information..

Choice (C) is incorrect. "Conjectural" means based on guesswork. The structure of the sentence indicates that the part of the sentence after the colon defines the term that belongs in the blank. Some marketing firms may rely on information that is based on guesswork, but it is somewhat illogical to describe "statistical data about the size, growth, and distribution of human populations" as conjectural information because statistical information is based on the collection and analysis of real data. The part of the sentence after the colon is a definition of demographic information, not conjectural information.. Choice (D) is incorrect. "Qualitative" means pertaining to something's qualities or components. The structure of the sentence indicates that the part of the sentence after the colon defines the term that belongs in the blank. There might be some qualitative aspects of "data about the size, growth, and distribution of human populations," but statistical data of this type is not necessarily qualitative. In fact, statistical data is more accurately described as quantitative, or dealing with quantities, not qualitative. Rather than describing qualitative information, the part of the sentence after the colon is a definition of demographic information.. Choice (E) is incorrect. The term "anecdotal" refers to information that is based on casual or unscientific observation. The structure of the sentence indicates that the part of the sentence after the colon defines the term that belongs in the blank. If the "data about the size, growth, and distribution of human populations" used by marketing firms is "statistical," or based on the collection and analysis of real data, then it certainly is not anecdotal, or casual and unscientific. The part of the sentence after the colon is a definition of demographic information, not anecdotal information. 3 Explanation for Correct Answer D. Choice (D) is correct. To "resign" is to quit or leave a job. "Autonomy" is selfgoverned freedom or independence. The structure of the sentence makes it clear that the first part of the sentence is meant to modify the term that belongs in the first blank; if certain people elect, or choose, not to stay in subordinate, or secondary, positions in large companies, it makes sense to say they would resign. The final part of the sentence explains why such people might quit their jobs, and we know from the structure of the sentence structure that the term that fits the second blank will be synonymous with "independence": the people who resign will seek more autonomy, or freedom, somewhere else.. Choice (A) is incorrect. To "compromise" is to give up something to get something else. "Servility" is forced or humiliating servitude. The structure of the sentence makes it clear that the first part of the sentence is meant to modify the term that belongs in the first blank; it might make sense to say that certain people who elect, or choose, to leave large companies are making a compromise, giving up one job and settling for another. However, the term "servility" does not logically complete the sentence. It is unlikely that any employee would desire more servility, or forced servitudeparticularly an employee who had rejected a "subordinate," or secondary, position.. Choice (B) is incorrect. To "persevere" is to keep trying in spite of opposition. "Competence" is the ability to perform something successfully. The structure of the sentence makes it clear that the first part of the sentence is meant to modify the term that belongs in the first blank; however, someone who elects, or chooses, to persevere would stay in, not leave, an unappealing job. The term "competence" might make some sense in the second blankan unhappy

employee might, after all, leave a company in search of more competent management elsewherebut the term persevere does not make sense in the first blank.. Choice (C) is incorrect. To "acquiesce" is to give in or submit passively. "Banality" is triviality or meaninglessness. The structure of the sentence makes it clear that the first part of the sentence is meant to modify the term that belongs in the first blank; however, someone who elects, or chooses, to acquiesce, or give in, would likely stay in an unappealing job, not leave it. Further, although any job could involve doing trivial work, the term banality is rather pejorative, or negative; it is unlikely that an employee would seek banality in a job.. Choice (E) is incorrect. To "recant" is to take back a statement. "Conformity" is the act of conforming or fitting in. The structure of the sentence makes it clear that the first part of the sentence is meant to modify the term that belongs in the first blank; however, recanting is not necessarily something that someone leaving a large company would do. Further, it is very unlikely that someone seeking independence somewhere else would also be looking for conformity, or rulefollowing sameness.4 Explanation for Correct Answer B. Choice (B) is correct. A vestige is a trace or mark left by something that no longer exists; a "vestigial" part of an animal's body is an undeveloped part that no longer functions as it once did (for the animal's ancestors). The structure of the sentence indicates that the missing term, which will describe the "kiwi bird's wings," refers to wings that are rudimentary, or only somewhat developed, and non-functioning. The term "vestigial" logically completes the sentence. Vestigial wings would be essentially leftover, non-functioning body parts that are a trace or mark of the bird's evolutionary past. Kiwi birds once had fully developed wings and, likely, the ability to fly; now, however, their wings are rudimentary and do not allow flight.. Choice (A) is incorrect. "Ostentatious" means showy or pretentious. The structure of the sentence indicates that the missing term, which will describe the kiwi bird's wings, refers to wings that are rudimentary, or only somewhat developed, and non-functioning. The bird's wings are not described as showy, so the term "ostentatious" does not logically complete the sentence. Instead, the term "vestigial" fits the blank: vestigial wings would be essentially leftover, nonfunctioning body parts that are a trace or mark of the bird's evolutionary past. Kiwi birds once had fully developed wings and, likely, the ability to fly; now, however, their wings are rudimentary and do not allow flight.. Choice (C) is correct. "Prodigious" typically means enormous in bulk, quality, or degree. The term can also be used to describe things that inspire amazement. The structure of the sentence indicates that the missing term, which will describe the kiwi bird's wings, refers to wings that are rudimentary, or only somewhat developed, and non-functioning. The bird's wings are not described as being enormousindeed, they are only somewhat developed and are likely smallor as being amazing, so the term "prodigious" does not logically complete the sentence. Instead, the term "vestigial" fits the blank: vestigial wings would be essentially leftover, non-functioning body parts that are a trace or mark of the kiwi bird's evolutionary past. Kiwi birds once had fully developed wings and, likely, the ability to fly; now, however, their wings are rudimentary and do not allow flight.. Choice (D) is incorrect. "Invasive" means aggressive or tending to infringe upon or invade something. The structure of the sentence indicates that the missing

term, which will describe the kiwi bird's wings, refers to wings that are rudimentary, or only somewhat developed, and non-functioning. It does not make sense to suggest that such wings are aggressive or infringe upon anything, so the term "invasive" does not logically complete the sentence. Instead, the term "vestigial" fits the blank: vestigial wings would be essentially leftover, nonfunctioning body parts that are a trace or mark of the kiwi bird's evolutionary past. Kiwi birds once had fully developed wings and, likely, the ability to fly; now, however, their wings are rudimentary and do not permit flight.. Choice (E) is incorrect. Something that is "kinetic" is active, lively, or in motion. The structure of the sentence indicates that the missing term, which will describe the kiwi bird's wings, refers to wings that are rudimentary, or only somewhat developed, and non-functioning. The sentence indicates that the bird's wings do not function, so it is unlikely that they would be described as kinetic; further, being rudimentary has nothing to do with being kinetic. The term vestigial describes the kiwi birds wings: vestigial wings would be essentially leftover, nonfunctioning body parts that are a trace or mark of the kiwi bird's evolutionary past. Kiwi birds once had fully developed wings and, likely, the ability to fly; now, however, their wings are rudimentary and do not allow flight.5 Explanation for Correct Answer D. Choice (D) is correct. To "denounce" something is to criticize it strongly. "Pragmatic" means practical. The sentence refers to the beliefs and actions of a group of "idealists," or people who favor their own standards of perfection over practical concerns. Idealists would certainly denounce, or criticize, any assertion that political enterprises should be purely pragmatic, or practical.. Choice (A) is incorrect. To "ignore" something is to dismiss or neglect it. "Universal" means applying to everyone. The sentence refers to the beliefs and actions of a group of "idealists," or people who favor their own standards of perfection over practical concerns. Some idealists might not want certain enterprises, or undertakings, to be adopted universally, but there is no reason to believe that idealists would necessarily ignore any assertion that political enterprises should be purely universal. On the other hand, idealists would certainly denounce, or criticize, any assertion that political enterprises should be purely pragmatic, or practical.. Choice (B) is incorrect. To "criticize" something is to speak negatively of it. "Visionary" means far-sighted or forward-thinking. The sentence refers to the beliefs and actions of a group of "idealists," or people who favor their own standards of perfection over practical concerns. Idealists are probably more likely than others to support political enterprises, or undertakings, that are forwardthinking, so there is no reason to believe that idealists would speak negatively of any assertion that political enterprises should be purely visionary. On the other hand, idealists would certainly denounce, or criticize, any assertion that political enterprises should be purely pragmatic, or practical.. Choice (C) is incorrect. To "condemn" something is to criticize it or pronounce it unfit. "Benevolent" means focused on doing good. The sentence refers to the beliefs and actions of a group of "idealists," or people who favor their own standards of perfection over practical concerns. Idealists are probably more likely than others to support political enterprises, or undertakings, that are focused on doing good, so there is no reason to believe that idealists would condemn any assertion that political enterprises should be purely benevolent. On the other hand, idealists would certainly denounce, or criticize, any assertion that political enterprises should be purely pragmatic, or practical..

Choice (E) is incorrect. To "condone" something is to tolerate or accept it. "Indulgent" means lenient or likely to let others have anything they want. The sentence refers to the beliefs and actions of a group of "idealists," or people who favor their own standards of perfection over practical concerns. There is no direct connection between being an idealist and supporting political enterprises, or undertakings, that are lenient, so there is no reason to say that idealists would necessarily condone, or tolerate, any assertions that political enterprises should be indulgent. On the other hand, idealists would certainly denounce, or criticize, any assertion that political enterprises should be purely pragmatic, or practical. 6 Explanation for Correct Answer E. Choice (E) is correct. Something that is strident is harsh, loud, or quarrelsome. The phrase in stark contrast indicates that the tone of the writers work is very different from his "disposition," or his usual mood. Therefore, if the writers disposition is gentle, or free from harshness and violence, then the tone of his writing would likely be characterized as harsh or violent. It makes sense to conclude that there is a strident tone in much of the writers work.. Choice (A) is incorrect. Something that is benign has a mild or favorable character. The phrase in stark contrast indicates that the tone of the writers work is very different from his disposition, or his usual mood; therefore, if the writers disposition is gentle, or free from harshness and violence, then the tone of his writing would likely be characterized as harsh or violent. The term benign does not logically complete the sentence; the writers mild tone would not contrast at all with his gentle disposition.. Choice (B) is incorrect. To be somber is to be very serious and sometimes gloomy. The phrase in stark contrast indicates that the tone of the writers work is very different from his disposition, or his usual mood; therefore, if the writers disposition is gentle, or free from harshness and violence, then the tone of his writing would likely be characterized as harsh or violent. The term somber does not logically complete the sentence. A writer could be very serious and have a gentle disposition, so a somber tone does not necessarily contrast with a gentle mood. Further, a serious tone is not necessarily a harsh tone.. Choice (C) is incorrect. To be stoic is to be characterized by a lack of feeling or passion. The phrase in stark contrast indicates that the tone of the writers work is very different from his disposition, or his usual mood; therefore, if the writers disposition is gentle, or free from harshness and violence, then the tone of his writing would likely be characterized as harsh or violent. The term stoic does not logically complete the sentence. A writer could have a gentle disposition and write with a tone that lacks passion. Further, whereas a stoic tone would be restrained rather than emotional, a harsh tone would most likely express strong feelings.. Choice (D) is incorrect. To be conciliatory is to be agreeable or friendly. The phrase in stark contrast indicates that the tone of the writers work is very different from his disposition, or his usual mood; therefore, if the writers disposition is gentle, or free from harshness and violence, then the tone of his writing would likely be characterized as harsh or violent. The term conciliatory does not logically complete the sentence; the writers agreeable tone would not contrast at all with his gentle disposition.7 Explanation for Correct Answer D. Choice (D) is correct. Someone who is resolute has a firm determination. To be polemical is to be argumentative. The structure of the sentence indicates that the part of the sentence after the colon defines the terms that will fit the blanks.

Peter held fast to his beliefs, so it makes sense to describe him as resolute; someone with a firm determination is likely to hold onto his or her beliefs. Even though he had firm beliefs, Peter avoided arguing about them with others, so it makes sense to say that he was not polemical; someone who is argumentative is unlikely to avoid arguing, as Peter did. The sentence suggests that Peter was resolute but not polemical.. Choice (A) is incorrect. Someone who is decisive is determined and acts firmly and unhesitatingly. A philanthropic person takes action to promote the good of others. The structure of the sentence indicates that the part of the sentence after the colon defines the terms that will fit the blanks. The sentence states that Peter held fast to his beliefs, so he might be described as decisive; someone who is determined and unhesitating would likely hold onto his or her beliefs. However, there is no reason to believe Peter was decisive without being philanthropic. The sentence indicates that Peter avoided arguing about beliefs, but there is no direct connection between avoiding such arguments and not taking actions to promote the good of others.. Choice (B) is incorrect. A haphazard person lacks a plan, order, or direction. To be quarrelsome is to be contentious or likely to get in an argument. The structure of the sentence indicates that the part of the sentence after the colon defines the terms that will fit the blanks. The sentence states that Peter avoided arguing about [his beliefs] with others. If Peter avoided arguments, it makes sense to say that he was not contentions, so the term quarrelsome fits the second blank. However, the term haphazard does not fit the first blank. The sentence indicates that Peter held fast to his beliefs, whereas a haphazard person would seem unlikely to stick to his or her beliefs.. Choice (C) is incorrect. Someone who is sentimental is directed by emotions and feelings rather than reason. A litigious person is likely to pursue lawsuits. The structure of the sentence indicates that the part of the sentence after the colon defines the terms that will fit the blanks. The terms sentimental and litigious do not logically complete the sentence. The sentence indicates that Peter held fast to his beliefs; it is possible that Peter behaved this way because he was sentimental, but there is not necessarily a connection between being directed by emotions and holding fast to beliefs. In fact, a person who is directed by emotions might change his or her beliefs when his or her feelings change. Further, the sentence states that Peter avoided arguing about [his beliefs] with others; although someone who refrains from being litigious would certainly avoid pursuing lawsuits, he or she would not necessarily avoid arguments altogether. A person could be argumentative without being litigious.. Choice (E) is incorrect. A steadfast person is firm in his or her beliefs or determinations. To be acquiescent is to accept or agree with something without resistance. The structure of the sentence indicates that the part of the sentence after the colon defines the terms that will fit the blanks. The sentence states that Peter held fast to his beliefs, so it makes sense to describe him as steadfast. However, the term acquiescent does not logically complete the sentence. The sentence indicates that Peter avoided arguing about [his beliefs] with others. People who readily agree with things might be likely to avoid arguments; if Peter refrained from being acquiescent, he might actually have been likely to confront others.8 Explanation for Correct Answer B. Choice (B) is correct. To be economical is to be characterized by careful and efficient use of resources. The structure of the sentence indicates that Lucille Cliftons style involves using a few unadorned words to [achieve] great

impact. It makes sense to describe Cliftons style as economical; her poems might be made up of relatively few words, but her careful and efficient use of those words allows her work to have great impact.. Choice (A) is incorrect. If something is incantatory it is part of, or suggestive of, a ritual of magic involving a recited formula of words. The structure of the sentence indicates that Lucille Cliftons style involves using a few unadorned words to [achieve] great impact. An incantation, or recitation related to a magic ritual, might be short but is not necessarily made up of only a few words; the term incantatory does not necessarily describe a style that involves using few words. Further, there is no indication that Cliftons poetry has any connection with magic rituals.. Choice (C) is incorrect. To be disaffected is to be rebellious or discontented and resentful. The structure of the sentence indicates that Lucille Cliftons style involves using a few unadorned words to [achieve] great impact. It is possible that a discontented writer might use only a few words, but he or she might also write texts expressing resentful feelings using many words; the term disaffected does not necessarily describe a style that involves using few words. Further, there is no indication that Cliftons poetry has anything to do with being rebellious or resentful.. Choice (D) is incorrect. To be unstinting is to not hold back or to give generously. The structure of the sentence indicates that Lucille Cliftons style involves using a few unadorned words to [achieve] great impact. It is somewhat illogical to describe Cliftons style as unstinting because an unstinting writer likely would not hold anything back; he or she might use many words rather than just a few. Cliftons style is best described as economical, or careful and efficient.. Choice (E) is incorrect. To be evenhanded is to be fair or impartial. The structure of the sentence indicates that Lucille Cliftons style involves using a few unadorned words to [achieve] great impact. A writer who is evenhanded might use only a few words, but he or she might also write texts using many words; the term evenhanded does not necessarily describe a style that involves using few words. Cliftons style is best described as economical, or careful and efficient, not evenhanded.9 Explanation for Correct Answer B. Choice (B) is correct. In the first sentence of Passage 1, the author explicitly states that chimpanzees exhibit behavior . . . remarkably similar to some human behavior. The author of Passage 2 also supports this idea. He or she refers to wild chimpanzees exhibiting human-like behavior, saying that they use tools and solve problems and noting that they are playful with their offspring. The author of Passage 2 draws a direct comparison when he or she states that they are emotional beings, as we [humans] are, with emotions . . . so obviously similar to ours. Both passages support the generalization that the behavior of wild chimpanzees often resembles that of humans.. Choice (A) is incorrect. The author of Passage 2 describes chimpanzees and their offspring, describing an adult male chasing a toddler round . . . a tree and a mother nibbling her babys toes; these descriptions suggest that the relationship between a parent and a child in both human and chimpanzee communities may be similar. However, the author of Passage 1 does not address the family structures of either chimpanzees or humans. He or she states that chimpanzees exhibit behavior . . . remarkably similar to some human behavior

but does not suggest that this behavior has anything to do with family structures.. Choice (C) is incorrect. The author of Passage 2 indirectly suggests that chimpanzees behavior might be prompted by psychological urges; he or she refers to wild chimpanzees as emotional beings and after watching their behavior concludes that They feel! However, Passage 1 does not support the idea that chimpanzees actions are the result of strong psychological urges. The author of Passage 1 writes about empathy, the act of identifying with the feelings or thoughts of another, which might be described as a psychological urge, but he or she is referring to humansspecifically, scientistsdeveloping empathy with chimpanzees when observing them. The author of Passage 1 does not refer to chimpanzees psychological urges.. Choice (D) is incorrect. Neither passage supports the idea that wild chimpanzees facial expressions can be interpreted accurately by nonspecialists. The author of Passage 2 is a nonspecialista nonscientistand he or she does describe a mother chimpanzee as looking blissful; however, there is no evidence in the passage that the authors interpretation of the chimpanzees facial expression is correct. The author of Passage 1 is talking about scientists, not nonspecialists, observing [chimpanzees] in the wild. He or she suggests that these specialists might interpret chimpanzees actions inaccurately if their interpretations are based on intuition and empathy; interpretations must be tested afterward against the facts set out in the data.. Choice (E) is incorrect. Neither passage supports the idea that the reactions of wild chimpanzees differ from those of other apes. Both passages make connections between chimpanzee behavior and human behavior, but neither mentions the behavior or reactions of other apes or other nonhuman animals. 10 Explanation for Correct Answer D. Choice (D) is correct. The purpose of Passage 2 is to convey the author's "discovery" about chimpanzees: "They feel!" The first two sentences of the passage indicate that the author's emphasis is on what he or she learned: the chimpanzees showed the author a richer and more satisfying world than [the author] had imagined, and the author suddenly recognized why he or she should care." The final two sentences relate the authors unexpected but obviously memorable insight about chimpanzees: They feel! That was my discovery.. Choice (A) is incorrect. The purpose of Passage 2 is not to describe a typical day's work on a scientific project. It is not clear that the author is working on a project, and the author explicitly states that he or she is a non-scientist. Further, the day the author describes is clearly not a typical day, but rather a special and unusual day: "My first day . . . showed me a richer and more satisfying world than I had imagined. I suddenly recognized why I . . . should care." The purpose of Passage 2 is to convey the author's "discovery" about chimpanzees: They feel!. Choice (B) is incorrect. The purpose of Passage 2 is not to describe an attempt to train chimpanzees; the author reports that he or she is "observing a community of forest chimpanzees," but there is no indication that he or shea non-scientist is training the chimpanzees. Instead, the purpose of Passage 2 is to convey the author's "discovery" about chimpanzees: "They feel!". Choice (C) is incorrect. The purpose of Passage 2 is not to propose an alternative method of studying animal behavior. The author is a non-scientist and indicates only that he or she is observing a community of forest chimpanzees. The

authors impressionistic approach may be different from some scientists methods of studying animal behavior, but he or she does not seem to be proposing an alternative approach to such study. Rather, the author is simply conveying his or her "discovery" about chimpanzees: "They feel!". Choice (E) is incorrect. The purpose of Passage 2 is not to relive an unusual and fleeting experience. Although the author is recalling an experience from one day (first day observing . . . forest chimpanzees), he or she seems more concerned with communicating information to others than with reliving the fleeting experience. The first two sentences of the passage indicate that the author's emphasis is on what he or she learned: the chimpanzees showed the author a richer and more satisfying world than [the author] had imagined, and the author suddenly recognized why he or she should care." The final two sentences relate the author's insight about chimpanzees: "They feel! That was my discovery."11 Explanation for Correct Answer A. Choice (A) is correct. In lines 8-10, the author of Passage 1 warns scientists about the danger of attributing human motivations to chimpanzees. Interestingly, the author of Passage 2 seems to commit the very error the author of Passage 1 is warning readers about, claiming that chimpanzees "are emotional beings, as we [humans] are and noting that their emotions are so obviously similar to ours." The experience described in Passage 2 suggests that the author of Passage 1 was right; observers of chimpanzees are clearly tempted to see these animals as having human emotions. The statement in Passage 1, then, accurately assesses the strength of a common human impulsethe impulse to attribute human characteristics to animals.. Choice (B) is incorrect. In lines 8-10, the author of Passage 1 warns scientists about the danger of attributing human motivations to chimpanzees. Interestingly, the author of Passage 2 seems to commit the very error the author of Passage 1 is warning readers about, claiming that chimpanzees "are emotional beings, as we [humans] are and noting that their emotions are so obviously similar to ours." The experience described in Passage 2 suggests that the author of Passage 1 was right; observers of chimpanzees are clearly tempted to see these animals as having human emotions. The statement in Passage 1 does not mention any actual physical dangers or risks a person faces when interacting with chimpanzees.. Choice (C) is incorrect. In lines 8-10, the author of Passage 1 warns scientists about the danger of attributing human motivations to chimpanzees. Interestingly, the author of Passage 2 seems to commit the very error the author of Passage 1 is warning readers about, claiming that chimpanzees "are emotional beings, as we [humans] are and noting that their emotions are so obviously similar to ours." The experience described in Passage 2 suggests that the author of Passage 1 was right; observers of chimpanzees are clearly tempted to see these animals as having human emotions. The statement in Passage 1 does not address the issue of keeping one's research current.. Choice (D) is incorrect. In lines 8-10, the author of Passage 1 warns scientists about the danger of attributing human motivations to chimpanzees. Interestingly, the author of Passage 2 seems to commit the very error the author of Passage 1 is warning readers about, claiming that chimpanzees "are emotional beings, as we [humans] are and noting that their emotions are so obviously similar to ours." The experience described in Passage 2 suggests that the author of Passage 1 was right; observers of chimpanzees are clearly tempted to see these animals as having human emotions. Although the author of Passage 2 may be faulted for

having an overly optimistic view of chimpanzees, the statement in Passage 1 does not discourage observers from expecting to make new discoveries.. Choice (E) is incorrect. In lines 8-10, the author of Passage 1 warns scientists about the danger of attributing human motivations to chimpanzees. Interestingly, the author of Passage 2 seems to commit the very error the author of Passage 1 is warning readers about, claiming that chimpanzees "are emotional beings, as we [humans] are and noting that their emotions are so obviously similar to ours." The experience described in Passage 2 suggests that the author of Passage 1 was right; observers of chimpanzees are clearly tempted to see these animals as having human emotions. The experience described in Passage 2 does not indicate that the author of Passage 1 fails to consider the consequences of a flawed research methodology; the author of Passage 1 seems quite aware of the consequences of the error of attributing human characteristics to animals.12 Explanation for Correct Answer E. Choice (E) is correct. The author of Passage 1 warns scientists about the possibility of misinterpreting chimpanzee behavior due to empathy ("empathy" refers to understanding, or believing that one understands, the feelings or thoughts of another). The last sentence of Passage 1 states that "interpretations" of chimpanzee behavior "may initially be based on . . . empathy, but they must be tested afterward against the facts set out in the data." As lines 26-27 indicate ("They feel! That was my discovery."), the author of Passage 2 is doing exactly what the author of Passage 1 warns scientists aboutinterpreting chimpanzee behavior based on the belief that he or she understands the chimpanzees feelings or thoughts. The author of Passage 1 would most likely caution the author of Passage 2 to test this empathy-based interpretation "against the facts set out in the data"in other words, to verify his or her conclusion with facts.. Choice (A) is incorrect. The author of Passage 1 would not applaud the author of Passage 2 for maintaining scientific objectivity. Passage 1 warns scientists about the possibility of misinterpreting chimpanzee behavior due to empathy ("empathy" refers to understanding, or believing that one understands, the feelings or thoughts of another). The last sentence of Passage 1 states that "interpretations" of chimpanzee behavior "may initially be based on . . . empathy, but they must be tested afterward against the facts set out in the data." As lines 26-27 indicate ("They feel! That was my discovery."), the author of Passage 2 is doing exactly what the author of Passage 1 warns scientists about not maintaining objectivity when he or she interprets chimpanzee behavior with the belief that he or she understands the chimpanzees feelings or thoughts. The author of Passage 1 would caution the author of Passage 2 to test his or her empathy-based interpretation "against the facts set out in the data"in other words, to verify his or her conclusion with facts.. Choice (B) is incorrect. The author of Passage 1 would not chide the author of Passage 2 for not submitting his or her findings for scientific review; the author of Passage 2 states the he or she is a non-scientist, and there is no reason to believe he or she is producing scientific findings that would require review. Passage 1 warns scientists about the possibility of misinterpreting chimpanzee behavior due to empathy ("empathy" refers to understanding, or believing that one understands, the feelings or thoughts of another). As lines 26-27 indicate ("They feel! That was my discovery."), the author of Passage 2 is doing exactly what the author of Passage 1 warns scientists aboutinterpreting chimpanzee behavior based on the belief that he or she understands the chimpanzees feelings or thoughts. The author of Passage 1 would caution the author of Passage 2 to test this empathy-based interpretation "against the facts set out in the data"in other words, to verify his or her conclusion with facts..

Choice (C) is incorrect. The author of Passage 1 would not criticize the author of Passage 2 for having poorly defined research goals; the author of Passage 2 states that he or she is a non-scientist and does not indicate that he or she is doing official research. Passage 1 warns scientists about the possibility of misinterpreting chimpanzee behavior due to empathy ("empathy" refers to understanding, or believing that one understands, the feelings or thoughts of another). As lines 26-27 indicate ("They feel! That was my discovery."), the author of Passage 2 is doing exactly what the author of Passage 1 warns scientists aboutinterpreting chimpanzee behavior based on the belief that he or she understands the chimpanzees feelings or thoughts. The author of Passage 1 would caution the author of Passage 2 to test this empathy-based interpretation "against the facts set out in the data"in other words, to verify his or her conclusion with facts.. Choice (D) is incorrect. The author of Passage 1 would not urge the author of Passage 2 to rely less on observations made in the wild. The author of Passage 1 does not seem to be concerned with all observations made in the wild; he or she is specifically concerned about the possibility of scientists misinterpreting chimpanzee behavior due to empathy (empathy refers to understanding, or believing that one understands, the feelings or thoughts of another) when observing chimpanzees in the wild. The author of Passage 1 states that "interpretations" of chimpanzee behavior "may initially be based on . . . empathy, but they must be tested afterward against the facts set out in the data." As lines 26-27 indicate ("They feel! That was my discovery."), the author of Passage 2 is doing exactly what the author of Passage 1 warns scientists aboutinterpreting chimpanzee behavior based on the belief that he or she understands the chimpanzees feelings or thoughts. The author of Passage 1 would caution the author of Passage 2 to test this empathy-based interpretation "against the facts set out in the data"in other words, to verify his or her conclusion with facts. 13 Explanation for Correct Answer D. Choice (D) is correct. Although the first paragraph describes the euphoria of observing bioluminescence, the passage as a whole is an overview of the various ways sea creatures use this characteristic to their advantage. Noctiluca algae use bioluminescence to discourage copepods from eating them; deep-sea anglerfish and Malacosteus use bioluminescence to attract or locate prey; other creatures use bioluminescence for camouflage. The general purpose of the passage seems to be to answer the question "Why do some sea creatures emit light?" by providing examples of the various functions of bioluminescence.. Choice (A) is incorrect. The passage does mention types of bioluminescence that some people might consider "unusual," but the passage is not a case study of one instance of bioluminescence, unusual or otherwise. Rather, the passage as a whole is an overview of the different ways sea creatures use bioluminescence to their advantage: Noctiluca algae use it to discourage copepods from eating them; deep-sea anglerfish and Malacosteus use it to attract or locate prey; other creatures use it for camouflage. The general purpose of the passage seems to be to answer the question "Why do some sea creatures emit light?" by providing examples of the various functions of bioluminescence.. Choice (B) is incorrect. The passage does not address any popular misconceptions about bioluminescence and its functions. Rather, the passage as a whole is an overview of the different ways sea creatures use bioluminescence to their advantage. Noctiluca algae use it to discourage copepods from eating them; deep-sea anglerfish and Malacosteus use it to attract or locate prey; other creatures use it for camouflage. The general purpose of the passage seems to be

to answer the question "Why do some sea creatures emit light?" by providing examples of the various functions of bioluminescence.. Choice (C) is incorrect. The passage does not address the evolutionary origins of bioluminescence; one might assume that some sea creatures evolved to become luminescent, but the passage does not discuss this idea. Instead, the passage as a whole is an overview of the different ways sea creatures use bioluminescence to their advantage. Noctiluca algae use it to discourage copepods from eating them; deep-sea anglerfish and Malacosteus use it to attract or locate prey; other creatures use it for camouflage. The general purpose of the passage seems to be to answer the question "Why do some sea creatures emit light?" by providing examples of the various functions of bioluminescence.. Choice (E) is incorrect. The passage does address the use of bioluminescence for the purpose of camouflage, but only in the last paragraph. The passage as a whole is best described as an overview of the different ways sea creatures use bioluminescence to their advantage; camouflage is just one of the ways. Noctiluca algae use bioluminescence to discourage copepods from eating them; deep-sea anglerfish and Malacosteus use it to attract or locate prey; other creatures use it for camouflage. The general purpose of the passage seems to be to answer the question "Why do some sea creatures emit light?" by providing examples of the various functions of bioluminescence.14 Explanation for Correct Answer E. Choice (E) is correct. Something that is granted is bestowed or permitted as a right, privilege, or favor. In lines 1-2, the author refers to "Sailing at night in luminescent seas" as a kind of privilege or favor that a person would be fortunate to receive; according to the author, this experience is "something splendid that is not given to everyone." In this context, "given" most nearly means granted.. Choice (A) is incorrect. In this context, to be inclined is to tend to favor or to lean toward a certain action or belief. In lines 1-2, the author refers to "Sailing at night in luminescent seas" as a kind of privilege that a person would be fortunate to receive. According to the author, this experience is "something splendid that is not given to everyone." It does not make sense to suggest that sailing through luminescent seas is an experience that is not inclined to everyone; an experience such as sailing is not something that can choose to favor someone or lean towards an action. In this context, given does not mean inclined.. Choice (B) is incorrect. Something that is "transported" is carried or moved from one place to another. In lines 1-2, the author refers to "Sailing at night in luminescent seas" as a kind of privilege that a person would be fortunate to receive. According to the author, this experience is "something splendid that is not given to everyone." It does not make sense to suggest that sailing through luminescent seas is an experience that is not transported to everyone; it is unclear how such an experience could be carried or moved to another location. In this context, given does not mean transported.. Choice (C) is incorrect. To be "devoted" is to be loyally committed to something or someone. In lines 1-2, the author refers to "Sailing at night in luminescent seas" as a kind of privilege that a person would be fortunate to receive. According to the author, this experience is "something splendid that is not given to everyone." It does not make sense to suggest that sailing through luminescent seas is an experience that is not devoted to everyone; an experience such as sailing is not something that can be loyal to a person or thing. In this context, "given" does not mean devoted..

Choice (D) is incorrect. In this context, something that is applied is put in effect or brought into action. In lines 1-2, the author refers to "Sailing at night in luminescent seas" as a kind of privilege or favor that a person would be fortunate to receive. According to the author, this experience is "something splendid that is not given to everyone." It is somewhat illogical to suggest that sailing through luminescent seas is an experience that is not applied to everyone; people can choose to go sailing, and the observation of bioluminescence seems to be a matter of chance, so the experience of sailing through luminescent seas would not be something that is put in effect for people. In this context, given does not mean applied.15 Explanation for Correct Answer E. Choice (E) is correct. In lines 1-4 the author describes the experience of Sailing at night in luminescent seas . . . . On a quiet night, with just enough wind to ghost along without the engine." Ghosts are commonly thought of or imagined as moving silently, so the phrase ghost along primarily suggests silent travel; the author is indicating that the boat moved along the water silently. The references to a quiet night and sailing without the engine further emphasize the silence of the experience.. Choice (A) is incorrect. Nothing in the second sentence of the passage suggests that the appearance of a boat sailing at night is misleading. When the author describes the experience of Sailing at night in luminescent seas he or she emphasizes the silence of the experience, referring to "a quiet night, with just enough wind to ghost along without the engine." Ghosts are commonly thought of or imagined as moving silently, so the phrase ghost along primarily suggests silent travel. There is no indication that the boat looks like anything but a boat.. Choice (B) is incorrect. Nothing in the second sentence of the passage suggests that the boat's motion is labored, or difficult. When the author describes the experience of Sailing at night in luminescent seas he or she emphasizes the silence of the experience, referring to a quiet night, with just enough wind to ghost along without the engine. Ghosts are commonly thought of or imagined as moving silently, so the phrase ghost along primarily suggests silent travel. There is no indication that the boats motion is labored; in fact, it is likely that the engine wouldbe used if the boat were moving along with difficulty.. Choice (C) is incorrect. Nothing in the second sentence of the passage suggests that sailing is an unexplained phenomenon; the author indicates that the boat is being propelled by the wind. Rather than suggesting that the boat is moving due to an unusual and inexplicable occurrence, the phrase ghost along simply suggests that the boat is moving without making any sound; ghosts are commonly thought of or imagined as moving silently. The author further emphasizes the silence of the experience of Sailing at night in luminescent seas when he or she refers to a quiet night and sailing without the engine.. Choice (D) is incorrect. Nothing in the second sentence of the passage suggests that sailing has alarming effects; indeed, the author is euphoric, not alarmed. When the author describes the experience of Sailing at night in luminescent seas he or she emphasizes the silence of the experience, referring to a quiet night, with just enough wind to ghost along without the engine." Ghosts are commonly thought of or imagined as moving silently, so the phrase ghost along primarily suggests silent travel. There is no indication that the experience of sailing at night causes the author to be alarmed.16 Explanation for Correct Answer C.

Choice (C) is correct. To become acclimated is to become accustomed to a new environment. In the first paragraph the author is speaking of sailing at night in luminescent seas. In describing this euphoric experience, he or she says that at night, on the water, it takes a little time to become adapted to the dark and to realize how much is going on in the sea around you. The author is describing a gradual adjustment to unfamiliar conditions, resulting in increasing awareness of the environment. In line 7, the term adapted most nearly means acclimated.. Choice (A) is incorrect. The term "evolved" refers to something that has developed gradually over time. In the first paragraph the author is speaking of sailing at night in luminescent seas. In describing this euphoric experience, he or she says that at night, on the water, it takes a little time to become adapted to the dark and to realize how much is going on in the sea around you. The author is describing a gradual adjustment to unfamiliar conditions, resulting in increasing awareness of the environment. The term evolved can be used to describe something that has adapted, but it almost always refers to development over a longer time period than the one described in the passagethink, for instance, of the scientific meanings of evolution. As it is used in line 7, the term adapted most nearly means acclimated, not evolved.. Choice (B) is incorrect. To be "committed" is to be devoted to or promised to something. In the first paragraph the author is speaking of sailing at night in luminescent seas. In describing this euphoric experience, he or she says that at night, on the water, it takes a little time to become adapted to the dark and to realize how much is going on in the sea around you. The author is describing a gradual adjustment to unfamiliar conditions, resulting in increasing awareness of the environment. It does not make much sense to say that one would become committed to the dark. As it is used in line 7, the term adapted most nearly means acclimated, not committed.. Choice (D) is incorrect. To be predisposed is to be inclined or suited to something in advance. In the first paragraph the author is speaking of sailing at night in luminescent seas. In describing this euphoric experience, he or she says that at night, on the water, it takes a little time to become adapted to the dark and to realize how much is going on in the sea around you. The author is describing a gradual adjustment to unfamiliar conditions, resulting in increasing awareness of the environment. It does not make sense to say that one would become predisposed to the dark while sitting in a boat at night; it is illogical to suggest that someone could become suited to the dark in advance while he or she is in the dark. As it is used in line 7, the term adapted most nearly means acclimated, not predisposed.. Choice (E) is incorrect. The term "altered" refers to something that has changed. In the first paragraph the author is speaking of sailing at night in luminescent seas. In describing this euphoric experience, he or she says that at night, on the water, it takes a little time to become adapted to the dark and to realize how much is going on in the sea around you. The author is describing a gradual adjustment to unfamiliar conditions, resulting in increasing awareness of the environment. It does not make sense to say that the term adapted most nearly means altered in this context: one would not say that he or she became changed to the dark. In line 7, the term adapted most nearly means acclimated, not altered.17 Explanation for Correct Answer A. Choice (A) is correct. The more interesting question referred to in line 15 basically asks, Why does the tiny alga Noctiluca flash? The author goes on to discuss a possible answer: theNoctiluca flashes because its main predator, the

copepod, avoids flashing prey, not wanting to be "lit up by the glow in its gut" (after consuming the Noctiluca), thus attracting its own predators. The author does not explicitly state that the Noctilucas flashing serves to deter potential predators, but he or she suggests that this is the best answer to the question of why the alga flashesit is the most likely possibility (line 36).. Choice (B) is incorrect. The more interesting question referred to in line 15 basically asks, Why does the tiny alga Noctiluca flash? The author goes on to discuss a possible answer: theNoctiluca flashes because its main predator, the copepod, avoids flashing prey, not wanting to be "lit up by the glow in its gut" (after consuming the Noctiluca), thus attracting its own predators. The author does discuss creatures that use bioluminescence to see betterhe or she refers to the Malacosteus in the fourth paragraphbut the author does not suggest that enhanced vision is the answer to the more interesting question about the Noctiluca; rather, he or she suggests that the answer is that the algas flashing serves to deter potential predators.. Choice (C) is incorrect. The more interesting question referred to in line 15 basically asks, Why does the tiny alga Noctiluca flash? The author goes on to discuss a possible answer: the Noctiluca flashes because its main predator, the copepod, avoids flashing prey, not wanting to be "lit up by the glow in its gut" (after consuming the Noctiluca), thus attracting its own predators. Although the author does discuss creatures that use bioluminescence to attract mates (lines 41-46), he or she does not suggest that attraction of mates is the answer to the more interesting question about the Noctiluca; rather, he or she suggests that the answer is that the algas flashing serves to deter potential predators.. Choice (D) is incorrect. The more interesting question referred to in line 15 basically asks, Why does the tiny alga Noctiluca flash? The author goes on to discuss a possible answer: the Noctiluca flashes because its main predator, the copepod, avoids flashing prey, not wanting to be "lit up by the glow in its gut" (after consuming the Noctiluca), thus attracting its own predators. The author does discuss creatures that use bioluminescence to attract prey or find foodhe or she refers to anglerfish and Malacosteusbut the author does not suggest that the ability to find food is the answer to the more interesting question about the Noctiluca; rather, he or she suggests that the answer is that the algas flashing serves to deter potential predators.. Choice (E) is incorrect. The more interesting question referred to in line 15 basically asks, Why does the tiny alga Noctiluca flash? The author goes on to discuss a possible answer: theNoctiluca flashes because its main predator, the copepod, avoids flashing prey, not wanting to be "lit up by the glow in its gut" (after consuming the Noctiluca), thus attracting its own predators. Although the author of the passage does discuss bioluminescence as a mode of communication in his or her discussion of creatures who attract mates with lights (lines 41-46), he or she does not suggest that communication is the answer to the more interesting question about the Noctiluca; rather, the author suggests that the answer is that the algas flashing serves to deter potential predators. 18 Explanation for Correct Answer D. Choice (D) is correct. The question referred to asks why it would be to Noctilucas advantage to flash, when it might be suicidethat is, an invitation to predators to do so. The author goes on to answer that Noctilucas flashing is not in fact a liability, explaining that the algas predators, copepods, might avoid it for that very reason (the copepod may not wish to be "lit up by the glow in its gut"). The question itself, however, raises an apparent, or seeming, drawback to

luminescence in Noctiluca, pointing out that the Noctiluca could be near enough invisible but advertises its presence by flashing.. Choice (A) is incorrect. The question referred to asks why it would be to Noctilucas advantage to flash, when it might be suicidethat is, an invitation to predatorsto do so. The author goes on to answer that Noctilucas flashing is not in fact a liability, explaining that the algas predators, copepods, might avoid it for that very reason (the copepod may not wish to be "lit up by the glow in its gut"). If the answer to the question were unknown, the author might be suggesting that scientific understanding is limited, but the subsequent answer demonstrates that, indeed, a fair amount is known about Noctiluca.. Choice (B) is incorrect. The question referred to asks why it would be to Noctilucas advantage to flash, when it might be suicidethat is, an invitation to predatorsto do so. The author goes on to answer that Noctilucas flashing is not in fact a liability, explaining that the algas predators, copepods, might avoid it for that very reason (the copepod may not wish to be "lit up by the glow in its gut"). If the answer to the question were unknown, the author might be suggesting that there are flaws in the current research on Noctiluca; however, he or she goes on to answer the question and does not suggest there are flaws in the research.. Choice (C) is incorrect. The question referred to asks why it would be to Noctilucas advantage to flash, when it might be suicidethat is, an invitation to predatorsto do so. The author goes on to answer that Noctilucas flashing is not in fact a liability, explaining that the algas predators, copepods, might avoid it for that very reason (the copepod may not wish to be "lit up by the glow in its gut"). The question and its answer do not suggest that an adaptation (Noctilucas flashing) has no biological purpose; in fact, the author indicates that the opposite is true.. Choice (E) is incorrect. The question referred to asks why it would be to Noctilucas advantage to flash, when it might be suicidethat is, an invitation to predatorsto do so. The author goes on to answer that Noctilucas flashing is not in fact a liability, explaining that the algas predators, copepods, might avoid it for that very reason (the copepod may not wish to be "lit up by the glow in its gut"). The question and its answer do indicate that luminescence has a function for Noctiluca, but the only function described is that of avoiding predators; the author refers to only one function, not several functions, of Noctilucas flashing.19 Explanation for Correct Answer B. Choice (B) is correct. The author states that copepods do their best to be transparent, and then explains in paragraph 3 that the copepod behaves this way so that it can avoid its predatorsfish, for example. If a copepod were to consume the flashing Noctiluca, the copepod itself would flash and, likely, attract predators. During the course of his or her explanation, the author muses on the causes of copepods unlikely but consistent apparent dislike of eating Noctiluca, and ultimately speculates that it arises from the copepods wish to remain invisible.. Choice (A) is incorrect. The author states that copepods do their best to be transparent, and then explains in paragraph 3 that the copepod behaves this way so that it can avoid its predatorsfish, for example. If a copepod were to consume the flashing Noctiluca, the copepod itself would flash and, likely, attract predators. The author is discussing the copepods avoidance of the luminescent alga Noctiluca, but he or she does not indicate that copepods are luminescent.

Indeed, the statement that copepods try to be transparent suggests that they probably are not luminescent.. Choice (C) is incorrect. The author states that copepods do their best to be transparent, and then explains in paragraph 3 that the copepod behaves this way so that it can avoid its predatorsfish, for example. If a copepod were to consume the flashing Noctiluca, the copepod itself would flash and, likely, attract predators. The author does state in lines 20-21 that copepods are the most abundant small predators in the sea, but he or she does not refer to their attempts to be transparent in order to account for the size of the copepod population. The author discusses the reason for copepods avoidance of Noctiluca, not the reason for their relative abundance.. Choice (D) is incorrect. The author states that copepods do their best to be transparent, and then explains in paragraph 3 that the copepod behaves this way so that it can avoid its predatorsfish, for example. If a copepod were to consume the flashing Noctiluca, the copepod itself would flash and, likely, attract predators. Nowhere in the passage does the author make the point that copepods are more difficult to detect than are Noctiluca, so the reference to copepods attempts to be transparent does not reinforce this point.. Choice (E) is incorrect. The author states that copepods do their best to be transparent, and then explains in paragraph 3 that the copepod behaves this way so that it can avoid its predatorsfish, for example. If a copepod were to consume the flashing Noctiluca, the copepod itself would flash and, likely, attract predators. The author refers to the size of the copepod population (the most abundant small predators in the sea), but he or she does not address copepods success as predators or indicate that their attempts to be transparent help them find and consume food. The author discusses the reason for copepods avoidance of Noctiluca, not the reason for their success as predators.20 Explanation for Correct Answer B. Choice (B) is correct. In the third paragraph of the passage, the author gives a complex explanation for why copepods do not eat prey that flashes. The author indicates that the algaNoctiluca doesn't seem to taste nasty, or sting, or do anything unpleasant to the predator," and states that "there must be some other explanation" for why these algae are avoided by copepods. "The most likely possibility," according to the author, is that consuming flashing organisms makes copepods themselves flash, thus attracting predators. The author argues that copepods avoid Noctiluca because they want to remain invisible and avoid detection by predators.. Choice (A) is incorrect. In the third paragraph of the passage, the author gives a complex explanation for why copepods do not eat prey that flashes. The author indicates that the algaNoctiluca doesn't seem to taste nasty, or sting, or do anything unpleasant to the predator, and states that "there must be some other explanation" for why these algae are avoided by copepods. "The most likely possibility," according to the author, is that consuming flashing organisms makes copepods themselves flash, thus attracting predators. Nowhere in the passage does the author argue that copepods avoid flashing prey because the light confuses them.. Choice (C) is incorrect. In the third paragraph of the passage, the author gives a complex explanation for why copepods do not eat prey that flashes. The author indicates that the algaNoctiluca doesn't seem to taste nasty, or sting, or do anything unpleasant to the predator, and states that "there must be some other explanation" for why these algae are avoided by copepods. "The most likely

possibility," according to the author, is that consuming flashing organisms makes copepods themselves flash, thus attracting predators. Nothing in the passage indicates that copepods rarely encounter luminescence; in fact, given the prevalence of luminescent sea creatures discussed in the passage, copepods might encounter luminescence frequently.. Choice (D) is incorrect. In the third paragraph of the passage, the author gives a complex explanation for why copepods do not eat prey that flashes. The author indicates that the algaNoctiluca doesn't seem to do anything unpleasant to the predator, and states that "there must be some other explanation" for why these algae are avoided by copepods. "The most likely possibility," according to the author, is that consuming flashing organisms makes copepods themselves flash, thus attracting predators. Nothing in the passage indicates that luminescent animals generally taste bitter; indeed, the author explicitly states that one luminescent organismthe Noctilucaso far as we know, doesnt taste nasty.. Choice (E) is incorrect. In the third paragraph of the passage, the author gives a complex explanation for why copepods do not eat prey that flashes. The author indicates that the algaNoctiluca doesn't seem to taste nasty, or sting, or do anything unpleasant to the predator, and states that "there must be some other explanation" for why these algae are avoided by copepods. "The most likely possibility," according to the author, is that consuming flashing organisms makes copepods themselves flash, thus attracting predators. Nowhere in the passage does the author argue that copepods avoid flashing prey because the light temporarily blinds them.21 Explanation for Correct Answer C. Choice (C) is correct. The author explains that copepods dont like prey that flashes, and describes their avoidance of the luminescent organism Noctiluca: copepods [g]rab it, it explodes into light in their scratchy little arms, and they drop it. The author goes on to explain that this behavior seems odd because Noctiluca is not known to have a bad taste or a sting, and that practically every animal that has ever been studied habituates to stimuli that repeatedly prove harmlesslearns, in other words, to become accustomed to such harmless things as flashing lights. The author indicates that the copepods behavior is unusual because it seems to defy typical response patterns in animals.. Choice (A) is incorrect. The author explains that copepods dont like prey that flashes, and describes their avoidance of the luminescent organism Noctiluca: copepods [g]rab it, it explodes into light in their scratchy little arms, and they drop it. The author goes on to explain that this behavior seems odd because Noctiluca is not known to have a bad taste or a sting, and that most animals learn to become accustomed to harmless stimuli (such as flashing lights). He or she suggests that copepods behavior is unusual because it differs from the typical response pattern of practically every animal that has ever been studiedin other words, it differs from the behavior of most animals, not just aquatic animals; the author does not specifically address the rarity of this behavior in aquatic animals.. Choice (B) is incorrect. The author explains that copepods dont like prey that flashes, and describes their avoidance of the luminescent organism Noctiluca: copepods [g]rab it, it explodes into light in their scratchy little arms, and they drop it. The author goes on to explain that this behavior seems odd because Noctiluca is not known to have a bad taste or a sting, and that most animals learn to become accustomed to harmless stimuli (such as flashing lights). The author does not indicate that copepods behavior is unusual

because it is an example of a rapid response to prolonged, or ongoing, stimuli, but rather that their behavior is odd because practically every animal that has ever been studied habituates to stimuli that repeatedly prove harmless.". Choice (D) is incorrect. The author explains that copepods dont like prey that flashes, and describes their avoidance of the luminescent organism Noctiluca: copepods [g]rab it, it explodes into light in their scratchy little arms, and they drop it. The author goes on to explain that this behavior seems odd because Noctiluca is not known to have a bad taste or a sting, and that most animals learn to become accustomed to harmless stimuli (such as flashing lights). It seems that the copepods reaction causes it to lose foodit drops its prey when the prey flashesbut the author does not suggest that the copepods behavior is odd because it does not help the organism locate food. Indeed, the copepod must be able to locate food if it can grab the Noctiluca in the first place.. Choice (E) is incorrect. The author explains that copepods dont like prey that flashes, and describes their avoidance of the luminescent organism Noctiluca: copepods [g]rab it, it explodes into light in their scratchy little arms, and they drop it. The author goes on to explain that this behavior seems odd because Noctiluca is not known to have a bad taste or a sting, and that most animals learn to become accustomed to harmless stimuli (such as flashing lights). The author speaks about copepods reaction to flashing prey in a general way that suggests this behavior is normal for copepods; however, he or she does not specifically address the frequency of this behavior. The author simply suggests that copepods behavior is unusual because practically every animal that has ever been studied habituates to stimuli that repeatedly prove harmless."22 Explanation for Correct Answer A. Choice (A) is correct. In keeping with the conversational style of much of the passage, the author uses the expression fair bet in speaking of certain luminescent aquatic species that probably use light to signal sex difference, allowing males and females to find each other for mating. These species, which breed in very dark underwater spaces, have different lighting depending on the creatures sex ("the patterns of light differ between sexes"), so the author says that it is a fair betor an assumption likely to be truethat luminescence serves as an identification signal, a means of bringing the sexes together. The author assumes that the difference in lighting patterns exists for the purpose of bringing the sexes together, but he or she does not know for sure; the assumption is just a hypothesis, but one the author feels is most likely accurate.. Choice (B) is incorrect. In keeping with the conversational style of much of the passage, the author uses the expression fair bet in speaking of certain luminescent aquatic species that probably use light to signal sex difference, allowing males and females to find each other for mating. These species, which breed in very dark underwater spaces, have different lighting depending on the creatures sex ("the patterns of light differ between sexes"), so the author says that it is a fair betor an assumption likely to be truethat luminescence serves as an identification signal, a means of bringing the sexes together. The author assumes that the difference in lighting patterns exists for the purpose of bringing the sexes together, but he or she does not know for sure. Rather than making a prediction, the author is offering a plausible explanationa hypothesis that seems likely to be correct.. Choice (C) is incorrect. In keeping with the conversational style of much of the passage, the author uses the expression fair bet in speaking of certain

luminescent aquatic species that probably use light to signal sex difference, allowing males and females to find each other for mating. These species, which breed in very dark underwater spaces, have different lighting depending on the creatures sex ("the patterns of light differ between sexes"), so the author says that it is a fair betor an assumption likely to be truethat luminescence serves as an identification signal, a means of bringing the sexes together. The authors assumption that the difference in lighting patterns exists for the purpose of bringing the sexes together might be called a theory, because it is an unproven assumption; however, the phrase fair bet suggests that the assumption is likely correct, not that it is likely to be rejected by the reader or by the scientific community.. Choice (D) is incorrect. In keeping with the conversational style of much of the passage, the author uses the expression fair bet in speaking of certain luminescent aquatic species that probably use light to signal sex difference, allowing males and females to find each other for mating. These species, which breed in very dark underwater spaces, have different lighting depending on the creatures sex ("the patterns of light differ between sexes"), so the author says that it is a fair betor an assumption likely to be truethat luminescence serves as an identification signal, a means of bringing the sexes together. The author assumes that the difference in lighting patterns exists for the purpose of bringing the sexes together, but he or she does not know for sure. The author is not really referring to an event here, but rather to a hypothesisthat of luminescence being an identification signal.. Choice (E) is incorrect. In keeping with the conversational style of much of the passage, the author uses the expression fair bet in speaking of certain luminescent aquatic species that probably use light to signal sex difference, allowing males and females to find each other for mating. These species, which breed in very dark underwater spaces, have different lighting depending on the creatures sex ("the patterns of light differ between sexes"), so the author says that it is a fair betor an assumption likely to be truethat luminescence serves as an identification signal, a means of bringing the sexes together. The author assumes that the difference in lighting patterns exists for the purpose of bringing the sexes together, but he or she does not know for sure. The author is not indicating whether this assumptionwhich might be called an observation, or a remark based on something the author has observedwill be understood; the author is simply offering an explanation for a phenomenon and stating that the explanation is probably true.23 Explanation for Correct Answer D. Choice (D) is correct. The author calls Malacosteus an outstandingly devious group of deep-sea predatory fish [that] has developed a system of red searchlights that are probably invisible to everybody except themselves. Later, after revealing how this system works, the author says that Malacosteus has the jump on almost everybody around, implying that this species enjoys advantages that other species do not. In other words, Malacosteus differs from other species in that its luminescence is probably visible only to other members of its species.. Choice (A) is incorrect. The author calls Malacosteus an outstandingly devious group of deep-sea predatory fish [that] has developed a system of red searchlights that are probably invisible to everybody except themselves. Later, after revealing how this system works, the author says that Malacosteus has the jump on almost everybody around, implying that this species enjoys advantages that other species do not. In other words, Malacosteus differs from other species in that its luminescence is probably visible only to other members of its species. The author does say that Malacosteus is a deep-sea fish, but he or she does not

say that the creatures searchlights are effective only in deep water. In fact, the author indicates that red light generally is difficult to see in water, "even at the sea surface.". Choice (B) is incorrect. The author calls Malacosteus an outstandingly devious group of deep-sea predatory fish [that] has developed a system of red searchlights that are probably invisible to everybody except themselves. Later, after revealing how this system works, the author says that Malacosteus has the jump on almost everybody around, implying that this species enjoys advantages that other species do not. In other words, Malacosteus differs from other species in that its luminescence is probably visible only to other members of its species. The author does not indicate that Malacosteus luminescence temporarily startles other animals; in fact, the author states that it is unlikely that other sea creatures can see this luminescence (most mid-to-deep water marine animals have visual systems that cannot perceive red light).. Choice (C) is incorrect. The author calls Malacosteus an outstandingly devious group of deep-sea predatory fish [that] has developed a system of red searchlights that are probably invisible to everybody except themselves. Later, after revealing how this system works, the author says that Malacosteus has the jump on almost everybody around, implying that this species enjoys advantages that other species do not. In other words, Malacosteus differs from other species in that its luminescence is probably visible only to other members of its species. The author does not say that Malacosteus luminescence is more obvious near the surface; on the contrary, he or she indicates that red light is very difficult to see in water, even at the sea surface.. Choice (E) is incorrect. The author calls Malacosteus an outstandingly devious group of deep-sea predatory fish [that] has developed a system of red searchlights that are probably invisible to everybody except themselves. Later, after revealing how this system works, the author says that Malacosteus has the jump on almost everybody around, implying that this species enjoys advantages that other species do not. In other words, Malacosteus differs from other species in that its luminescence is probably visible only to other members of its species. The author discusses Malacosteus luminescence as an aid in finding prey, not as a form of protection from predators.24 Explanation for Correct Answer A. Choice (A) is correct. In the last paragraph of the passage the author compares the way in which a luminescent sea creature can use its light to avoid predators to a camouflage tactic employed during World War II. This tactic involved hiding a submarine-hunting plane from a targeted submarine that was recharging its batteries at the surfacemaking it so the unprotected submarine on the surface could not detect the aircraft until it was too late. A submarine is an underwater vehicle; when it comes temporarily to the surface, it loses its natural protection (the water in which it is usually submerged). This situation is most similar to a crab left temporarily unprotected by its natural protection (its shell).. Choice (B) is incorrect. In the last paragraph of the passage the author compares the way in which a luminescent sea creature can use its light to avoid predators to a camouflage tactic employed during World War II. This tactic involved hiding a submarine-hunting plane from a targeted submarine that was recharging its batteries at the surfacemaking it so the unprotected submarine on the surface could not detect the aircraft until it was too late. A submarine is an underwater vehicle, and when it comes temporarily to the surface it loses its natural protection (the water in which it is usually submerged): this situation is most similar to a crab left temporarily unprotected by its natural protection (its shell).

This situation is not similar to a shark continually foraging for food; a shark hunting in the ocean is an aggressive predator, not unprotected prey.. Choice (C) is incorrect. In the last paragraph of the passage, the author compares the way in which a luminescent sea creature can use its light to avoid predators to a camouflage tactic employed during World War II. This tactic involved hiding a submarine-hunting plane from a targeted submarine that was recharging its batteries at the surfacemaking it so the unprotected submarine on the surface could not detect the aircraft until it was too late. A submarine is an underwater vehicle, and when it comes temporarily to the surface it loses its natural protection (the water in which it is usually submerged): this situation is most similar to a crab left temporarily unprotected by its natural protection (its shell). This situation is not similar to a koala living in captivity in a zoo; a creature imprisoned outside its natural habitat is not necessarily unprotected prey.. Choice (D) is incorrect. In the last paragraph of the passage, the author compares the way in which a luminescent sea creature can use its light to avoid predators to a camouflage tactic employed during World War II. This tactic involved hiding a submarine-hunting plane from a targeted submarine that was recharging its batteries at the surfacemaking it so the unprotected submarine on the surface could not detect the aircraft until it was too late. A submarine is an underwater vehicle, and when it comes temporarily to the surface it loses its natural protection (the water in which it is usually submerged): this situation is most similar to a crab left temporarily unprotected by its natural protection (its shell). This situation is not similar to a bat emitting sounds to track an insect; a bat tracking an insect is an aggressive predator, not unprotected prey..

Choice (E) is incorrect. In the last paragraph of the passage, the author compares the way in which a luminescent sea creature can use its light to avoid predators to a camouflage tactic employed during World War II. This tactic involved hiding a submarine-hunting plane from a targeted submarine that was recharging its batteries at the surfacemaking it so the unprotected submarine on the surface could not detect the aircraft until it was too late. A submarine is an underwater vehicle, and when it comes temporarily to the surface it loses its natural protection (the water in which it is usually submerged): this situation is most similar to a crab left temporarily unprotected by its natural protection (its shell). This situation is not similar to a goose flying south for the winter; an animal temporarily changing its habitat is not necessarily disabling its natural defenses and exposing itself to danger. Section #5: View Explanations 1 Explanation for Correct Answer D. Choice (D) is correct. Opposition is the act of opposing, or being against, something. Something that is abandoned is given up or forsaken. The second part of the sentence reveals a cause and effect relationship: the action of the home owners caused something to happen to the proposal for a new property tax. The sentence indicates that the proposed tax had gained momentum, or strength. However, the term but suggests that for some reason the momentum stopped. It makes sense to suggest that great opposition from home owners could stop the momentum of a tax that would affect them, resulting in the proposal being abandoned..

Choice (A) is incorrect. Anticipation is the act of looking forward to something. To endorse something is to approve it openly. The second part of the sentence reveals a cause and effect relationship: the action of the home owners caused something to happen to the proposal for a new property tax, a proposal which had previously gained momentum, or strength. The term but suggests that for some reason the momentum stopped. If home owners were looking forward to the tax, it is possible they could have helped get the proposal endorsed, or approved. However, the term "but" makes the resulting sentence illogical; endorsement of the proposal is not evidence that the proposal lost momentum. Further, it is unlikely that the anticipation of home owners for a new property tax, which they would presumably have to pay, would have been "great.". Choice (B) is incorrect. In this context, publicity is information issued to gain public attention or support. To reveal something is to make it known or visible. The second part of the sentence reveals a cause and effect relationship: the action of the home owners caused something to happen to the proposal for a new property tax, a proposal which had previously gained momentum, or strength. However, the term but suggests that for some reason the momentum stopped. Negative publicity might have stopped the proposals momentum, but nothing in the sentence suggests that the home owners released negative information about the new tax. Further, positive publicity could have helped the proposals momentum. Additionally, there is no reason to suggest that the proposal had to be revealed because there is no indication that it was previously hidden; indeed, the home owners seem to have known about it.. Choice (C) is incorrect. In this context, a penalty is a punishment. To abridge something is to condense or shorten it by omitting words. The second part of the sentence reveals a cause and effect relationship: the action of the home owners caused something to happen to the proposal for a new property tax, a proposal which had previously gained momentum. However, the term but suggests that for some reason the momentum stopped. The terms penalty and abridged do not logically complete the sentence. The phrase penalty from home owners suggests that the home owners were issuing a punishment, but nothing in the sentence indicates who would have been punished, and there is no reason to believe that punishment would lead to the proposal being abridged. Additionally, condensing or reducing the length of a proposal for a new tax would not be a likely response to criticism unless the abridgement resulted in the tax being lessenedand this is not stated.. Choice (E) is incorrect. A backlash is a strong adverse reaction to something. To include something is to take it in and make it part of a whole or group. The second part of the sentence reveals a cause and effect relationship: the action of the home owners caused something to happen to the proposal for a new property tax, a proposal which had previously gained momentum. However, the term but suggests that for some reason the momentum stopped. If the home owners expressed a strong adverse reaction to the proposed new tax, it is possible that this backlash could have caused the proposal to lose momentum. But it does not make sense to suggest that a backlash would have caused the proposal to be included; an adverse reaction would be likely to cause the proposal to be excluded from something, not included. Further, there is no indication of what the proposal would be included in.2 Explanation for Correct Answer C. Choice (C) is correct. Something that is tedious is tiresome because of length or dullness. The structure of the sentence indicates that the information after the semi-colon elaborates on the information presented before the semi-colon.

Because the lecturers speech was so dull that some of the students nodded off, or fell asleep, it makes sense to describe the speech as tediousclearly, some of the students thought the speech was tiresome due to its dullness.. Choice (A) is incorrect. Something that is illuminating is clear and spiritually or intellectually enlightening. The structure of the sentence indicates that the information after the semi-colon elaborates on the information presented before the semi-colon. The sentence states that the lecturers speech was so dull that some of the students nodded off, or fell asleep. There is no reason to believe that an illuminating speech would be described as dull and would cause students to fall asleep. On the contrary, students might find a speech that is clear and enlightening to be interesting, not dull.. Choice (B) is incorrect. Something that is cryptic is mysterious or appears to have a hidden meaning. The structure of the sentence indicates that the information after the semi-colon elaborates on the information presented before the semi-colon. The sentence states that the lecturers speech was so dull that some of the students nodded off, or fell asleep. Although students might consider a mysterious speech to be dull, it is also possible that they would be interested and would want try to figure out its meaning. Because mysteriousness is not directly related to dullness, the term cryptic does not logically complete the sentence.. Choice (D) is incorrect. Something that is disjointed lacks coherence or an orderly sequence. The structure of the sentence indicates that the information after the semi-colon elaborates on the information presented before the semicolon. The sentence states that the lecturers speech was so dull that some of the students nodded off, or fell asleep. Students might consider an incoherent speech to be dull if they are unable to follow it and get meaning out of it. However, incoherence is not directly related to dullness, so the term disjointed does not logically complete the sentence.. Choice (E) is incorrect. Something that is unsettling has the effect of disturbing or upsetting someone or something. The structure of the sentence indicates that the information after the semi-colon elaborates on the information presented before the semi-colon. The sentence states that the lecturers speech was so dull that some of the students nodded off, or fell asleep. It is somewhat illogical to suggest that the students considered the speech to be unsettling; if Most of the students were disturbed or upset by the speech, it is unlikely that they would fall asleep during it or describe it as dull.3 Explanation for Correct Answer A. Choice (A) is correct. To contain something is to keep it within certain limits, or to halt it. In this context, to check something is to control it or lessen its force or power. The first part of the sentence indicates that epidemiologists were dealing with an infectious disease, but their attempts at doing something were futile, or ineffective. As scientists who study diseases and try to control them in populations, epidemiologists would likely attempt to limit the spread of a disease, so the term contain fits the first blank. The part of the sentence after the colon further explains how the scientists work ended up being futile, suggesting that the epidemiologists were able to control the disease in certain areas only to see the disease emerge somewhere else. It makes sense to say that the epidemiologists were able to check the disease in some areas.. Choice (B) is incorrect. To control something is to regulate it or have power over it. In this context, to extend something is to increase it in extent or scope. The first part of the sentence indicates that epidemiologists were dealing with an

infectious disease, but their attempts at doing something were futile, or ineffective. Epidemiologists are scientists who study diseases and try to control them in populations, so the term control fits the first blank; epidemiologists certainly would try to regulate the disease to stop it from spreading. However, if the epidemiologists were attempting to stop the spread of the disease, it does not make sense to say that they managed to extend it in an area, or succeeded in increasing the scope of the disease. Even if their attempts to control the spread of the disease were ineffective, it is very unlikely that epidemiologists would try to increase the scope of a disease they did not want to spread.. Choice (C) is incorrect. To eradicate something is to do away with it completely, or exterminate it. To unleash something is to let it loose or set it in motion. The first part of the sentence indicates that epidemiologists were dealing with an infectious disease, but their attempts at doing something were futile, or ineffective. Epidemiologists are scientists who study diseases and try to control them in populations, so the term eradicate fits the first blank; epidemiologists likely would want to do away with an infectious disease completely. However, if the epidemiologists were attempting to exterminate the disease, it does not make sense to say that they managed to unleash it, or succeeded in letting it loose, in an area. Even if their attempts to eradicate the disease were ineffective, scientists trying to do away with a disease would not try to unleash it.. Choice (D) is incorrect. To defend something is to protect it from danger. In this context, to cure something is to deal with it in a way that eliminates it. The first part of the sentence indicates that epidemiologists were dealing with an infectious disease, but their attempts at doing something were futile, or ineffective. Epidemiologists are scientists who study diseases and try to control them in populations, so the term cure makes sense in the second blank; epidemiologists certainly would want to cure, or eliminate, an infectious disease in a community, and the disease might emerge in other locations if their efforts were futile. However, the term defend does not make sense in the first blank, because epidemiologists trying to cure a disease would not attempt to protect that disease.. Choice (E) is incorrect. To replicate something is to duplicate or repeat it. To monitor something is to watch or keep track of it. The first part of the sentence indicates that epidemiologists were dealing with an infectious disease, but their attempts at doing something were futile, or ineffective. Epidemiologists are scientists who study diseases and try to control them in populations, and it is unlikely that they would attempt to duplicate an infectious disease in a community; epidemiologists probably would try to eradicate a disease, not replicate it. Further, although it is logical to suggest that epidemiologists would monitor an infectious disease in a certain area, they most likely would try to do more than simply keep track of it. Therefore, the terms replicate and monitor do not best complete the sentence.4 Explanation for Correct Answer E. Choice (E) is correct. To be nonchalant is to seem to be indifferent, uninterested, or unconcerned. Something that is perfunctory lacks enthusiasm or is routine. The structure of the sentence indicates that the statement after the semi-colon further explains the air, or attitude, of the accused man. The term just suggests that the mans denial of guilt was not as strong as a vehement protestation, or powerful declaration, so it makes sense to suggest that his denial was simply perfunctory. And his behavior might be described as affecting a nonchalant air, or seeming to be unconcerned; the accused mans unenthusiastic denial likely would have suggested that he was unconcerned or uninterested..

Choice (A) is incorrect. To be subdued is to lack intensity or strength. Something that is scathing is bitterly severe or caustic. The structure of the sentence indicates that the statement after the semi-colon further explains the air, or attitude, of the accused man. The sentence states that the man made no vehement protestation of innocencethat is, he did not powerfully declare that he was innocent. A bitterly severe denial of guilt would likely be considered a vehement protestation, so the term scathing does not make sense in the second blank. Further, making a scathing denial is not evidence of affecting a subdued air, or seeming to lack intensity; a scathing statement would reveal the mans intensity.. Choice (B) is incorrect. To be bombastic is to be pompous or arrogant. To be remorseful is to feel regretful or guilty for a past wrong. The structure of the sentence indicates that the statement after the semi-colon further explains the air, or attitude, of the accused man. The term just suggests that the mans denial of guilt was not as strong as a vehement protestation, or powerful declaration. A remorseful statement might not be vehement, but it is somewhat illogical to suggest that the accused man would have guiltily denied his guilt. Further, making a remorseful denial is not evidence of affecting a bombastic air, or seeming to be pompous; there is no direct connection between feeling regretful and appearing pompous.. Choice (C) is incorrect. To be pugnacious is to be argumentative or belligerent. Something that is terse is short and abrupt. The structure of the sentence indicates that the statement after the semi-colon further explains the air, or attitude, of the accused man. The term just suggests that the mans denial of guilt was not as strong as a vehement protestation, or powerful declaration. A short denial might not be considered a powerful declaration, so the term terse might make sense in the second blank. However, the term pugnacious does not make sense in the first blank. Terse denials and a lack of vehement protestation are not evidence of affecting a pugnacious air, or seeming to be argumentative. Indeed, a pugnacious person would likely argue at length and make powerful statements, and the sentence explicitly states that the man did not behave this way.. Choice (D) is incorrect. To be quizzical is to be puzzled, curious, or disbelieving. To be loquacious is to be wordy or talkative. The structure of the sentence indicates that the statement after the semi-colon further explains the air, or attitude, of the accused man. The term just suggests that the mans denial of guilt was not as strong as a vehement protestation, or powerful declaration. A loquacious denial might not be as strong as a vehement protestation, but one could powerfully declare his or her innocence in a wordy, talkative way, so the term loquacious is not the best choice for the second blank. Further, a wordy denial is not evidence of affecting a quizzical air, or seeming to be puzzled or curious; there is not necessarily a connection between being talkative and appearing puzzled.5 Explanation for Correct Answer A. Choice (A) is correct. Something that is "convoluted" is complicated and intricate. The sentence indicates that the "intricacy," or complexity, of the novel "has a daunting effect on readers"in other words, the story in the novel is so complicated that readers are intimidated by it. It makes sense describe such a tale as convoluted.. Choice (B) is incorrect. To be "culpable" is to be guilty or deserving of blame. It is somewhat illogical to suggest that the story in a novel is "a culpable tale"; the term "culpable" is not normally used to describe novels, and it is unclear how a

story could be guilty of something. The sentence indicates that the "intricacy," or complexity, of the novel "has a daunting effect on readers"in other words, the story in the novel is so complicated that readers are intimidated by it. It makes sense to describe such a tale as convoluted, or complicated and intricate.. Choice (C) is incorrect. Something that is "succinct" is brief and precise. The sentence indicates that the "intricacy," or complexity, of the story in the novel "has a daunting effect on readers"in other words, the story is so complicated that readers are intimidated by it. It is unlikely that a novel with an intricate story would be described as a succinct tale, and readers would be unlikely to be daunted by a brief and precise work.. Choice (D) is incorrect. Something that is "enthralling" is captivating or spellbinding. A novel described as enthralling would be one that is hard to put down. The sentence indicates that the intricacy, or complexity, of the story in the novel has a daunting effect on readersin other words, the story is so complicated that readers are intimidated by it. It is somewhat unlikely that a novel that is daunting would be described as enthralling; it seems likely that very few readers would be captivated by a novel they find intimidating.. Choice (E) is incorrect. Something that is "felicitous" is apt, well-suited, or delightful. The sentence indicates that the intricacy, or complexity, of the story in the novel has a daunting effect on readersin other words, the story is so complicated that readers are intimidated by it. It is unlikely that a novel that is daunting would be considered felicitous; it seems likely that few readers would describe a novel they find intimidating as delightful.6 Explanation for Correct Answer B. Choice (B) is correct. The opening sentence of the passage introduces the scene: "This all started on a Saturday morning in May, one of those warm spring days that smell like clean linen." The reference to the smell of clean linen, in conjunction with the information about it being a warm spring day, evokes a particular sensationa feeling of freshness, cleanliness, and warmth. As a whole, the sentence suggests fresh starts and new possibilities, and the reference to clean linen is a key detail the author uses to create that feeling.. Choice (A) is incorrect. The opening sentence of the passage introduces the scene: "This all started on a Saturday morning in May, one of those warm spring days that smell like clean linen." There is no indication anywhere in the passage that Delia is doing anything with clean linen, and the smell of clean linen has nothing to do with going to the supermarket; the reference does not explain any course of action that Delia takes in the passage. Instead, the reference to the smell of clean linen, in conjunction with the information about it being a warm spring day, evokes a particular sensationa feeling of freshness, cleanliness, and warmth.. Choice (C) is incorrect. The opening sentence of the passage introduces the scene: "This all started on a Saturday morning in May, one of those warm spring days that smell like clean linen." There is no indication that the reference to clean linen describes an unexpected development; in fact, by saying that this was just one of those . . . spring days, the author implies that it is not unusual for a spring day to smell like clean linen. The reference to the smell, in conjunction with the information about it being a warm spring day, simply evokes a particular sensationa feeling of freshness, cleanliness, and warmth.. Choice (D) is incorrect. The opening sentence of the passage introduces the scene: "This all started on a Saturday morning in May, one of those warm spring

days that smell like clean linen." It is possible that Delia is reminded of past experiences when she smells clean linen, but nothing in the passage suggests that the smell causes her to feel nostalgic, or wistfully longing, for a past experience. Instead, the reference to the smell of clean linen, in conjunction with the information about it being a warm spring day, evokes a particular sensation a feeling of freshness, cleanliness, and warmth associated with fresh startsthat helps the reader imagine the scene.. Choice (E) is incorrect. The opening sentence of the passage introduces the scene: "This all started on a Saturday morning in May, one of those warm spring days that smell like clean linen." There is no indication anywhere in the passage that Delia is supposed to do anything with clean linen, so there is no reason to believe the reference to the smell of clean linen points out any pressing obligation facing Delia. Instead, the reference to the smell, in conjunction with the information about it being a warm spring day, evokes a particular sensationa feeling of freshness, cleanliness, and warmththat helps the reader imagine the scene.7 Explanation for Correct Answer D. Choice (D) is correct. A vivid description is a lively description that gives the reader a distinct image or picture of what is being described. In lines 6-8, Delia wonders why celery is not called "corduroy plant"presumably because a piece of celery is vertically ribbed, much like corduroy fabric. To Delia, it "would be much more colorful" to call celery corduroy plant. In this context, the word "colorful" gives a sense of something vividly descriptive; Delia feels that calling the vegetable "corduroy plant" would be much more vividly descriptive than calling it "celery.". Choice (A) is incorrect. Something that is garish is often tastelessly flashy, showy, or brightly colored. In lines 6-8, Delia wonders why celery is not called "corduroy plant"presumably because a piece of celery is vertically ribbed, much like corduroy fabric. To Delia, it "would be much more colorful" to call celery corduroy plant. In this context, the word "colorful" gives a sense of something vividly descriptive; Delia feels that calling the vegetable "corduroy plant" would be much more vividly descriptive than calling it "celery." There is no indication that Delia sees the phrase "corduroy plant" as garish or tasteless; she seems to like the phrase.. Choice (B) is incorrect. Something that is robust is strong and sturdy. In lines 6-8, Delia wonders why celery is not called "corduroy plant"presumably because a piece of celery is vertically ribbed, much like corduroy fabric. To Delia, it "would be much more colorful" to call celery corduroy plant. In this context, the word "colorful" gives a sense of something vividly descriptive; Delia feels that calling the vegetable "corduroy plant" would be much more vividly descriptive than calling it "celery." There is no indication that Delia sees the phrase "corduroy plant" as particularly robust, and it does not make sense to say that she feels it would be sturdy to call celery by another name.. Choice (C) is incorrect. Something that is subtly hued is delicately colored. In lines 6-8, Delia wonders why celery is not called "corduroy plant"presumably because a piece of celery is vertically ribbed, much like corduroy fabric. To Delia, it "would be much more colorful" to call celery corduroy plant. In this context, the word "colorful" gives a sense of something vividly descriptive; Delia feels that calling the vegetable "corduroy plant" would be much more vividly descriptive than calling it "celery." It does not make sense to say that Delia feels it would be subtly hued to call celery by another name; corduroy plant is a phrase and would not have colors or hues..

Choice (E) is incorrect. Something that is eye-catching is visually stimulating or attractive to the eye. In lines 6-8, Delia wonders why celery is not called "corduroy plant"presumably because a piece of celery is vertically ribbed, much like corduroy fabric. To Delia, it "would be much more colorful" to call celery corduroy plant. In this context, the word "colorful" gives a sense of something vividly descriptive; Delia feels that calling the vegetable "corduroy plant" would be much more vividly descriptive than calling it "celery." Delia might find a piece of celery eye-catching, but a phrase or name cannot by itself be eye-catching, unless, perhaps, it is written and on display. There is no reason to believe Delia would think the name "corduroy plant" is eye-catching.8 Explanation for Correct Answer C. Choice (C) is correct. The passage indicates that Smith's novels, which criticized powerful English institutions (English life and laws, Englands social organization, the English legal system), led to the author being called "subversive" and a "menace." A subversive is someone who is attempting to overturn or overthrow something, usually a government or a social order. A menace is someone who poses a threat or a danger. The fact that critics used these two terms to describe Smith suggests that they saw her novels as harmful to the established social order.. Choice (A) is incorrect. The passage does not mention the style of Smiths descriptions, and there is no indication that Smith's critics considered her novels to be frivolous or pointless. Indeed, the passage suggests that critics seemed to take the author's novels very seriously; they called Smith "a subversive," or someone who is attempting to overturn or overthrow something (usually a government or a social order), and a "menace," or someone who poses a threat or a danger. The fact that critics used these two terms to describe Smith suggests that they saw her novels as harmful to the established social order.. Choice (B) is incorrect. There is no indication in the passage that Smith's critics thought her novels contained many historically inaccuracies. Rather than focusing on factual mistakes her novels may have contained, critics seem to have been concerned with the criticism in Smiths novels, calling her "a subversive" and a "menace." A subversive is someone who is attempting to overturn or overthrow something, usually a government or a social order. A menace is someone who poses a threat or a danger. The fact that critics used these two terms to describe Smith suggests that they saw her novels as harmful to the established social order.. Choice (D) is incorrect. The passage does not mention questions of morality, and there is no indication that Smith's critics thought her novels were indifferent to, or unconcerned with, such questions. Rather, critics seemed to have been concerned with the criticism in Smiths novels, calling her "a subversive" and a "menace." A subversive is someone who is attempting to overturn or overthrow something, usually a government or a social order. A menace is someone who poses a threat or a danger. The fact that critics used these two terms to describe Smith suggests that they saw her novels as harmful to the established social order.. Choice (E) is incorrect. There is no indication in the passage that Smith's critics thought her novels were disrespectful of any literary traditions; rather, they seem to have felt that Smith was disrespectful of powerful institutions (English life and laws, Englands social organization, the English legal system), calling her "a subversive" and a "menace." A subversive is someone who is attempting to overturn or overthrow something, usually a government or a social order. A

menace is someone who poses a threat or a danger. The fact that critics used these two terms to describe Smith suggests that they saw her novels as harmful to the established social order.9 Explanation for Correct Answer E. Choice (E) is correct. The author of the passage describes Charlotte Smiths literary attacks on English institutions (such as the legal system) as somewhat timorous when compared to later exposs by . . . novelists such as Charles Dickens. An expos serves to expose, or display, something that is illegal or unethical. If Dickens exposs make Smiths attacks look somewhat timorous, or timid, his attacks (made in novels, which are literary works) must be bolder than those made by Smith.. Choice (A) is incorrect. The author of the passage does state that Charles Dickens wrote expossthat is, he produced writings intended to expose, or display, something that is illegal or unethical. However, the author suggests that Dickens exposs do attack social institutions rather than simply exposing them. He or she notes that Charlotte Smiths literary attacks on English institutions seem somewhat timorous, or timid, when compared to exposs by Dickens, implying that Dickens works feature attacks bolder than those made by Smith. Further, there is no mention of whether or not Dickens "found success.". Choice (B) is incorrect. Rather than indicating that Charles Dickens rebelled against a literary tradition that Charlotte Smith had originated, the author of the passage suggests that Dickens works follow in the footsteps of Smiths works. The author states that Smiths novels contain some of the earliest attacks on the English legal system and that it was Smith who introduced such a target for later novelists. The passage suggests that Dickens was one of those later novelists who attacked English institutions as Smith had. Indeed, Dickens exposs seem to contain attacks even bolder than those made by Smith.. Choice (C) is incorrect. The author of the passage compares the writing of Charlotte Smith with that of Charles Dickens, suggesting that both authors made literary attacks against English institutions such as the legal system; therefore, one could say that Dickens developed themes similar to Smiths. Additionally, the focus of the passage is on criticism Smith received, and there is no mention of any criticism Dickens may have received; one might infer that Dickens was criticized less than Smith. However, the author does not specifically say that Dickens received far less criticism for his exposs than Smith did for hers, and he or she does not mention Dickens primarily as someone who received less criticism than Smith. Dickens is mentioned as someone whose attacks were so bold that they made Smith's attacks seem timorous, or timid.. Choice (D) is correct. The author of the passage states that Smith made literary attacks on the English legal system. However, the author does not address Dickens reaction to Smiths criticisms. One might infer that Dickens and Smith both attacked the legal system, but nothing in the passage suggests that Dickens agreed with the content of Smith's attacks; indeed, Dickens may not have even read Smith's novels. Dickens is not mentioned primarily as someone who agreed with Smith's attacks on the legal system, but rather as someone whose attacks were so bold that they made Smith's attacks seem timorous, or timid. 10 Explanation for Correct Answer B. Choice (B) is correct. The narrator notes that although his friends knowledge was remarkable, the friend appeared to know next to nothing of contemporary literature, philosophy and politics. The narrator then provides an example of his friends lack of knowledge of these subjects, stating that his friend did not know

who Thomas Carlyle was (Carlyle was a prominent essayist and historian of the time). The narrators statements in lines 1-5 suggest that he believes educated people should know who Carlyle was; he implies that someone who is unaware of Carlyle is displaying "ignorance.". Choice (A) is incorrect. The narrator refers to Thomas Carlyle when describing his friends remarkable ignorance, stating that his friend was extraordinarily knowledgeable but seemed to know next to nothing of contemporary literature, philosophy and politics and did not know who Carlyle was. Although the narrator suggests that Carlyle was an important person, he does not indicate that Carlyle was the foremost writer of his agein fact, he does not even mention that Carlyle was a writer. (We learn in the footnote that Carlyle was an historian and essayist.) The narrator simply implies that people who know about current literature, philosophy, and politics should know who Carlyle was.. Choice (C) is incorrect. The narrator refers to Thomas Carlyle when describing his friends remarkable ignorance, stating that his friend was extraordinarily knowledgeable but seemed to know next to nothing of contemporary literature, philosophy and politics and did not know who Carlyle was. There is no mention in lines 1-5 of Carlyles views, and the narrator does not indicate whether Carlyles views were popular or unpopular with certain groups of people. The narrator simply implies that people who know about current literature, philosophy, and politics should know who Carlyle was.. Choice (D) is incorrect. The narrator refers to Thomas Carlyle when describing his friends remarkable ignorance, stating that his friend was extraordinarily knowledgeable but seemed to know next to nothing of contemporary literature, philosophy and politics and did not know who Carlyle was. There is no mention in lines 1-5 of Carlyles ideas, and the narrator does not discuss the relevance of those ideas to most peoples daily lives. The narrator simply implies that people who know about current literature, philosophy, and politics should know who Carlyle was.. Choice (E) is incorrect. The narrator refers to Thomas Carlyle when describing his friends remarkable ignorance, stating that his friend was extraordinarily knowledgeable but seemed to know next to nothing of contemporary literature, philosophy and politics and did not know who Carlyle was. The narrator seems to think Carlyle was important, and he may have appreciated the complexity of Carlyles thinking. However, there is no mention in lines 1-5 of Carlyles thinking or of readers appreciating or not appreciating its complexity. The narrator simply implies that people who know about current literature, philosophy, and politics should know who Carlyle was.11 Explanation for Correct Answer C. Choice (C) is correct. In the first paragraph the narrator describes his surprise at the occasional ignorance of his friend, a remarkably knowledgeable person. He notes that he was most surprised to learn that his friend was ignorant of the Copernican Theory and of the composition of the solar system, expressing disbelief that any civilized human being in this nineteenth century should not be aware that the earth traveled round the sun. The narrator seems to consider his friend to be a civilized human being, and as such, seems to have expected that his friend would be aware of the common knowledge that the earth travels around the sun. The narrators remarks about any civilized human being reveal his shock at the unexpected revelation of his friends ignorance, a revelation so extraordinary that he could hardly realize it..

Choice (A) is incorrect. In the first paragraph the narrator describes his surprise at the occasional ignorance of his friend, a remarkably knowledgeable person. He notes that he was most surprised to learn that his friend was ignorant of the Copernican Theory and of the composition of the solar system, expressing disbelief that any civilized human being in this nineteenth century should not be aware that the earth traveled round the sun. The narrator does seem to have been somewhat annoyed by his friend's ignorance, but this state of affairs could not accurately be described as "common." It is more appropriate to say that the narrator was shocked by the unexpected revelation that his friendpresumably a civilized personwas unaware of the composition of the solar system.. Choice (B) is incorrect. In the first paragraph the narrator describes his surprise at the occasional ignorance of his friend, a remarkably knowledgeable person. He notes that he was most surprised to learn that his friend was ignorant of the Copernican Theory and of the composition of the solar system, expressing disbelief that any civilized human being in this nineteenth century should not be aware that the earth traveled round the sun. The primary emotion the narrator expresses is shock, not regret; there is no suggestion that the narrator felt any sorrow. Rather, the narrator was deeply surprised by the unexpected revelation that his friendpresumably a civilized personwas unaware of something the narrator feels almost anyone (any civilized human being) would know.. Choice (D) is incorrect. In the first paragraph the narrator describes his surprise at the occasional ignorance of his friend, a remarkably knowledgeable person. He notes that he was most surprised to learn that his friend was ignorant of the Copernican Theory and of the composition of the solar system, expressing disbelief that any civilized human being in this nineteenth century should not be aware that the earth traveled round the sun. The narrator might have been disappointed that his friend was not aware of the Copernican Theory, but he does not refer to any of the friends actions. Rather, he indicates that he was shocked by the unexpected revelation that his friendpresumably a civilized personwas unaware of something the narrator feels almost anyone (any civilized human being) would know.. Choice (E) is incorrect. In the first paragraph the narrator describes his surprise at the occasional ignorance of his friend, a remarkably knowledgeable person. He notes that he was most surprised to learn that his friend was ignorant of the Copernican Theory and of the composition of the solar system, expressing disbelief that any civilized human being in this nineteenth century should not be aware that the earth traveled round the sun. The narrator does express surprise, but he does not indicate that his friend was gullible, or easily duped. Rather than suggesting that his friend was duped into believing something about the composition of the solar system, he indicates that his friend was unaware of the Copernican Theory altogether. The narrator was shocked by the unexpected revelation that his friend was unaware of something the narrator feels almost anyone (any civilized human being) would know.12 Explanation for Correct Answer D. Choice (D) is correct. In this context, to conceive something is to imagine or understand it. In the last sentence of the first paragraph the narrator expresses his astonishment at the fact that any civilized human being in this nineteenth century should not be aware that the earth traveled round the sun, a fact so extraordinary that he could hardly realize it. As it is used here, the term realize most nearly means conceive; the narrator can hardly imagine that a civilized person might not know the composition of the solar system..

Choice (A) is incorrect. To clarify something is to make it clear, free from confusion. In the last sentence of the first paragraph the narrator expresses his astonishment at the fact that any civilized human being in this nineteenth century should not be aware that the earth traveled round the sun, a fact so extraordinary that he could hardly realize it. The author is very surprised, but he does not seem to be confused. Rather than suggesting that he could not clarify the idea that a civilized person might not know the composition of the solar system, he indicates that he could hardly conceive, or imagine, the idea in the first place.. Choice (B) is incorrect. To actualize something is to make it exist. In the last sentence of the first paragraph the narrator expresses his astonishment at the fact that any civilized human being in this nineteenth century should not be aware that the earth traveled round the sun, a fact so extraordinary that he could hardly realize it. It is somewhat illogical to suggest that the narrator could not actualize a fact; a fact is something that has actual existence. Further, the narrators friend seems to prove the fact that a civilized human being might not be aware of the composition of the solar system. In this context, the term realize does not mean actualize.. Choice (C) is incorrect. To accomplish something is to bring it to completion. In the last sentence of the first paragraph the narrator expresses his astonishment at the fact that any civilized human being in this nineteenth century should not be aware that the earth traveled round the sun, a fact so extraordinary that he could hardly realize it. It does not make sense to say that the narrator could not accomplishcould not completethe fact that a civilized person might not be aware of something. In line 11, the term realize means conceive, not accomplish; the narrator can hardly imagine that a civilized person might not be aware of the composition of the solar system.. Choice (E) is incorrect. To gain something is to acquire or get possession of it. In the last sentence of the first paragraph the narrator expresses his astonishment at the fact that any civilized human being in this nineteenth century should not be aware that the earth traveled round the sun, a fact so extraordinary that he could hardly realize it. It is somewhat illogical to suggest that the narrator could not gaincould not acquirethe fact that a civilized person might not be aware of something. In line 11, the term realize means conceive, not gain; the narrator can hardly imagine that a civilized person might not be aware of the composition of the solar system.13 Explanation for Correct Answer C. Choice (C) is correct. The narrator reacts with an expression of surprise" when he learns that his friend appears not to know "that the earth traveled round the sun." Next the narrator is told that his friend will do [his] best to forget" this important information about the solar system. When the narrator repeats part of his friends unexpected statement about forgetting new informationTo forget it!the phrase emphasizes his surprised disbelief. The narrator is shocked that his friend would try to forget such crucial facts as the Copernican Theory.. Choice (A) is incorrect. In the first paragraph the narrator states that despite his friends remarkable . . . knowledge, the friend was ignorant of certain things that the narrator feels any civilized human being should know. The narrator learns that his friend does not know about the Copernican Theory or the composition of the solar system and that he actually intends to do [his] best to forget these newly-acquired facts. The narrator might not approve of his friends dismissal of important facts, but the narrators reaction is more shocked than displeased: the narrator has an expression of surprise on his face, and his

friend notes that the narrator seems astonished. When the narrator repeats part of his friends unexpected statement about forgetting new informationTo forget it!the phrase primarily emphasizes his surprise and disbelief.. Choice (B) is incorrect. In the first paragraph the narrator states that despite his friends remarkable . . . knowledge, the friend was ignorant of certain things that the narrator feels any civilized human being should know. The narrator learns that his friend does not know about the Copernican Theory or the composition of the solar system and that he actually intends to do [his] best to forget these newly-acquired facts. There is no indication that the narrator is embarrassed by his friends dismissal of important facts. Rather than reacting with embarrassment or shame of some kind, the narrator reacts with shock: the narrator has an expression of surprise on his face, and his friend notes that the narrator seems astonished. When the narrator repeats part of his friends unexpected statement about forgetting new informationTo forget it!the phrase primarily emphasizes his surprise and disbelief.. Choice (D) is incorrect. In the first paragraph the narrator states that despite his friends remarkable . . . knowledge, the friend was ignorant of certain things that the narrator feels any civilized human being should know. The narrator learns that his friend does not know about the Copernican Theory or the composition of the solar system and that he actually intends to do [his] best to forget these newly-acquired facts. There is no indication that the narrator feels any sense of urgency, or a pressing need to do anything in response to his friends dismissal of important facts. Instead, the narrator simply reacts with shock: the narrator has an expression of surprise on his face, and his friend notes that the narrator seems astonished. When the narrator repeats part of his friends unexpected statementTo forget it!the phrase primarily emphasizes his surprise and disbelief.. Choice (E) is incorrect. In the first paragraph the narrator states that despite his friends remarkable . . . knowledge, the friend was ignorant of certain things that the narrator feels any civilized human being should know. The narrator learns that his friend does not know about the Copernican Theory or the composition of the solar system and that he actually intends to do [his] best to forget these newly-acquired facts. The author might not like his friends dismissal of important facts, but his reaction is more shocked than regretful: the narrator has an expression of surprise on his face, and his friend notes that the narrator seems astonished. When the narrator repeats part of his friends unexpected statement about forgetting new informationTo forget it!the phrase primarily emphasizes his surprise and disbelief, not any feelings of regret or sorrow.14 Explanation for Correct Answer E. Choice (E) is correct. The narrators friend explains his view that a mans brain originally is like a little empty attic that has to be stocked with furniture. A fool, he states, takes in all the lumber of every sort he comes across, while a skillful workman is very careful . . . as to what he takes into his brain-attic, taking nothing but the tools which may help him in doing his work. The friend indicates that unlike a person who chooses to take in only useful information, a foolish person acquires information indiscriminately, or haphazardly and not carefully; a fool takes in all the lumber . . . he comes acrossall the information he encounters, no matter what it isinstead of choosing only the tools which may help him.. Choice (A) is incorrect. The narrators friend explains his view that a mans brain originally is like a little empty attic that has to be stocked with furniture. A

fool, he states, takes in all the lumber of every sort he comes across, while a skillful workman is very careful . . . as to what he takes into his brain-attic, taking nothing but the tools which may help him in doing his work. The friend suggests that a fool does not possess only specialized knowledge; on the contrary, a fool takes in all the information he encounters, no matter what it is (all the lumber of every sort). The friend implies that it is the skillful workman who might possess only specialized knowledgeknowledge that may help him.. Choice (B) is incorrect. The narrators friend explains his view that a mans brain originally is like a little empty attic that has to be stocked with furniture. A fool, he states, takes in all the lumber of every sort he comes across, while a skillful workman is very careful . . . as to what he takes into his brain-attic, taking nothing but the tools which may help him in doing his work. The friends explanation does not deal with practical experience, or real experience, but rather with the acquisition of information. The friend suggests that a fool takes in all the information he encounters, no matter what it is (all the lumber of every sort), instead of choosing to take in only useful information (tools which may help him).. Choice (C) is incorrect. The narrators friend explains his view that a mans brain originally is like a little empty attic that has to be stocked with furniture. A fool, he states, takes in all the lumber of every sort he comes across, while a skillful workman is very careful . . . as to what he takes into his brain-attic, taking nothing but the tools which may help him in doing his work. The friend does not suggest that fools are easily deceived; he does not imply that fools take in information of any kind because they are tricked into doing so. Rather, he suggests that a fool chooses to take in all the information he encounters, no matter what it is (all the lumber of every sort), instead of choosing to take in only useful information (tools which may help him).. Choice (D) is incorrect. The narrators friend explains his view that a mans brain originally is like a little empty attic that has to be stocked with furniture. A fool, he states, takes in all the lumber of every sort he comes across, while a skillful workman is very careful . . . as to what he takes into his brain-attic, taking nothing but the tools which may help him in doing his work. The friend implies that a fool might in fact respect learninghe certainly indicates that a fool takes in lots of information, which suggests an appreciation for learning. Rather than indicating that a fool has little respect for learning, the friend suggests that it is foolish to acquire information indiscriminately, or not carefully; a fool takes in all the information he encounters, no matter what it is (all the lumber of every sort), instead of choosing to take in only useful information (tools which may help him).15 Explanation for Correct Answer B. Choice (B) is correct. Pragmatic means practical and relating to matters of fact. The narrators friend explains that he sees the brain as an empty space (a little empty attic) that needs to be stocked with useful information (tools which may help him). He feels that the brain can hold a limited amount of informationthe little room does not have elastic walls and cannot distend to any extent and suggests that at some point, every addition of knowledge causes the forgetting of a previously-held piece of information. The friend states that it is therefore important not to have useless facts elbowing out the useful ones. His attitude toward the acquisition of knowledge is best characterized as pragmatic; he feels that people should only acquire knowledge that is useful and practical.. Choice (A) is incorrect. To be open-minded is to be receptive or open to arguments or ideas. The narrators friend does not seem to be very open-minded

when it comes to the acquisition of knowledge; he seems to have a very firm idea of how one should take in information. He suggests that because the brain can hold a limited amount of informationit does not have elastic wallsthere comes a time when every addition of knowledge causes the forgetting of a previously-held piece of information. The friend states plainly that it is important not to have useless facts elbowing out the useful ones. There is no indication that the friend would be receptive to other peoples ideas about the acquisition of knowledge.. Choice (C) is incorrect. To be delighted is to be highly pleased. The narators friend seems to appreciate taking in useful information, but his attitude toward the acquisition of knowledge is not best characterized as delighted. Rather than expressing great pleasure, the friend is practical and matter-of-fact; he seems to have a very firm idea of how one should take in information. He suggests that because the brain can hold a limited amount of informationit does not have elastic wallsthere comes a time when every addition of knowledge causes the forgetting of a previously-held piece of information. The friend states plainly that it is important not to have useless facts elbowing out the useful ones. He does not express delight.. Choice (D) is incorrect. To be disillusioned is to be disappointed or dissatisfied. Although the narrators friend seems to disapprove of the way some people take in information, his attitude toward the acquisition of knowledge in general is not disillusioned. The friend looks at the acquisition of knowledge in a pragmatic or practical way: he suggests that because the brain can hold a limited amount of informationit does not have elastic wallsthere comes a time when every addition of knowledge causes the forgetting of a previously-held piece of information, and that it is therefore important not to have useless facts elbowing out the useful ones. The friend certainly is not dissatisfied with the acquisition of useful facts.. Choice (E) is incorrect. To be apathetic is to be indifferent or to have little or no interest in or concern about something. Rather than seeming to be indifferent when it comes to the acquisition of knowledge, the friend seems to have a strong opinion on the subject. He explains his very firm idea of how one should take in information: because the brain can hold a limited amount of informationit does not have elastic wallsthere comes a time when every addition of knowledge causes the forgetting of a previously-held piece of information. Therefore, it is important not to have useless facts elbowing out the useful ones. The friend seems to be very interested in the acquisition of knowledge; his attitude is not one of apathy.16 Explanation for Correct Answer E. Choice (E) is correct. The primary purpose of the passage is to offer the authors personal view of a cultural developmentthe shift toward Asian Americans being portrayed on television more frequently, and more realistically, than they once were. The author begins the passage by discussing how, in her childhood, it was rare to see a real Asian American on television; people of Asian descent were virtually nonexistent on television, and her family would drop everything when they saw regular Asians like [them], on the news, game shows, variety programs, or beauty pageants. She then describes her own experience with being one of those real Asians on TV when she went on a game show. She notes that the two actresses hired to play [her] were older and were Miss World material, musing that The available selection of Asian American actors must have been as sparse as the roles available for them to play. In the last paragraph, the author states that things have changed: Asian sightings are more common now. She refers to seeing a Chinese man . . . as the star of a

television series and a movie like Mulan . . . using real Asian American actors voices.. Choice (A) is incorrect. The primary purpose of the passage is not to demand an end to regressive industry practices; on the contrary, the purpose is to offer a view of a cultural development that the author feels is progressive, not regressive. The author shares her personal view of the shift toward Asian Americans being portrayed on television more frequently, and more realistically, than they once were. The author suggests that some progress has been made in the television industry as far as Asian American actors are concerned; she does not indicate that practices in that industry are regressing, or moving backwards and becoming worse.. Choice (B) is incorrect. The author discusses television, which is a modern invention, but the primary purpose of the passage is not to discuss the impact of television or any other modern invention. Instead, the purpose is to offer the authors view of a cultural developmentthe shift toward Asian Americans being portrayed on television more frequently, and more realistically, than they once were. Rather than discussing the impact of television, the author discusses television as a reflection of and a stage for a cultural development.. Choice (C) is incorrect. The author does discuss the past (her childhood), but she does not analyze the causes of a historical phenomenon, or a particularly unusual event in history. Instead, her primary purpose is to offer her view of a cultural developmentthe shift toward Asian Americans being portrayed on television more frequently, and more realistically, than they once were. Even if one were to see this shift as a historical phenomenon, the author does not really discuss its causes; rather, she discusses her personal view of it and her small role in it.. Choice (D) is incorrect. The author does discuss an event from her lifetimeher appearance on a TV game showbut she does not indicate that this event was difficult or that it changed her life. Rather, the author mentions her appearance on television in part of her discussion of a cultural development; she offers her personal view of the shift toward Asian Americans being portrayed on television more frequently, and more realistically, than they once were.17 Explanation for Correct Answer A. Choice (A) is correct. The author indicates that when there was an Asian sightingwhen a family member saw a real Asian American on televisionher family would react with excitement. She says that when A hoot went out everyone would drop everything and make a frenzied rush to the tube, eager to see who had entered that mysterious TV land where people of Asian descent were virtually nonexistent. The familys hooting and frenzied rushing indicate that they found these rare Asian sightings exciting.. Choice (B) is incorrect. The author states that when there was an Asian sightingwhen a family member saw a real Asian American on television there would be A hoot, and everyone would drop everything and make a frenzied rush to the tube to see who had entered that mysterious TV land where people of Asian descent were virtually nonexistent. She notes that even her parents participated enthusiastically in the routine. Although she indicates that these Asian sightings were rare, the authors description of her familys behavior does not suggest that they reacted with shock, or stunned surprise. The family members hooting and frenzied rushing reveal excitement, not shock.. Choice (C) is incorrect. The author states that when there was an Asian sightingwhen a family member saw a real Asian American on television

there would be A hoot, and everyone would drop everything and make a frenzied rush to the tube to see who had entered that mysterious TV land where people of Asian descent were virtually nonexistent. The author does not suggest that the family reacted to Asian sightings with respect or deference of any kind. Rather, the family members hooting and frenzied rushing primarily reveal their excitement.. Choice (D) is incorrect. The author states that when there was an Asian sightingwhen a family member saw a real Asian American on television there would be A hoot, and everyone would drop everything and make a frenzied rush to the tube to see who had entered that mysterious TV land where people of Asian descent were virtually nonexistent. She does not suggest that her family reacted to these Asian sightings with anxiety, or apprehensive uneasiness. Instead, she indicates that her family members reacted with excitement, hooting and rushing in a frenzy.. Choice (E) is incorrect. The author states that when there was an Asian sightingwhen a family member saw a real Asian American on television there would be A hoot, and everyone would drop everything and make a frenzied rush to the tube to see who had entered that mysterious TV land where people of Asian descent were virtually nonexistent. She does not suggest that her family reacted to these Asian sightings with disdain, or feelings of contempt; the hoot seems to be a cheerful whooping noise to alert the rest of the family, and the family seems to be enthusiastic as they rush to the television. The family members hooting and frenzied rushing reveal excitement, not disdain. 18 Explanation for Correct Answer B. Choice (B) is incorrect. As used in line 5, Real most nearly means authentic, or actual and not an imitation or stereotype. The author suggests that regular Asian Americans like her and her family members are Real Asians. On the contrary, "Real Asians didn't include Hop Sing," a stereotypical Asian character from a television show, or white actors who donned makeup to play Asiansin other words, the character Hop Sing and the white actors wearing makeup do not count as "real" or "regular Asians. In this context, the phrase Real Asians refers to authentic, contemporary Asians or Asian Americans, not stereotypical TV characters or white actors who are pretending to be Asian.. Choice (A) is incorrect. In the second paragraph the author suggests that regular Asian Americans like her and her family members are Real Asians. On the contrary, "Real Asians didn't include Hop Sing," a stereotypical Asian character from a television show, or white actors who donned makeup to play Asians in other words, the character Hop Sing and the white actors wearing makeup do not count as "real" or "regular Asians. There is no indication that the author is using the word "Real" to distinguish between serious people and people who are not serious. In line 5 the word Real most nearly means authentic, not serious.. Choice (C) is incorrect. In the second paragraph the author suggests that regular Asian Americans like her and her family members are Real Asians. On the contrary, "Real Asians didn't include Hop Sing," a stereotypical Asian character from a television show, or white actors who donned makeup to play Asians in other words, the character Hop Sing and the white actors wearing makeup do not count as "real" or "regular Asians. There is no indication that the author is using the word "Real" to refer to people who are practical, or people who are concerned with reality instead of theories and ideals; rather than using the word to refer to Asian people who are practical, the author uses it to describe authentic, or actual, Asians..

Choice (D) is incorrect. In the second paragraph the author suggests that regular Asian Americans like her and her family members are Real Asians. On the contrary, "Real Asians didn't include Hop Sing," a stereotypical Asian character from a television show, or white actors who donned makeup to play Asians in other words, the character Hop Sing and the white actors wearing makeup do not count as "real" or "regular Asians. The term utter, which indicates something that is carried to the utmost point or to the highest degree, is not usually used in this context; it is somewhat illogical to suggest that some people are the utmost Asians. In line 5 the word Real most nearly means authentic, or actual, not utter.. Choice (E) is incorrect. In the second paragraph the author suggests that regular Asian Americans like her and her family members are Real Asians. On the contrary, "Real Asians didn't include Hop Sing," a stereotypical Asian character from a television show, or white actors who donned makeup to play Asians in other words, the character Hop Sing and the white actors wearing makeup do not count as "real" or "regular Asians. The term fundamental describes something that is principal or of central importance, but the author indicates that she is referring to regular people (regular Asian Americans like us), not people of central importance to something. Rather than using the word Real to refer to Asian people who are a principal part of something, the author uses it to describe authentic, or actual, Asians.19 Explanation for Correct Answer D. Choice (D) is correct. The author suggests that her parents commentary when watching a real Asian on television was thorough, or complete, and systematic, or methodical. Her parents had a "routine" in which they would "assess" all of the important details of the person on television, including vital signs such as looks, ethnicity, demeanor, and intelligence. This activity is best characterized as thorough and systematic.. Choice (A) is incorrect. The author describes the commentary her parents would deliver when watching a real Asian on television, noting that they would "assess" all of the details of the person on television, including vital signs such as looks, ethnicity, demeanor and intelligence. Her parents observations may have been offered innocently, but the details seem to have been presented seriously and not in an amusing fashion; there is no indication that her parents made humorous remarks about the Asian Americans they saw on TV. The parents commentary is presented as simply being thorough and systematic.. Choice (B) is incorrect. The author describes the commentary her parents would deliver when watching a real Asian on television, noting that they would "assess" all of the details of the person on television, including vital signs such as looks, ethnicity, demeanor and intelligence. The details her parents listed are not particularly technicalthey do not really require special knowledge to be understoodand the author does not suggest that she found these details bewildering, or perplexing and confusing. Instead, she suggests that the commentary was thorough and systematicher parents methodically evaluated the complete list of vital signs.. Choice (C) is incorrect. The author describes the commentary her parents would deliver when watching a real Asian on television, noting that they would "assess" all of the details of the person on television, including vital signs such as looks, ethnicity, demeanor and intelligence. She does not suggest that her parents were especially critical, or inclined to criticize severely or unfavorably, or that their commentary was demoralizing, or upsetting and discouraging. There is no indication that the parents criticized the people they saw on TV in a way that

would upset anyone; they seem to have simply presented a thorough and enthusiastic evaluation of basic details.. Choice (E) is incorrect. The author describes the commentary her parents would deliver when watching a real Asian on television, noting that they would "assess" all of the details of the person on television, including vital signs such as looks, ethnicity, demeanor and intelligence. She does not suggest that her parents commentary was contentious, or likely to start an argument or a debate; there is no indication that her parents said anything about the people they saw on TV that would lead to an argument. The author also does not suggest that the commentary was overwrought, or elaborately excessive or overdone; on the contrary, the parents list of details seems to have been thorough but straightforward.20 Explanation for Correct Answer C. Choice (C) is correct. In lines 16-20 the author relates her parents typical reaction to seeing a real Asian on television: they would assess . . . the looks, ethnicity, demeanor, intelligence and other vital signs of the real Asian, delivering these details in a manner as succinct and passionate as that of a sports announcer. She then notes in lines 20-22 that her parents would also compare the author and her siblings to the person on televisiona habit she describes as irksome, or irritating and wearying. The author suggests that her parents critiques, or evaluations, of Asians on television were comprehensive, including all of the details her parents felt were important. In short, the author indicates that she found these extensive critiques to be irritating and annoying.. Choice (A) is incorrect. In lines 16-20 the author relates her parents typical reaction to seeing a real Asian on television: they would assess . . . the looks, ethnicity, demeanor, intelligence and other vital signs of the real Asian, delivering these details in a manner as succinct and passionate as that of a sports announcer. She then notes in lines 20-22 that her parents would also compare the author and her siblings to the person on television. The author does not indicate that her parents opinions were outdated, or old-fashioned; indeed, she does not mention what opinions her parents held but simply lists the many details that they would comment on. Moreover, the author does not seem angry about her parents' comparisonsonly somewhat irritated by them.. Choice (B) is incorrect. In lines 16-20 the author relates her parents typical reaction to seeing a real Asian on television: they would assess . . . the looks, ethnicity, demeanor, intelligence and other vital signs of the real Asian, delivering these details in a manner as succinct and passionate as that of a sports announcer. She then notes in lines 20-22 that her parents would also compare the author and her siblings to the person on television. The author does not indicate that her parents observations were perceptive; indeed, she does not discuss the keenness of her parents observations but simply lists the many details that they would comment on. Further, the author describes her parents habit as irksome, suggesting that she finds their observations irritating, not impressive.. Choice (D) is incorrect. In lines 16-20 the author relates her parents typical reaction to seeing a real Asian on television: they would assess . . . the looks, ethnicity, demeanor, intelligence and other vital signs of the real Asian, delivering these details in a manner as succinct and passionate as that of a sports announcer. She then notes in lines 20-22 that her parents would also compare the author and her siblings to the person on television. The author does suggest that her parents contributionstheir commentswere spirited, or animated, since she describes their manner as passionate. However, she

describes her parents habit as irksome, suggesting that rather than being delighted, the author is irritated.. Choice (E) is incorrect. In lines 16-20 the author relates her parents typical reaction to seeing a real Asian on television: they would assess . . . the looks, ethnicity, demeanor, intelligence and other vital signs of the real Asian, delivering these details in a manner as succinct and passionate as that of a sports announcer. She then notes in lines 20-22 that her parents would also compare the author and her siblings to the person on television. The author does not indicate that her parents put forth any arguments, or persuasive statements, inconsistent or otherwise; she suggests that they simply listed details and made comparisons.21 Explanation for Correct Answer C. Choice (C) is correct. The author describes her experience on a game show, explaining that a panel of celebrities would have to guess the real contestant from imposters after receiving clues about the real person; in this case, the celebrities had to guess which contestant was the real Asian American college student who went to China. She suggests that the two actors hired as imposters would be unlikely to fool the panelists because they did not seem like regular college students, especially when compared to the authorthe actors were older than [the author] and were Miss World material. The author implies that the game show must have been unable to find actors who looked like regular college students, noting that the selection of Asian American actors must have been as sparse as the roles available for them to play.. Choice (A) is incorrect. The author describes her experience on a game show, explaining that a panel of celebrities would have to guess the real contestant from imposters after receiving clues about the real person; in this case, the celebrities had to guess which contestant was the real Asian American college student who went to China. The author refers to the imposters as actors, suggesting that they might be professional actors, but she does not comment on their professional experience. Instead, she simply implies that their looks made them unlikely to fool the panelists into thinking they were regular college students: the actors were older than [the author] and were Miss World material.. Choice (B) is incorrect. The author describes her experience on a game show, explaining that a panel of celebrities would have to guess the real contestant from imposters after receiving clues about the real person; in this case, the celebrities had to guess which contestant was the real Asian American college student who went to China. Nothing in lines 39-43 suggests that the actors caused the author to question her own identity. The author simply implies that the actors looks made them unlikely to fool the panelists (the actors were older and Miss World material) and that the game show must have been unable to find actors who looked like college students (the selection of Asian American actors must have been . . . sparse).. Choice (D) is incorrect. The author describes her experience on a game show, explaining that a panel of celebrities would have to guess the real contestant from imposters after receiving clues about the real person; in this case, the celebrities had to guess which contestant was the real Asian American college student who went to China. In lines 39-43 the author mentions that the imposters were older and were Miss World material and then states that the selection of Asian American actors must have been as sparse as the roles available for them to play. The author does not refer to the small number of roles to suggest that the actors needed money; instead, she suggests that there

was a relatively small number of both Asian American actors and roles for them to play, and that, as a result, the game show was unable to find actors who looked like a regular Asian American college student. The author does not indicate in lines 39-43 that the actors needed money more than she, a struggling student, did.. Choice (E) is incorrect. The author describes her experience on a game show, explaining that a panel of celebrities would have to guess the real contestant from imposters after receiving clues about the real person; in this case, the celebrities had to guess which contestant was the real Asian American college student who went to China. The author suggests that the actors were unlike her they were older and Miss World materialbut she does not suggest that they were unlike other Asian American performers. She only refers to other performers briefly, noting that the selection of Asian American actors must have been . . . sparse, because the game show was unable to find actors who looked like regular college students.22 Explanation for Correct Answer D. Choice (D) is correct. The author describes going on a game show on which a panel of celebrities had to guess the real contestant from imposters after receiving clues about the real person; in this case, the author was the real contestant. She notes that the two actors hired to be imposters were older than she was and were Miss World material. The author then states, When it came time to pick the real Asian American college student who went to China, somehow all the panelists picked me. The word somehow often indicates a sense of wonder or surprise, but here, the author uses the word sarcastically. She suggests that she was not surprised that the panelists identified her as the real college student, because the actors hired as imposters did not look like college students; the author stood out as the actual student when compared to older actors who looked like contestants in a beauty pageant.. Choice (A) is incorrect. The author describes going on a game show on which a panel of celebrities had to guess the real contestant from imposters after receiving clues about the real person; in this case, the author was the real contestant. She states, When it came time to pick the real Asian American college student who went to China, somehow all the panelists picked me. Nothing in the passage suggests that the author did not enjoy being on the game show, and this statement does not reveal that she was relieved that the experience was over. Instead, she simply implies that she stood out as the real college student when compared to older actors who were Miss World material. The authors sarcastic use of the word somehow suggests that she was not surprised that the panelists identified her as the student.. Choice (B) is incorrect. The author describes going on a game show on which a panel of celebrities had to guess the real contestant from imposters after receiving clues about the real person; in this case, the author was the real contestant. She notes that the two actors hired to be imposters were older than she was and were Miss World material. The author then states, When it came time to pick the real Asian American college student who went to China, somehow all the panelists picked me. The author indicates that the actors were different from her in certain ways, but her statement does not suggest that she is resentful of them for any reason. Instead, she simply implies that she stood out as the real college student when compared to older actors who looked like contestants in a beauty pageant. The authors sarcastic use of the word somehow" suggests that she was not surprised that the panelists identified her as the student..

Choice (C) is incorrect. The author describes going on a game show on which a panel of celebrities had to guess the real contestant from imposters after receiving clues about the real person; in this case, the author was the real contestant. She states, When it came time to pick the real Asian American college student who went to China, somehow all the panelists picked me. Earlier in the passage the author mentions that her parents would compare the author and her siblings to Asians they saw on television, and later she states that her parents commented on her voice and makeup when they saw her on TV; however, the statement in lines 43-45 does not indicate that the author was concerned about her familys reaction to her TV appearance. Her sarcastic use of the word somehow suggests that she was not surprised that the panelists identified her, and not one of the older, Miss World material actors, as the real college student.. Choice (E) is incorrect. The author describes going on a game show on which a panel of celebrities had to guess the real contestant from imposters after receiving clues about the real person; in this case, the author was the real contestant. She states, When it came time to pick the real Asian American college student who went to China, somehow all the panelists picked me. Her sarcastic use of the word somehow suggests that she was not surprised that the panelists identified her, and not one of the older, Miss World material actors, as the real college student. The authors statement in lines 43-45 does not address any repercussions of this event; it simply reveals her reaction to the panelists choice. Further, the fourth paragraph as a whole suggests that rather than being untroubled, the author may have been somewhat bothered by one repercussion of the episodethe fact that she, a struggling student who wouldnt get paid for appearing on the show, did not win any prize money. 23 Explanation for Correct Answer C. Choice (C) is correct. In lines 49-51 the author states that in the prevideocassette recorder daysthat is, the days before people had videocassette recorders (VCRs)it was startling to see yourself on screen. This observation implies that it became more common for people to see themselves on screen, on their home television sets, after the invention of the VCR. With VCRs people began to get used to watching themselves on screen, in home movies or other videotaped programs.. Choice (A) is incorrect. In lines 49-51 the author states that in the prevideocassette recorder daysthat is, the days before people had videocassette recorders (VCRs)it was startling to see yourself on screen. This observation does not suggest that an engineering breakthrough has had unfortunate consequences. The author implies that a technological breakthrough (the invention of the VCR) has had a consequence, but she seems to regard this consequence (the now more common experience of seeing oneself on a TV screen) neutrally, not negatively.. Choice (B) is incorrect. In lines 49-51 the author states that in the prevideocassette recorder daysthat is, the days before people had videocassette recorders (VCRs)it was startling to see yourself on screen. This observation does not suggest that the author was startled to see herself on screen because the programs film quality was poor; she seems to have been startled not by the way she looked on TV but by the very experience of seeing herself on TV. The authors statement suggests that it was rare to see oneself on a TV screen before the invention of the VCR, but that VCRs made the experience more common.. Choice (D) is incorrect. In lines 49-51 the author states that in the prevideocassette recorder daysthat is, the days before people had videocassette

recorders (VCRs)it was startling to see yourself on screen. This statement does refer to something that later became a common piece of equipmentthe VCRbut does not suggest that this equipment madetelevision more accessible. Instead, the statement suggests that this equipment gave people the opportunity to see themselves on television; with a VCR a person might see himself or herself on screen in a home movie or in a taped appearance on the local news, for example.. Choice (E) is incorrect. In lines 49-51 the author states that in the prevideocassette recorder daysthat is, the days before people had videocassette recorders (VCRs)it was startling to see yourself on screen. This statement does refer to something that later became an everyday object for many people the VCRbut does not suggest that this object reshaped the publics preferences in any way. Instead of referring to preferences, the statement simply implies that the VCR made the experience of seeing oneself on a TV screen more common. Before VCRs it was rare to have such an experience, but with a VCR a person might see himself or herself on screen in a home movie or in a taped appearance of the local news, for example.24 Explanation for Correct Answer A. Choice (A) is correct. In the last paragraph the author explains that, although still infrequent, Asian sightings are more common nowone now sees real Asians . . . on the screen. In lines 63-64 the author provides the example of cheering to see a Chinese man, chubby and middle-aged, as the star of a television series. The author implies that the Chinese man is an ordinary Asian Americanhe seems to be a regular person who does not fit the constricting stereotypes of Asians on television. The reader is meant to recall the earlier descriptions of white actors wearing makeup to play Asians and the "Miss World material" Asian actresses on the game show. The example in lines 63-64 applauds the increasing visibility of apparently ordinary Asian Americans on television.. Choice (B) is incorrect. In the last paragraph the author explains that, although still infrequent, Asian sightings are more common nowone now sees real Asians . . . on the screen. In lines 63-64 the author provides the example of cheering to see a Chinese man, chubby and middle-aged, as the star of a television series. This example does not serve to reveal the authors dismay over some characters featured in recent television shows; cheering usually indicates happiness or excitement, not dismay or disappointment. The example serves to applaud the increasing visibility of regular Asian Americans on TVsomething the author is happy about.. Choice (C) is incorrect. In the last paragraph the author explains that, although still infrequent, Asian sightings are more common nowone now sees real Asians . . . on the screen. In lines 63-64 the author provides the example of cheering to see a Chinese man, chubby and middle-aged, as the star of a television series. Although the author is expressing approval, she does not refer to the quality of the Chinese mans television performance (or the performances of other Asian Americans). Instead, she approves of the very fact that an ordinary Asian American is the star of a TV seriesshe applauds the increasing visibility of regular Asian Americans on TV.. Choice (D) is incorrect. In the last paragraph the author explains that, although still infrequent, Asian sightings are more common nowone now sees real Asians . . . on the screen. In lines 63-64 the author provides the example of cheering to see a Chinese man, chubby and middle-aged, as the star of a television series. The author implies that the Chinese man is an ordinary Asian

Americanhe seems to be a regular person who does not fit the constricting stereotypes of Asians on television. The author clearly thinks that ethnic diversity in the entertainment industry is important, but the example in lines 6364 does not deal with ethnic diversity in entertainment in general; instead, it deals specifically with the inclusion of regular Asian Americans on television..

Choice (E) is incorrect. In the last paragraph the author explains that, although still infrequent, Asian sightings are more common nowone now sees real Asians . . . on the screen. In lines 63-64 the author provides the example of cheering to see a Chinese man, chubby and middle-aged, as the star of a television series. The author implies that the Chinese man is an ordinary Asian Americanhe seems to be a regular person who does not fit the constricting stereotypes of Asians on television. Rather than suggesting that there is a need for more highquality dramatic series on television, the example serves to applaud the increasing visibility of regular Asian Americans on television. Section #7: View Explanations 1 Explanation for Correct Answer E. Choice (E) is correct. It avoids the pronoun agreement error and unidiomatic preposition of the original by providing the appropriate plural pronoun they and the appropriate preposition by to follow cherished.. Choice (A) involves a vague pronoun error and an unidiomatic preposition. The singular pronoun it does not agree with the plural subject (Teddy bears). In addition, the preposition to does not idiomatically follow cherished.. Choice (B) involves illogical phrasing and an unidiomatic preposition. It does not make sense to say that teddy bears are still cherished by those born in the future. In addition, the preposition to does not idiomatically follow cherished.. Choice (C) involves a pronoun agreement error and illogical phrasing. The singular pronoun it does not agree with the plural subject (Teddy bears). In addition, it does not make sense to say that teddy bears are still cherished by those born in the future.. Choice (D) results in a sentence fragment. There is no main verb to carry out the action of the second clause, only the participle being.2 Explanation for Correct Answer C. Choice (C) is correct. It avoids the comma-splice error of the original by replacing the first independent clause The film was full of suspense with the phrase The suspenseful film.. Choice (A) involves a comma splice. Two independent clauses (The film was full of suspense and this made Riya keep her hand gripped tightly to her seat) are improperly joined by only a comma.. Choice (B) results in a sentence fragment. There is no main verb to carry out the action of the sentence, only the participle keeping..

Choice (D) results in redundancy and includes an error in pronoun reference. The word Because establishes the cause-effect relationship between the suspenseful film and Riyas reaction to it. So the phrase this is what is unnecessary. In addition, the personal pronoun her, which follows the underlined part of the sentence, should not be used to refer to a possessive (Riyas hands).. Choice (E) results in a modification error. It does not make sense to say that Riya was Being full of suspense.3 Explanation for Correct Answer D. Choice (D) is correct. It avoids the wordiness of the original by reducing the phrase and he was carrying a small easel to the concise modifying phrase carrying a small easel.. Choice (A) involves wordiness. The phrase and he was carrying a small easel is wordy and can be reduced to the more succinct carrying a small easel.. Choice (B) involves a shift in verb tense. The present-tense verb carries is not consistent with the past-tense verb walked.. Choice (C) involves a modification error. The word where can only be used to refer to a place. Here, however, it is used improperly to refer to a time.. Choice (E) results in illogical phrasing. The sentence illogically implies that James McNeill Whistler walked to the beach in order to carry a small easel and his paints.4 Explanation for Correct Answer B. Choice (B) is correct. It avoids the subject-verb agreement error and wordiness of the original by providing the plural verb are to agree with the plural subject ideas and by providing the phrase trying to find creative activities to replace the wordy when they try to find creative activities.. Choice (A) involves a subject-verb agreement error and wordiness. The singular verb is does not agree with the plural subject ideas, and the phrase when they try to find creative activities can be reduced to the concise modifying phrase trying to find creative activities.. Choice (C) results in illogical phrasing. It does not make sense to say that the ideas for songs, games, and skits are those with the invaluable creative activities scout leaders try to find.. Choice (D) involves subject-verb disagreement, improper coordination, and redundancy. The singular verb is does not agree with the plural subject ideas. In addition, the comma is used inappropriately to isolate the sentences main verb (is). And finally, the title of the guide makes its intended audience clear, so for scout leaders is redundant.. Choice (E) results in a sentence fragment. There is no main verb to carry out the action of the sentence.5 Explanation for Correct Answer A. Choice (A) is correct. It avoids the errors of the other options by providing a parallel list of nouns facts, opinions, untruths, and different accounts of the same events that logically describe what the biographer must sift through when researching an individuals life..

Choice (B) results in illogical phrasing and wordiness. It does not make sense to say that the biographer must sift throughdifferent people. In addition, the phrase different people telling different accounts of the same events can be reduced to the more succinct and clear different accounts of the same events.. Choice (C) results in wordiness. The phrase events told in different ways by different people can be reduced to the more succinct different accounts of the same events.. Choice (D) results in improper modification. It is unclear whether the sentence is indicating that the accounts are different or the events themselves.. Choice (E) results in illogical phrasing. It does not make sense to say that the biographer must sift throughpeople. In addition, it is not clear whether the people are telling different accounts of the same or different events. 6 Explanation for Correct Answer D. Choice (D) is correct. It avoids the lack of parallelism and the pronoun shift of the original by providing the gerund form of the verb using to agree with the preceding verbs talking and gesturing, and by avoiding the pronoun you, which is not consistent with the preceding pronoun one.. Choice (A) involves unidiomatic phrasing and a pronoun-shift error. The phrase to use gestures when you converse makes it easier is not idiomatic in this context and so should be changed to using gesturesmakes it easier. In addition, the pronoun you is not consistent with the preceding pronoun one.. Choice (B) involves a lack of parallelism, vague phrasing, and a pronoun-shift error. The infinitive form of the verb to make does not agree with the preceding verbs talking and gesturing. In addition, the shift from the noun phrase many people to the pronoun you makes it unclear who is thinking and speaking and who is gesturing. Lastly, the second person pronoun you is not consistent with the preceding pronoun one.. Choice (C) involves unidiomatic phrasing. The phrase for the ease of many people during conversations is not idiomatic.. Choice (E) involves a lack of parallelism and vague phrasing. The infinitive to speak is not parallel with the preceding verb form thinking. In addition, the phrase when gesturing while conversing is vague. It is not clear who or what is doing the gesturing.7 Explanation for Correct Answer D. Choice (D) is correct. It avoids the comma splice error of the original by providing the introductory conjunction Although.. Choice (A) involves a comma splice. Two independent clauses (There is increasingly widespread reliance on electronic mail and some people still resist using it, especially those who prefer handwritten letters) are improperly joined by only a comma.. Choice (B) results in a comma splice and includes a subject-verb agreement error. Two independent clauses (There are increasingly widespread reliance on electronic mail and some people still resist using it, especially those who prefer handwritten letters) are improperly joined by only a comma. In addition, the plural verb are does not agree with the singular subject reliance..

Choice (C) results in a coordination error and a subject-verb agreement error. It does not make sense to say that some people still resist electronic mail because there is increasingly widespread reliance on it. In addition, the plural verb are does not agree with the singular subject reliance.. Choice (E) results in a coordination error. The dependent clause beginning with In that does not appropriately establish the contrast between the concept that there is increasingly widespread reliance on electronic mail and the concept that some people still resist using it.8 Explanation for Correct Answer E. Choice (E) is correct. It avoids the comma splice error of the original by providing a dependent clause beginning with Because.. Choice (A) involves a comma splice. Two independent clauses (The ancient Egyptians had a varied diet and ate no refined sugar and they did not suffer from the tooth decay associated withprocessed sugar) are improperly joined by only a comma.. Choice (B) results in a sentence fragment. There is no main verb to carry out the action of the sentence, only the relative clause who had a varied diet and did not eat refined sugar.. Choice (C) involves unidiomatic phrasing. The phrase and them not eating refined sugar is not idiomatic and should be reworded as the modifying phrase free of refined sugar.. Choice (D) results in wordiness and improper modification. The word What, which introduces the sentence, is unnecessary and should be deleted. In addition, it does not make sense to say that the Egyptians were being free of refined sugar.9 Explanation for Correct Answer D. Choice (D) is correct. It avoids the improper modification error of the original by placing the subject tourist immediately after the modifying phrase When visiting a rain forest.. Choice (A) involves improper modification. It does not make sense to say that the preservation of the environment is visiting a rain forest.. Choice (B) involves improper modification. It does not make sense to say that the environment and its preservation are visiting a rain forest.. Choice (C) involves improper modification. It does not make sense to say that the preservation of the environment is visiting a rain forest.. Choice (E) results in wordiness. The wordy clause the tourist, regarding the preservation of the environment, should be concerned should be replaced with the concise the tourist should be concerned with the preservation of the environment.10 Explanation for Correct Answer B. Choice (B) is correct. It avoids the subject-verb agreement error of the original by providing the plural verb require to agree with the plural subject regulations..

Choice (A) involves a subject-verb agreement error. The singular verb requires does not agree with the plural subject regulations.. Choice (C) results in a sentence fragment. There is no main verb to carry out the action of the sentence, only the participle producing.. Choice (D) results in a sentence fragment. There is no main verb to carry out the action of the sentence, only the verb requires, which is part of the modifying phrase that requires automakers to produce.. Choice (E) results in a sentence fragment. There is no main verb to carry out the action of the sentence, only the participle producing. 11 Explanation for Correct Answer D. Choice (D) is correct. It avoids the comma splice error and the unidiomatic phrasing of the original by providing a semicolon to appropriately join two independent clauses (A severe northeastern storm struck Boston on New Years Day, 1900, slowing down the loading of ships in the harbor and the city was caught off guard because the Weather Bureau was closed for the holidays). In addition, the unidiomatic phrase because of the Weather Bureau being closed is replaced with the idiomatic because the Weather Bureau was closed.. Choice (A) involves a comma splice and unidiomatic phrasing. Two independent clauses (A severe northeastern storm struck Boston on New Years Day, 1900, slowing down the loading of ships in the harbor and the city was caught off guard because of the Weather Bureau being closed for the holidays) are improperly joined by only a comma. In addition, the phrase because of the Weather Bureau being closed is unidiomatic and should be replaced with because the Weather Bureau was closed.. Choice (B) results in a comma splice. Two independent clauses (A severe northeastern storm struck Boston on New Years Day, 1900, slowing down the loading of ships in the harbor and because the Weather Bureau was closed for the holidays, the city was caught off guard) are improperly joined by only a comma.. Choice (C) results in a coordination error. An independent clause (A severe northeastern storm struck Boston on New Years Day, 1900, slowing down the loading ships in the harbor) is improperly linked to a dependent clause (catching the city off guard because the Weather Bureau was closed for the holidays) with a semicolon.. Choice (E) results in improper modification and an improper noun case. It is unclear to what the modifying phrase which was caused by the Weather Bureau being closed for the holidays is referring. In addition, the noun Weather Bureau must be the possessive Weather Bureaus when preceding the gerund being.12 Explanation for Correct Answer C. The error in this sentence occurs at (C). The participle issuing cannot function as a main verb to complete the action of the sentence and should be replaced by the past-tense verb issued.. There is no error at (A). The preposition for idiomatically follows the noun medal and appropriately introduces the prepositional phrase for his groundbreaking discoveries..

There is no error at (B). The adjective groundbreaking appropriately modifies the plural noun discoveries.. There is no error at (D). The singular pronoun him agrees with the subject to which it refers (Ernest Just).. 13 Explanation for Correct Answer B. The error in this sentence occurs at (B). The possessive pronoun their and the gerund having do not provide a main verb to carry out the action of the sentence and should be replaced with the pronoun they and the present-tense verb have.. There is no error at (A). The verb phrase would have idiomatically indicates an action that Many jazz enthusiasts must take under a certain set of circumstances. The infinitive form of the verb to admit idiomatically follows the verb phrase would have.. There is no error at (C). The adverb unjustly appropriately modifies the past participle overlooked.. There is no error at (D). The relative pronoun who appropriately refers to a person, Bennie Nawahi, and the past tense verb popularized appropriately describes an action that took place in the 1930s.. 14 Explanation for Correct Answer C. The error in this sentence occurs at (C). The modifying clause who established the Pulitzer Prizes in journalism and literature does not provide a main verb to complete the action of the sentence. The relative pronoun who should be deleted, making established the main verb.. There is no error at (A). The noun immigrant is the appropriate word to describe Joseph Pulitzer, who was born in Hungary and then moved to the United States.. There is no error at (B). The preposition in appropriately introduces the prepositional phrase in the field, which modifies innovator.. There is no error at (D). The preposition in idiomatically follows the proper noun Pulitzer Prizes and appropriately introduces the prepositional phrase in journalism and literature.. 15 Explanation for Correct Answer A. The error in this sentence occurs at (A). The relative pronoun which does not appropriately refer to a person (Ethel Payne) and should be replaced with the relative pronoun who.. There is no error at (B). The preposition of idiomatically follows the noun coverage and appropriately introduces the prepositional phrase of the Civil Rights movement.. There is no error at (C). The past-tense verb received is consistent with the past-tense verb (kept) that follows..

There is no error at (D). The conjunction while indicates that the action of keeping a diary occurred at the same time that Ethel Payne was living in Japan. In addition, the preposition in idiomatically follows the participle living and introduces the prepositional phrase in Japan.. 16 Explanation for Correct Answer C. The error in the sentence occurs at (C). The phrase is why is awkward and unnecessary and should be replaced with a comma to join the introductory prepositional phrase (By incorporating Pueblo figures into her strongly geometric and abstract work) to the main clause (Pueblo artist Helen Hardin has had a significant impact on contemporary Native American art).. There is no error at (A). The present participle incorporating appropriately describes an action that is ongoing at the time of the main verb phrase has had a significant impact.. There is no error at (B). The adverb strongly appropriately modifies the adjective geometric, which modifies the noun work.. There is no error at (D). The preposition on idiomatically follows the noun impact and appropriately introduces the prepositional phrase on contemporary Native American art.. 17 Explanation for Correct Answer A. The error in this sentence occurs at (A). The plural verb are does not agree with the singular subject list and should be replaced by the singular verb is.. There is no error at (B). The conjunction but appropriately serves to link the two contrasting adjectives short and substantive.. There is no error at (C). The past participle resulted, which is idiomatically followed by the preposition from, appropriately modifies the plural noun innovations.. There is no error at (D). The preposition of idiomatically follows the noun efforts and appropriately introduces the prepositional phrase of Winifred Merrill.. 18 Explanation for Correct Answer C. The error in this sentence occurs at (C). The verb phrase did not become is inconsistent with the present perfect verb phrase have had and should be replaced with the past perfect verb phrase had not become to indicate an action that was completed before something (e.g., the bus taking the detour) in the past.. There is no error at (A). The infinitive verb phrase to take serves appropriately as the direct object of the verb phrase have had.. There is no error at (B). The phrase instead of is idiomatic and serves appropriately in the context of the sentence to mean in place of.. There is no error at (D). The preposition in appropriately introduces the prepositional phrase in the aftermath, and the preposition of, which

idiomatically follows the noun aftermath, appropriately introduces the prepositional phrase of an ice storm.. 19 Explanation for Correct Answer E. The correct choice is (E). There are no errors in this sentence.. There is no error at (A). The superlative most appropriately modifies the adverb frequently, which appropriately modifies the past participle taught.. There is no error at (B). The singular verb remains agrees with the singular subject Light in August.. There is no error at (C). The article the appropriately precedes the superlative least, which appropriately modifies the past participle understood.. There is no error at (D). The coordinating conjunction either appropriately links the two adjectives intellectual and aesthetic.20 Explanation for Correct Answer D. The error in the sentence occurs at (D). The singular pronoun it does not agree with the plural noun (lightning strikes) to which it refers.. There is no error at (A). The infinitive verb phrase to record appropriately serves to modify the noun sensors.. There is no error at (B). The preposition throughout appropriately introduces the prepositional phrase throughout the United States.. There is no error at (C). The past participle determined serves as part of the present perfect verb phrase have determined, which appropriately describes an action completed in an unspecified time before the present. The conjunction that idiomatically follows the verb phrase have determined and introduces the subordinate clause.. 21 Explanation for Correct Answer E. The correct choice is (E). There are no errors in this sentence.. There is no error at (A). The preposition on idiomatically follows the noun research and appropriately introduces the prepositional phrase on the diagnosis. The preposition of idiomatically follows the noun diagnosis and appropriately introduces the prepositional phrase of heart disease.. There is no error at (B). The adverb increasingly appropriately modifies the adjective sophisticated.. There is no error at (C). The preposition in idiomatically follows the phrase the use of sophisticated technology and appropriately introduces the prepositional phrase in its treatment. The singular possessive pronoun its agrees with the noun heart disease to which it refers.. There is no error at (D). The singular verb is agrees with the singular subject condition.22 Explanation for Correct Answer B.

The error in this sentence occurs at (B). The plural possessive pronoun their does not agree with the singular subject African violet and should be replaced with the singular possessive pronoun its.. There is no error at (A). The preposition among appropriately describes the African violet as part of a group of the most popular houseplants.. There is no error at (C). The plural verb come agrees with the plural subject flowers.. There is no error at (D). The preposition of idiomatically follows the noun array and appropriately introduces the prepositional phrase of colors.. 23 Explanation for Correct Answer E. The correct choice is (E). There are no errors in this sentence.. There is no error at (A). The word but appropriately serves as a conjunction meaning except in the context of this sentence.. There is no error at (B). The adverb readily appropriately modifies the verb lend.. There is no error at (C). The singular verb phrase is known agrees with the singular subject a person.. There is no error at (D). The phrase to be idiomatically serves to connect the verb phrase is known with the noun phrase a frequent gambler. 24 Explanation for Correct Answer B. The error in this sentence occurs at (B). The singular pronoun it does not agree with the plural subject plans. The pronoun it should be replaced with the plural pronoun they.. There is no error at (A). The adverb perfectly appropriately modifies the adjective clear.. There is no error at (C). The semicolon, the adverb however, and the comma serve together to appropriately link the first independent clause (The plans proposed by the students for the homecoming celebrations are perfectly clear budget) with the second independent clause (the plans are not likely to be approved by the administration).. There is no error at (D). The infinitive form of the verb to be idiomatically follows the verb phrase are not likely. The past participle approved idiomatically follows the infinitive verb phrase to be and appropriately modifies the noun plans.. 25 Explanation for Correct Answer E. The correct choice is (E). There are no errors in this sentence.. There is no error at (A). The adverb carefully appropriately modifies the verb phrase had planned.. There is no error at (B). The conjunction for, along with the comma, appropriately links the first independent clause (It was obvious that all of the

candidates had planned carefully for the televised debate) with the second independent clause (each answer to the opening question showed evidence of having been rehearsed). In addition, the adjective each appropriately modifies the noun answer.. There is no error at (C). The preposition to idiomatically follows the noun answer and introduces the prepositional phrase to the opening question.. There is no error at (D). The preposition of idiomatically follows the noun evidence and the participial phrase having been rehearsed.26 Explanation for Correct Answer D. The error in this sentence occurs (D) The adjective desirous means having or expressing desire. It does not make sense to say that an understanding of current issues is expressing desire. The adjective desirous should be replaced with desirable, which means worth having or wanting.. There is no error at (A). The preposition For appropriately introduces the prepositional phrase For any mayor of a vast metropolitan area.. There is no error at (B). The gerund understanding is appropriately followed by the preposition of, which introduces the prepositional phrase of current issues.. There is no error at (C). The singular pronoun is agrees with the singular subject an understanding.. 27 Explanation for Correct Answer E. The correct choice is (E). There are no errors in this sentence.. There is no error at (A). The phrase Because of is an idiom that appropriately serves to introduce the phrase Because of the exceptionally hilly terrain, which modifies the main clause the final miles of the racecourse were difficult for the other runners and me.. There is no error at (B). The adverb exceptionally appropriately modifies the adjective hilly.. There is no error at (C). The plural verb were agrees with the plural subject miles, which is appropriately modified by the subject compliment difficult.. There is no error at (D). The pronoun me is part of the noun phrase the other runners and me, which appropriately serves as the object of the preposition for.28 Explanation for Correct Answer A. The error in this sentence occurs at (A). The preposition to in the prepositional phrase to her family does not idiomatically follow the verb phrase was considered in the context of this sentence and should be replaced by the preposition by.. There is no error at (B). The adverb already appropriately modifies the verb able.. There is no error at (C). The adverb delicately appropriately modifies the verb grasp..

There is no error at (D). The preposition between appropriately introduces the prepositional phrase between her thumb and forefinger, and the possessive pronoun her agrees with the pronoun she that precedes it.. 29 Explanation for Correct Answer E. The correct choice is (E). There are no errors in this sentence.. There is no error at (A). The preposition To appropriately introduces the prepositional phrase To those of us, which modifies participle surprising. The plural pronouns those and us are in agreement, and those is appropriately followed by the preposition of.. There is no error at (B). The past perfect verb phrase had heard appropriately describes an action that was completed before another action in the past.. There is no error at (C). The preposition about appropriately introduces the prepositional phrase about the budget, which modifies the verb talk.. There is no error at (D). The past-tense verb was appropriately describes and action that was completed in the past and the adverb not appropriately serves to negate the participle surprising.30 Explanation for Correct Answer D. Choice (D) is correct. The noun phrase this sentiment appropriately replaces the vague pronoun it.. Choice (A) is unsatisfactory. The underlined portion of sentence 2 contains a vague pronoun. There is nothing in the sentence to which it can logically refer.. Choice (B) is unsatisfactory. Replacing the underlined portion of sentence 2 with the phrase to be told about it does not solve the problem of the vague pronoun it.. Choice (C) is unsatisfactory. Replacing the underlined portion of sentence 2 with the prepositional phrase from the source creates a sentence that does not make sense in the context of the sentence or paragraph.. Choice (E) is unsatisfactory. Replacing the underlined portion of sentence 2 with the phrase to be perceived creates a sentence that does not make sense in the context of the sentence or paragraph.31 Explanation for Correct Answer A. Choice (A) is correct. The best version of sentence 3 is clear and grammatically correct.. Choice (B) is unsatisfactory. The introduction of the second person pronoun you is inappropriate in the context of the paragraph. In addition, the past-progressive verb phrase have been making is inappropriate following the infinitive To fail.. Choice (C) is unsatisfactory. The prepositional phrase In failing is unidiomatic and should be replaced with the infinitive for of the verb To fail. In addition, the adverb first should precede the verb (make) that it modifies.. Choice (D) is unsatisfactory. The intent of this sentence (First, you must make an attempt to have failed) is different from the intent of the original (To fail,

one must first make an attempt). In addition, the introduction of the second person pronoun you is inappropriate in the context of the paragraph.. Choice (E) is unsatisfactory. The adverb first should be placed immediately before the verb make that it modifies. 32 Explanation for Correct Answer B. Choice (B) is correct. The introductory phrase In other words indicates that sentence 5 is a restatement of the ideas in sentence 4.. Choice (A) is unsatisfactory. The introductory word Nevertheless inappropriately indicates that the concept in sentence 5 (even if one does not always accomplish what he or she sets out to do, one gains new knowledge from the attempt) happens despite the concept in sentence 4 (experience expands, new ideas blossom, and viewpoints change).. Choice (C) is unsatisfactory. The introductory word However inappropriately indicates that the ideas in sentence 5 (even if one does not always accomplish what he or she sets out to do, one gains new knowledge from the attempt) are in contrast to the ideas in sentence 4(experience expands, new ideas blossom, and viewpoints change).. Choice (D) is unsatisfactory. The introductory phrase In contrast inappropriately indicates that the concept in sentence 5 (even if one does not always accomplish what he or she sets out to do, one gains new knowledge from the attempt) is the opposite of the concept in sentence 4(experience expands, new ideas blossom, and viewpoints change).. Choice (E) is unsatisfactory. The introductory word Supposedly inappropriately indicates that the ideas in sentence 5 are supposed or unconfirmed. This does not make sense in the context of the sentence or paragraph.33 Explanation for Correct Answer B. Choice (B) is correct. The phrase One example provides an appropriate transition from sentence 5, and the resulting sentence is grammatically correct.. Choice (A) is unsatisfactory. There is no appropriate transition from sentence 5 to the revised sentence.. Choice (C) is unsatisfactory. This revision of the underlined portion of sentences 6 and 7 contains wordy, redundant phrasing. The phrases, Take as an example medication, for instance should be replaced with One example is medication.. Choice (D) is unsatisfactory. There is nothing in the paragraph to indicate that the first person plural pronouns We and us should be used in a revision of the underlined portion of sentences 6 and 7.. Choice (E) is unsatisfactory. It results in improper modification. It does not make sense to say that drugs are Regarding medication. 34 Explanation for Correct Answer B. Choice (B) is correct. The modifying phrase failing at least once appropriately modifies the subject drugs.. Choice (A) is unsatisfactory. It involves a sentence fragment. There is no main verb following the conjunction and, only the participle failing..

Choice (C) is unsatisfactory. This revision results in a sentence fragment. The semicolon should separate two independent clauses; however, the second clause failure occurring at least once is a dependent clause.. Choice (D) is unsatisfactory. The past-tense verb failed is inconsistent with the preceding present-tense verb go.. Choice (E) is unsatisfactory. The present perfect verb phrase have failed is inconsistent with the preceding present-tense verb go.35 Explanation for Correct Answer A. Choice (A) is correct. The subject (A scientist) is appropriately modified by the relative clause who finds that a hypothesis is false.. Choice (B) is unsatisfactory. The verb phrase can have tried is unidiomatic and should be replaced with the present-tense verb phrase can try.. Choice (C) is unsatisfactory. The phrase Finding a hypothesis as false is unidiomatic and should be replaced with Finding a hypothesis false. In addition, the verb phrase would try is inappropriate in the context of the paragraph and should be replaced with can try.. Choice (D) is unsatisfactory. The clause A false hypothesis can have a scientist now try is awkward and unidiomatic..

Choice (E) is unsatisfactory. There is nothing in the sentence to which the pronoun they can logically refer. Section #9: View Explanations 1 Explanation for Correct Answer B. Choice (B) is correct. To improvise is to invent, compose, or perform with little or no preparation. In this context, to introduce something is to present it for the first time. According to the sentence, Louis Armstrong often was spontaneous during performanceshis performances contained elements that arose from momentary impulse rather than from careful planning. Therefore, it makes sense to say that Armstrong "frequently improvised." Further, the sentence implies that Armstrong's performances were always changing"Unlike some entertainers whose performances rarely varyso it is logical to suggest that when he played existing songs he would introduce "new phrasing" that his audience had never heard before.. Choice (A) is incorrect. In this context, to vocalize is to sing. To eliminate something means to put an end to it or to get rid of it. According to the sentence, Louis Armstrong often was spontaneous during performanceshis performances contained elements that arose from momentary impulse rather than from careful planning. Armstrong may have vocalized, or sung, during his performances, but this information is unrelated to the claim that Armstrong was spontaneous and was not like "some entertainers whose performances rarely vary." Additionally, the sentence mentions "new phrasing" in Armstrong's songs. It does not make sense to suggest that Armstrong, if he were acting spontaneously, would eliminate new phrasing in his songs; rather than eliminating new phrasing, a

spontaneous singer would be likely to get rid of old phrasing and include new phrasing.. Choice (C) is incorrect. To deliberate is to think about or discuss issues and decisions carefully. Someone who satirizes uses irony, sarcasm, or wit to attack or make fun of something. According to the sentence, Louis Armstrong often was spontaneous during performanceshis performances contained elements that arose from momentary impulse rather than from careful planning. Therefore, it does not make sense to say that Armstrong "deliberated" during his performances. Further, although Armstrongs songs may very well have contained an element of satire, it is somewhat illogical to suggest that he would satirize, or make fun of, his own new phrasing in songs.. Choice (D) is incorrect. In this context, to ad-lib is to perform spontaneously or to make something up on the spot. To rehearse something is to practice it in preparation for a performance. According to the sentence, Louis Armstrong often was spontaneous during performanceshis performances contained elements that arose from momentary impulse rather than from careful planning. The term "ad-libbed" makes sense in the first blank, but the term "rehearsing" does not fit the second blank. These two terms are opposite in meaning: an ad-libbed performance is by definition an unrehearsed performance. Further, a performer typically rehearses before a performance in preparation for it, not during a performance.. Choice (E) is incorrect. To extemporize is to improvise or to do something spontaneously, with little or no preparation. To bungle something is to mishandle or botch it. According to the sentence, Louis Armstrong often was spontaneous during performanceshis performances contained elements that arose from momentary impulse rather than from careful planning. Because Armstrong was spontaneous when he performed, it makes sense to say he "extemporized." However, the term "bungling" does not logically complete the sentence. It is certainly possible that a performer could mishandle a new part of a song that is unfamiliar to him or her, but there is not necessarily a connection between extemporizing and bungling "new phrasing in existing songs." Armstrong is referred to as a jazz great and seems to have added new phrasing of his choice, so there is no reason to believe that he would have bungled his performances.2 Explanation for Correct Answer B. Choice (B) is correct. Something that is premature happens before it is expected to happen. To suppress something is to restrain or subdue it. The structure of the sentence indicates that there is a contrasting relationship between the two missing terms: Julia did not want to speak in a certain manner, so she did something that would result in the opposite outcome. The terms "prematurely" and "suppressed" support this contrast and logically complete the sentence. Because Julia did not want to "speak prematurely" and "announce her scientific findings" before the appropriate timebefore she had double-checked the data she suppressed or restrained her urge to make her findings known to the public.. Choice (A) is incorrect. Someone who speaks "harshly" does so in a severe or cruel manner. To indulge is to give in to something or yield to a desire. The structure of the sentence indicates that there is a contrasting relationship between the two missing terms: Julia did not want to speak in a certain manner, so she did something that would result in the opposite outcome. The terms "harshly" and "indulged" do not exhibit this kind of relationship. Julia would not necessarily have avoided speaking harshly by indulging and giving in to "the urge to announce her scientific findings; there is no direct connection between speaking harshly and giving in to an urge. Further, it does not make much sense

to say that "Julia indulged the urge to announce her scientific findings until she had double-checked the data"; it would make more sense to say that Julia indulged the urge to announce her findings before the data had been checked.. Choice (C) is incorrect. Someone who speaks inappropriately speaks in an improper way. To acknowledge something is to admit the existence, reality, or truth of it. The structure of the sentence indicates that there is a contrasting relationship between the two missing terms: Julia did not want to speak in a certain manner, so she did something that would result in the opposite outcome. The terms "inappropriately" and "acknowledged" do not exhibit such a relationship. Even if Julia acknowledged "the urge to announce her scientific findings," it is not clear whether or not she actually made the announcement; she could have simply admitted that she wanted to announce her findings. Because it is unclear whether or not Julia made the announcement at an improper time (before double-checking the data), it cannot be determined whether or not she is speaking inappropriately.. Choice (D) is incorrect. To speak "rashly" is to do so quickly, with no deliberation. In this context, to advance something is to cause it to occur sooner. The structure of the sentence indicates that there is a contrasting relationship between the two missing terms: Julia did not want to speak in a certain manner, so she did something that would result in the opposite outcome. The terms "rashly" and "advanced" do not support the necessary contrast. If Julia "advanced the urge to announce her scientific findings," it is likely that she would have revealed the information quickly and without double-checking the data; if that were the case, Julia would have indeed spoken rashly. Further, it is somewhat awkward to say that Julia advanced the urge to announce her findings; it would make more sense to say that she advanced the actual announcement.. Choice (E) is incorrect. To speak "enthusiastically" is to speak with great excitement or feeling. In this context, to define something is to explain its nature or basic qualities. The structure of the sentence indicates that there is a contrasting relationship between the two missing terms: Julia did not want to speak in a certain manner, so she did something that would result in the opposite outcome. The terms "enthusiastically" and "defined" do not exhibit such a relationship. Julia would not necessarily have avoided speaking enthusiastically by defining her urge to announce her scientific findings before she double-checked the data. In fact, an explanation of a strong urge might involve great excitement or feeling.3 Explanation for Correct Answer D. Choice (D) is correct. "Immoral" means conflicting with generally or traditionally held moral principles. "Transgressions" are infringements or violations of a law, command, or duty. According to the sentence, the protagonists, or main characters, in Patricia Highsmith's crime fiction "are not always punished." As a result, "Some readers find" Highsmith's fiction to be a certain way. The sentence suggests that these readers are concerned about Highsmith's failure to "always" punish her characters for things that (presumably) deserve punishment. It is logical to expect people to be punished for violations of laws, so the term "transgressions" fits in the second blank. Further, traditional moral principles demand that lawbreakers be punished; if Highsmith's characters sometimes escape punishment for breaking the law, some readers might consider Highsmith's books immoral.. Choice (A) is incorrect. "Unprincipled" means lacking moral principles. "Altruism" is unselfish regard for or devotion to the welfare of others. According to the sentence, the protagonists, or main characters, in Patricia Highsmith's crime

fiction "are not always punished." As a result, "Some readers find" Highsmith's fiction to be a certain way. The sentence suggests that these readers are concerned about Highsmith's failure to "always" punish her characters for things that (presumably) deserve punishment. The term "unprincipled" fits the first blankreaders might consider an author's writing to be unprincipled if the main characters are not punished when they should bebut the term "altruism" does not logically complete the sentence. Readers concerned with moral principles would not wish to see characters punished for their altruism, or selfless service to others.. Choice (B) is incorrect. "Ethical" means conforming to accepted standards of conduct. "Malfeasance" is wrongdoing or misconduct, especially by a public official. According to the sentence, the protagonists, or main characters, in Patricia Highsmith's crime fiction "are not always punished." As a result, "Some readers find" Highsmith's fiction to be a certain way. The sentence suggests that these readers are concerned about Highsmith's failure to "always" punish her characters for things that (presumably) deserve punishment. It is illogical to suggest that readers would consider an author's writing to be ethical if the main characters are not punished for their malfeasance; in an ethical book, characters that commit acts of misconduct likely would suffer for those acts.. Choice (C) is incorrect. In this context, "redemptive" means of, relating to, or bringing about the process of atoning or making amends for one's mistakes. "Temperance" is moderation in action, thought, or feeling. According to the sentence, the protagonists, or main characters, in Patricia Highsmith's crime fiction "are not always punished." As a result, "Some readers find" Highsmith's fiction to be a certain way. The sentence suggests that these readers are concerned about Highsmith's failure to "always" punish her characters for things that (presumably) deserve punishment. It is somewhat illogical to suggest that readers would say that characters should be punished for their temperance. Additionally, there is no reason to believe that readers would consider an author's writing to be redemptive because the main characters are not punished for their temperance.. Choice (E) is incorrect. "Prescriptive" means dictating or authorizing, and usually refers to a certain rule or code of conduct. A foible is a minor flaw or shortcoming in character or behavior. According to the sentence, the protagonists, or main characters, in Patricia Highsmith's crime fiction "are not always punished." As a result, "Some readers find" Highsmith's fiction to be a certain way. The sentence suggests that these readers are concerned about Highsmith's failure to "always" punish her characters for things that (presumably) deserve punishment. The term "foible" fits the second blanksome readers might feel that people should be punished for their flaws or shortcomingsbut the term "prescriptive" does not logically complete the sentence. If the main characters are not punished for their foibles, there is no reason to believe that readers would think that the author's writing asserting certain rules of conduct.4 Explanation for Correct Answer E. Choice (E) is correct. "Regressive" means tending to regress or go backward. The sentence indicates that humans learned to use fire "slowly over time," with "countless advances offset by" other periods. Something that offsets something else is a counterbalance: if one thing is offset by another, the two things must be opposites. Therefore, the missing term must contrast with the term "advances." The term "regressive" creates the necessary contrast: the progress made in learning to use fire was offset, or counterbalanced, by regressive periods, or periods of backward movement..

Choice (A) is incorrect. "Explosive" means likely to blow up or produce hostile action or violence. The sentence indicates that humans learned to use fire "slowly over time," with "countless advances offset by" other periods. Something that offsets something else is a counterbalance: if one thing is offset by another, the two things must be opposites. Therefore, the missing term must contrast with the term "advances"; the term "explosive" does not necessarily create such a contrast. Although a time period of hostility and violence might impede any progress being made, the opposite might also be true; moreover, it does not make much sense to describe a time period in which little progress was made as an explosive period.. Choice (B) is incorrect. In this context, "fertile" means productive. The sentence indicates that humans learned to use fire "slowly over time," with "countless advances offset by" other periods. Something that offsets something else is a counterbalance: if one thing is offset by another, the two things must be opposites. Therefore, the missing term must contrast with the term "advances." The term "fertile" does not create such a contrast; in fact, it is likely that many advances, or steps in progress, would be made during a fertile time period. A fertile period would likely be characterized by progress.. Choice (C) is incorrect. Something that is "predictable" is something that one could predict; in other words, it is as one would expect it to be. The sentence indicates that humans learned to use fire "slowly over time," with "countless advances offset by" other periods. Something that offsets something else is a counterbalance: if one thing is offset by another, the two things must be opposites. Therefore, the missing term must contrast with the term "advances." The term "predictable" does not create such a contrast. Because a predictable period of time would not necessarily counterbalance progress that had been made, the term "predictable" does not logically complete the sentence.. Choice (D) is incorrect. "Contemplative" means marked by or given to thoughtful observation or study. The sentence indicates that humans learned to use fire "slowly over time," with "countless advances offset by" other periods. Something that offsets something else is a counterbalance: if one thing is offset by another, the two things must be opposites. Therefore, the missing term must contrast with the term "advances." The term "contemplative" does not create such a contrast. Indeed, a period of thoughtful study could potentially enhance or strengthen any progress that had been made.5 Explanation for Correct Answer E. Choice (E) is correct. In this context, to "arrest" something is to put a stop to it. According to the sentence, conservationistspeople who advocate the protection of natural resourceshoped to "preserve natural habitats." In an effort to do so, they tried to put certain laws in place; it is logical to assume that they lobbied for legislation that would stop or halt commercial developmentthat is, the construction of business offices and buildingsbecause such development might endanger or destroy natural habitats. Therefore, it makes sense to suggest that the conservationists lobbied for laws that would arrest commercial development in natural habitats.. Choice (A) is incorrect. To "mandate" is to order something or make something mandatory. According to the sentence, conservationists hoped to "preserve natural habitats." In an effort to do so, they tried to put certain laws in place; it is logical to assume that they lobbied for legislation that would stop or halt commercial developmentthat is, the construction of business offices and buildingsbecause such development might endanger or destroy natural habitats. It does not make sense to suggest that conservationists would

encourage legislation that mandated, or ordered, commercial development in natural habitats; such development likely would destroy those habitats, not preserve them.. Choice (B) is incorrect. In this context, to "accommodate" is to allow for the consideration of something. According to the sentence, conservationists hoped to "preserve natural habitats." In an effort to do so, they tried to put certain laws in place; it is logical to assume that they lobbied for legislation that would stop or halt commercial developmentthat is, the construction of business offices and buildingsbecause such development might endanger or destroy natural habitats. It does not make sense to suggest that conservationists would encourage legislation that allows for the consideration of commercial development in natural habitats; they likely would want to stop, not accommodate, commercial development.. Choice (C) is incorrect. To "economize" is to be frugal or to save money. According to the sentence, conservationists hoped to "preserve natural habitats." In an effort to do so, they tried to put certain laws in place; it is logical to assume that they lobbied for legislation that would stop or halt commercial development that is, the construction of business offices and buildingsbecause such development might endanger or destroy natural habitats. It is unlikely that conservationists would be concerned with making commercial development more economical by saving money; rather, they probably would not support commercial development at all. Therefore, the term economize does not logically complete the sentence.. Choice (D) is incorrect. To "diversify" something is to increase its diversity or give it variety. According to the sentence, conservationists hoped to "preserve natural habitats." In an effort to do so, they tried to put certain laws in place; it is logical to assume that they lobbied for legislation that would stop or halt commercial developmentthat is, the construction of business offices and buildingsbecause such development might endanger or destroy natural habitats. There is no reason to believe that conservationists would lobby for legislation that allowed businesses to diversify in natural environments; they probably would want to preserve natural habitats by putting a stop to the expansion or development of commercial centers altogether.6 Explanation for Correct Answer E. Choice (E) is correct. To "relinquish" something is to give it up. The term "itinerant" refers to traveling from place to place. The sentence indicates that Joan used to be a traveling musician but that her "new job required daily attendance"in other words, that she be there every day. A traveling musician would travel from place to place, so it makes sense to describe Joans lifestyle as a musician as itinerant. And because traveling would not allow her to be at her new job every day, it is likely that Joan would have had to to relinquish, or give up, this itinerant lifestyle.. Choice (A) is incorrect. To "forgo" something is to give it up. Sedentary means physically inactive. The sentence indicates that Joan used to be a traveling musiciana musician who travels from place to placebut took a new job that required daily attendancethat is, that she be there every day. Because traveling would not allow her to be at her new job every day, it is likely that Joan would have had to forgo, or give up, being a traveling musician. However, it is somewhat illogical to describe the lifestyle of such a musician as sedentary, or inactive, because this lifestyle involves extensive traveling from one place to another..

Choice (B) is incorrect. To perpetuate something is to prolong its existence. The term nomadic refers to roaming about from place to place frequently or without a fixed pattern of movement. The sentence indicates that Joan used to be a traveling musician but that her new job required daily attendancein other words, that she be there every day. A traveling musician would travel from place to place, so it makes sense to describe the lifestyle of such a musician as nomadic. However, traveling would not allow Joan to be at her new job every day, so it is unlikely that she would have had to perpetuate her nomadic lifestyle; Joan probably would have had to give up, not prolong, that lifestyle. Choice (C) is incorrect. To "glorify" something is to honor, praise, or admire it. The term "dissolute" refers to lacking restraint or indulging in immoral behavior. The sentence indicates that Joan used to be a traveling musician but that her new job required daily attendancein other words, that she be there every day. Nothing in the sentence suggests that Joan had a dissolute lifestyle as a traveling musician; musicians who travel from place to place do not necessarily indulge in immoral behavior. Further, there is no reason to believe that Joans new job would require her to glorify, or praise, her former lifestyle.. Choice (D) is incorrect. To "augment" something is to make it greater, more numerous, or more intense. Lavish means characterized or marked by extravagance or excess. The sentence indicates that Joan used to be a traveling musician but that her new job required daily attendancein other words, that she be there every day. Nothing in the sentence indicates that Joans lifestyle as a traveling musician was lavish; the lifestyle of such a musician does not necessarily involve extravagance or excess. Further, because traveling would not allow her to be at her job every day, Joan probably would have had to give up her former lifestyle, not augment or intensify it.7 Explanation for Correct Answer D. Choice (D) is correct. Both authors are very interested in the translation of ancient writings, or the recasting of very old poems into modern language for modern readers. The two authors offer differing viewpoints on the issue of whether or not literal translation is the best way to recast an ancient work. The author of Passage 1 examines this issue by presenting the opposing views of two nineteenth-century scholarsFrancis William Newman, who attempted a literal translation of the works of Homer and believed that a literal translation must be the most faithful translation, and Matthew Arnold, who reviewed Newmans work and pointed out that a literal translation made for oddity and for uncouthness and created false emphases. Passage 2 focuses on Ezra Pounds translation of The Seafarer, one that was not so much a translation as a new poem in the spirit of the original.. Choice (A) is incorrect. The authors of the two passages are not similar in their idealization of certain ancient poets. The author of Passage 1 praises Homer, but nothing in Passage 2 suggests that the author idealizes, or especially admires, certain ancient poets. Although the author of Passage 2 refers to an ancient poem (the original version of The Seafarer), he or she does not express great admiration for its original author of for any other ancient poet. Both authors focus on the translation of ancient poetry rather than on the ancient poets themselves.. Choice (B) is incorrect. The authors of the two passages certainly do not express any disdain, or contempt, for scholarly debate. On the contrary, both authors use scholarly debate as the basis for their passages: they offer differing viewpoints on the issue of whether or not literal translation is the best way to recast an ancient work..

Choice (C) is incorrect. The authors of the two passages discuss the translation of ancient poetryits purpose and its impact on modern readers understanding of the literary works. The authors offer differing viewpoints on the issue of whether or not literal translation is the best way to recast an ancient work. Neither author directly compares ancient poetry and modern poetry, and neither author expresses a preference for ancient over modern poetry.. Choice (E) is incorrect. The authors offer differing viewpoints on the issue of whether or not literal translation is the best way to recast an ancient work. However, only the author of Passage 2 includes a discussion of the rendering of sound as part of the translation process; he or she focuses on Ezra Pounds translation of The Seafarer, a translation that aims to reproduce the feel of the original by reproducing Anglo-Saxon sounds. The author of Passage 1 does not mention the rendering of sound in his or her examination of the process of translation.8 Explanation for Correct Answer D. Choice (D) is correct. The author of Passage 2 discusses Ezra Pounds translation of the The Seafarer in order to express the importance of conveying the sense and spirit of an original work, whether or not the modern words correspond literally to the meaning of the original words. In other words, the author suggests that literal translation does not necessarily result in the most accurate interpretation of a literary work and that the translator should try, like Pound, "to reproduce the 'feel' of the original." The author of Passage 2 would most likely be interested in whether or not Popes translation of Homer faithfully preserved the essence or the "feel" of the original poem.. Choice (A) is incorrect. The author of Passage 2 discusses Ezra Pounds translation of the The Seafarer in order to express the importance of conveying the sense and spirit of an original work, whether or not the modern words correspond literally to the meaning of the original words. The translator should try, like Pound, "to reproduce the 'feel' of the original." Pound was able to replicate some Anglo-Saxon sounds in English, and the author of Passage 2 might like to see Pope do the same thing with ancient Greek verse; however, the author of Passage 2 would say that the simple replication of sounds is not enough. He or she is concerned about the preservation of the spirit of the original, which includes its meaning; replicating sounds is only a component of this larger concern.. Choice (B) is incorrect. The author of Passage 2 discusses Ezra Pounds translation of the The Seafarer in order to express the importance of conveying the sense and spirit of an original work, whether or not the modern words correspond literally to the meaning of the original words. In other words, the author suggests that literal translation does not necessarily result in the most accurate interpretation of a literary work and that the translator should try, like Pound, "to reproduce the 'feel' of the original." The author of Passage 2 is concerned about the preservation of the spirit of the original rather than the exact reproduction of each Greek word. He or she explicitly states that to translate accurately you must do more than find words that have the same meaning as words in another language.. Choice (C) is incorrect. The author of Passage 2 discusses Ezra Pounds translation of the The Seafarer in order to express the importance of conveying the sense and spirit of an original work, whether or not the modern words correspond literally to the meaning of the original words. The author suggests that the translator should try, like Pound, "to reproduce the 'feel' of the original." He or she is not concerned with the thoughts and beliefs of modern scholars with

regard to the translation of ancient works. The author of Passage 2 simply references the translation methods of one poet in order to convey his or her viewpoint on the issue. Although he or she mentions that Pound offended some scholars when he published his translation in the early 20th century, the author of Passage 2 does not discuss standards of modern scholarship.. Choice (E) is incorrect. The author of Passage 2 discusses Ezra Pounds translation of the The Seafarer in order to express the importance of conveying the sense and spirit of an original work, whether or not the modern words correspond literally to the meaning of the original words. In other words, the author suggests that literal translation does not necessarily result in the most accurate interpretation of a literary work and that the translator should try, like Pound, "to reproduce the 'feel' of the original." The author of Passage 2 is concerned with preserving the meaning and spirit of the original work; he or she does not mention any effort to make archaic ways of thinking appealing to modern readers.9 Explanation for Correct Answer B. Choice (B) is correct. According to Passage 1, nineteenth-century scholar Francis William Newman believed that a literal translation must be the most faithful translation. Ezra Pound would likely challenge Newmans assumption about the importance and accuracy of literal (word-for-word) translation. The author of Passage 2 points out that Pounds intent in translating a literary work was to reproduce the feel of the original . . . whether or not the modern words correspond literally to the meaning of the original words. Pound, who "offended scholars" with his non-literal translation of an old Anglo-Saxon poem, in which he focused on the sounds of the words as well as their meanings, likely would argue that the unique qualities of a poem go beyond the definitions of individual words.. Choice (A) is incorrect. According to Passage 1, nineteenth-century scholar Francis William Newman believed that a literal translation must be the most faithful translation." Ezra Pound would likely challenge Newmans assumption about the importance and accuracy of literal (word-for-word) translation. The author of Passage 2 points out that Pound showed that to translate accurately you must do more than find words that have the same meaning as words in another language. The author states that Pounds version of The Seafarer does something that a literal translation fails to dorenders it into poetic English, finding new equivalents for old emotions. Both Newman and Pound seem to have assumed that modern readers will have no trouble understanding ideas prized by ancient audiences if the poems are translated accurately; they just differed on what constitutes an accurate translation.. Choice (C) is incorrect. According to Passage 1, nineteenth-century scholar Francis William Newman believed that a literal translation must be the most faithful translation." Ezra Pound would likely challenge Newmans assumption about the importance and accuracy of literal (word-for-word) translation. The author of Passage 2 points out that Pounds intent in translating a literary work was to reproduce the feel of the original . . . whether or not the modern words correspond literally to the meaning of the original words. There is nothing in the passage to suggest that Pound believed that a translators choice of words is limited by a poems subtlety of meaning. On the contrary, Pound would likely argue that a translators choice of words is not limited, in that he or she can choose words that convey the appropriate idea or emotion even if those words do not have the exact meaning of the original words.. Choice (D) is incorrect. According to Passage 1, nineteenth-century scholar Francis William Newman believed that a literal translation must be the most

faithful translation." Ezra Pound would likely challenge Newmans assumption about the importance and accuracy of literal (word-for-word) translation. The author of Passage 2 points out that Pounds intent in translating a literary work was to reproduce the feel of the original . . . whether or not the modern words correspond literally to the meaning of the original words. Pound seems to have believed that the vision of the original poet can be preserved in loose or nonliteral translationsthe purpose of translation is to find words that bring over the sense and spirit of the originalso it is unlikely that he would use this point to challenge Newman.. Choice (E) is incorrect. According to Passage 1, nineteenth-century scholar Francis William Newman believed that a literal translation must be the most faithful translation." Ezra Pound would likely challenge Newmans assumption about the importance and accuracy of literal (word-for-word) translation. The author of Passage 2 points out that Pounds intent in translating a literary work was to reproduce the feel of the original . . . whether or not the modern words correspond literally to the meaning of the original words. The author states that Pounds version of The Seafarer does something that a literal translation fails to dorenders it into poetic English, finding new equivalents for old emotions. Pounds reference to these equivalents suggests that he might not have thought it was necessary for a reader to be familiar with a poems social context in order to understand it; he would probably argue that a proper translation allows modern readers to understand ideas without extensive background knowledge.10 Explanation for Correct Answer C. Choice (C) is correct. The author of Passage 1 refers to famous essayist and poet Matthew Arnold, who argued that a translator should always convey the impression of . . . qualities found in an original text, even in cases where the original text did not bear them out. Arnold felt that a literal translation made for oddity and for uncouthness. The author of Passage 1 provides an example of this oddity by discussing the literal translation of the Romance languages description of a winter day as It makes cold rather than familiar English phrase, It is cold. This example illustrates Arnolds claim that if a text is translated literally, then false emphases are created. In other words, Arnold felt that literal translation results in the meaning of a text becoming distorted and misleading. As it is used in line 24, the term false most nearly means misleading.. Choice (A) is incorrect. Something that is fictional is invented by the imagination, and is not real. The author of Passage 1 refers to famous essayist and poet Matthew Arnold, who argued that a translator should always convey the impression of . . . qualities found in an original text, even in cases where the original text did not bear them out. Arnold felt that a literal translation made for oddity and for uncouthness. The author of Passage 1 provides an example of this oddity by discussing the literal translation of the Romance languages description of a winter day as It makes cold rather than familiar English phrase, It is cold. This example illustrates Arnolds claim that if a text is translated literally, then false emphases are created. In other words, Arnold felt that literal translation results in the meaning of a text becoming distorted and misleading. Nothing in the passage suggests that Arnold meant that literal translations lead to invented, fictional meanings in a text.. Choice (B) is incorrect. Something that is counterfeit is a forgery, made in imitation of something else with the intent to deceive. The author of Passage 1 refers to famous essayist and poet Matthew Arnold, who argued that a translator should always convey the impression of . . . qualities found in an original text, even in cases where the original text did not bear them out. Arnold felt that a literal translation made for oddity and for uncouthness. The author of Passage 1

provides an example of this oddity by discussing the literal translation of the Romance languages description of a winter day as It makes cold rather than familiar English phrase, It is cold. This example illustrates Arnolds claim that if a text is translated literally, then false emphases are created. In other words, Arnold felt that literal translation results in the meaning of a text becoming distorted and misleading. Nothing in the passage suggests that Arnold meant that literal translations lead to the creation of counterfeit, or forged, texts.. Choice (D) is incorrect. The term treacherous means dangerous or marked by betrayal of confidence or trust. The author of Passage 1 refers to famous essayist and poet Matthew Arnold, who argued that a translator should always convey the impression of . . . qualities found in an original text, even in cases where the original text did not bear them out. Arnold felt that a literal translation made for oddity and for uncouthness. The author of Passage 1 provides an example of this oddity by discussing the literal translation of the Romance languages description of a winter day as It makes cold rather than familiar English phrase, It is cold. This example illustrates Arnolds claim that if a text is translated literally, then false emphases are created. In other words, Arnold felt that literal translation results in the meaning of a text becoming distorted and misleading. Nothing in the passage suggests that Arnold meant that literal translations emphasize ideas in a way that is dangerous, or that literal translators betray anyones trust.. Choice (E) is incorrect. The term insincere means not genuine or untrue. When one is insincere, one is often deliberately dishonest and deceitful. The author of Passage 1 refers to famous essayist and poet Matthew Arnold, who argued that a translator should always convey the impression of . . . qualities found in an original text, even in cases where the original text did not bear them out. Arnold felt that a literal translation made for oddity and for uncouthness. The author of Passage 1 provides an example of this oddity by discussing the literal translation of the Romance languages description of a winter day as It makes cold rather than familiar English phrase, It is cold. This example illustrates Arnolds claim that if a text is translated literally, then false emphases are created. In other words, Arnold felt that literal translation results in the meaning of a text becoming distorted and misleading. Nothing in the passages suggests that Arnold meant that literal translations emphasize ideas in a way that is not genuine, or that literal translators are deliberately dishonest in any way.11 Explanation for Correct Answer B. Choice (B) is correct. In lines 37-39, the author of Passage 1 points out that many of us accept only literal translations because we want to give the original authors their due. He or she then goes on to contrast this attitude with that of European translators in ages past who thought of translation not in terms of a literal rendering but in terms of something being re-created. This contrast clearly illustrates that translators of the past did not hesitate to change the wording or meaning of an original work in order to evolve the work from themselves . . . from the known possibilities of their own languages, whereas todays translators are more concerned with honoring the original author of a work by adhering closely to the authors intended meaning.. Choice (A) is incorrect. In lines 37-39, the author of Passage 1 points out that many of us accept only literal translations because we want to give the original authors their due. He or she then goes on to contrast this attitude with that of European translators in ages past who thought of translation not in terms of a literal rendering but in terms of something being re-created. This contrast clearly illustrates that translators of the past did not hesitate to change the wording or meaning of an original work in order to evolve the work from themselves . . .

from the known possibilities of their own languages, whereas todays translators are more concerned with honoring the original author of a work by adhering closely to the authors intended meaning. The author of Passage 1 does not indicate that giving original authors their due involves debating the merits, or praiseworthy qualities, of certain poets.. Choice (C) is incorrect. In lines 37-39, the author of Passage 1 points out that many of us accept only literal translations because we want to give the original authors their due. He or she then goes on to contrast this attitude with that of European translators in ages past who thought of translation not in terms of a literal rendering but in terms of something being re-created. This contrast clearly illustrates that translators of the past did not hesitate to change the wording or meaning of an original work in order to evolve the work from themselves . . . from the known possibilities of their own languages, whereas todays translators are more concerned with honoring the original author of a work by adhering closely to the authors intended meaning. The author of Passage 1 does not suggest that giving original authors their due involves comparing different translations of one work.. Choice (D) is incorrect. In lines 37-39, the author of Passage 1 points out that many of us accept only literal translations because we want to give the original authors their due. He or she then goes on to contrast this attitude with that of European translators in ages past who thought of translation not in terms of a literal rendering but in terms of something being re-created. This contrast clearly illustrates that translators of the past did not hesitate to change the wording or meaning of an original work in order to evolve the work from themselves . . . from the known possibilities of their own languages, whereas todays translators are more concerned with honoring the original author of a work by adhering closely to the authors intended meaning. The author of Passage 1 does not suggest that giving original authors their due involves overlooking any flaws in authors famous works.. Choice (E) is incorrect. In lines 37-39, the author of Passage 1 points out that many of us accept only literal translations because we want to give the original authors their due. He or she then goes on to contrast this attitude with that of European translators in ages past who thought of translation not in terms of a literal rendering but in terms of something being re-created. This contrast clearly illustrates that translators of the past did not hesitate to change the wording or meaning of an original work in order to evolve the work from themselves . . . from the known possibilities of their own languages, whereas todays translators are more concerned with honoring the original author of a work by adhering closely to the authors intended meaning. The author of Passage 1 suggests that we give original authors their due by creating translations that express the same message in exactly the same way, not by treating translations as superior to original works.12 Explanation for Correct Answer D. Choice (D) is correct. The word crime in line 40 refers to what the author of Passage 1 imagines European translators in ages past would think of literal translations. These translators of long ago believed that it was far worthier to prove that the vernacular, the language of their contemporaries, could produce a work as great as an original work written in an ancient language; they thought in terms of somehow evolving [a] work from themselves, from their own might, from the known possibilities of their own languages. Because the European translators seem to have believed that it was their duty to evolve a work being translated, they most likely would argue that those who translate

original works word for word treat authors with excessive reverencethat is, that they show the original authors too much respect.. Choice (A) is incorrect. The word crime in line 40 refers to what the author of Passage 1 imagines European translators in ages past would think of literal translations. These translators of long ago believed that it was far worthier to prove that the vernacular, the language of their contemporaries, could produce a work as great as an original work written in an ancient language; they thought in terms of somehow evolving [a] work from themselves, from their own might, from the known possibilities of their own languages. The author of Passage 1 suggests that because the European translators seem to have believed that it was their duty to evolve a work being translated, they would say that people who translate original works word for word show the original authors too much respect. There is no indication that these translators of long ago would think that literal translations force ancient ideas into a modern world.. Choice (B) is incorrect. The word crime in line 40 refers to what the author of Passage 1 imagines European translators in ages past would think of literal translations. These translators of long ago believed that it was far worthier to prove that the vernacular, the language of their contemporaries, could produce a work as great as an original work written in an ancient language; they thought in terms of somehow evolving [a] work from themselves, from their own might, from the known possibilities of their own languages. The author of Passage 1 suggests that because the European translators seem to have believed that it was their duty to evolve a work being translated, they would say that people who translate original works word for word show the original authors too much respect. There is no indication that these translators of long ago would think that those who produce literal translations are encouraging literary accomplishment but neglecting traditional scholarship; indeed, there is no mention of the European translators opinion concerning traditional scholarship.. Choice (C) is incorrect. The word crime in line 40 refers to what the author of Passage 1 imagines European translators in ages past would think of literal translations. These translators of long ago believed that it was far worthier to prove that the vernacular, the language of their contemporaries, could produce a work as great as an original work written in an ancient language; they thought in terms of somehow evolving [a] work from themselves, from their own might, from the known possibilities of their own languages. There is no reason to suggest that the European translators would say that people who produce literal translations are likely to insist that ancient poems be read only in their original language; a work that has been translated is no longer in its original language, and someone who creates translations is unlikely to think that translations should not be read.. Choice (E) is incorrect. The word crime in line 40 refers to what the author of Passage 1 imagines European translators in ages past would think of literal translations. These translators of long ago believed that it was far worthier to prove that the vernacular, the language of their contemporaries, could produce a work as great as an original work written in an ancient language; they thought in terms of somehow evolving [a] work from themselves, from their own might, from the known possibilities of their own languages. The author of Passage 1 suggests that because the European translators seem to have believed that it was their duty to evolve a work being translated, they would say that people who translate original works word for word show the original authors too much respect. These translators of long ago might say that those who produce literal translations are not achieving artistic excellence, but there is no indication

that they would argue that literal translators are pursuing commercial goals instead of artistic ones.13 Explanation for Correct Answer A. Choice (A) is correct. The European translators mentioned in line 40 were mainly concerned with proving that the vernacular, the language of their contemporaries, was as capable of a great poem as the ancient language in which the original poem was composed. These translators were not concerned with literal translation; they read a work and then evolved that work from themselves, from their own might, from the known possibilities of their own languages. According to the author of Passage 2, Ezra Pounds translation of the poem The Seafarer illustrates a method of translating which . . . produces not so much a translation as a new poem in the spirit of the original. The author of Passage 2 goes on to say that Pounds translation conveys the important meaning of the Anglo-Saxon poem and does something that a literal translation fails to dorenders it into poetic English. The European translators referred to in Passage 1 would most likely praise Ezra Pounds version of The Seafarer because it captures the spirit of the original work while skillfully illustrating the beauties of modern English.. Choice (B) is incorrect. The European translators mentioned in line 40 were mainly concerned with proving that the vernacular, the language of their contemporaries, was as capable of a great poem as the ancient language in which the original poem was composed; they were concerned with reading a work and then evolving it from themselves, from their own might, from the known possibilities of their own languages. According to the author of Passage 2, Ezra Pounds translation of the poem The Seafarer illustrates a method of translating which . . . produces not so much a translation as a new poem in the spirit of the original. The author of Passage 2 goes on to say that Pounds translation conveys the important meaning of the Anglo-Saxon poem and does something that a literal translation fails to dorenders it into poetic English. Rather than admiring Pounds version of The Seafarer for its demonstration of the original poets genius, the European translators referred to in Passage 1 would most likely praise Pound for demonstrating the possibilities and beauties of his own language.. Choice (C) is incorrect. The European translators mentioned in line 40 were mainly concerned with proving that the vernacular, the language of their contemporaries, was as capable of a great poem as the ancient language in which the original poem was composed. These translators were not concerned with literal translation; they read a work and then evolved that work from themselves, from their own might, from the known possibilities of their own languages. According to the author of Passage 2, Ezra Pounds translation of the poem The Seafarer illustrates a method of translating which . . . produces not so much a translation as a new poem in the spirit of the original. The author of Passage 2 goes on to say that Pounds translation conveys the important meaning of the Anglo-Saxon poem and does something that a literal translation fails to dorenders it into poetic English. The European translators referred to in Passage 1 would not have wanted to directly replicate ancient languages, so it is unlikely that they would praise Pound for reproducing Anglo-Saxon sounds; instead, they probably would praise Pound for displaying the beauties of his own language.. Choice (D) is incorrect. The European translators mentioned in line 40 were mainly concerned with proving that the vernacular, the language of their contemporaries, was as capable of a great poem as the ancient language in which the original poem was composed. These translators were not concerned with

literal translation; they read a work and then evolved that work from themselves, from their own might, from the known possibilities of their own languages. According to the author of Passage 2, Ezra Pounds translation of the poem The Seafarer illustrates a method of translating which . . . produces not so much a translation as a new poem in the spirit of the original. The author of Passage 2 goes on to say that Pounds translation conveys the important meaning of the Anglo-Saxon poem and does something that a literal translation fails to dorenders it into poetic English. Rather than admiring Pounds version of The Seafarer for its representation of emotions, the European translators referred to in Passage 1 would most likely praise Pound for demonstrating the possibilities and beauties of his own language.. Choice (E) is incorrect. The European translators mentioned in line 40 were mainly concerned with proving that the vernacular, the language of their contemporaries, was as capable of a great poem as the ancient language in which the original poem was composed. These translators were not concerned with literal translation; they read a work and then evolved that work from themselves, from their own might, from the known possibilities of their own languages. According to the author of Passage 2, Ezra Pounds translation of the poem The Seafarer illustrates a method of translating which . . . produces not so much a translation as a new poem in the spirit of the original. The author of Passage 2 goes on to say that Pounds translation conveys the important meaning of the Anglo-Saxon poem and does something that a literal translation fails to dorenders it into poetic English. The European translators do not seem to have been concerned with imaginative interpretations of single words; rather, they were concerned with contemporary languageand its possibilitiesas a whole. These translators would probably praise Pound for demonstrating the possibilities and beauties of his own language, not just for interpreting individual words in an imaginative way.14 Explanation for Correct Answer C. Choice (C) is correct. In lines 64-65 the author of Passage 2 refers to scholars who believed that translation must be literal to be accurate. The author of Passage 1 refers to a scholar named Francis William Newman who attempted a translation of the works of Homer and supposed that a literal translation must be the most faithful translation. Newman and the scholars mentioned in Passage 2 share the attitude that the most accurate translations are literal translations.. Choice (A) is incorrect. In lines 64-65 the author of Passage 2 refers to scholars who believed that translation must be literal to be accurate. The author of Passage 1 claims that Nowadays . . . we are fond of literal translation, but he or she also cites Matthew Arnold, an essayist who did not think literal translations are always accurate: Arnold pointed out that a literal translation made for oddity and for uncouthness and believed that if a text is translated literally, then false emphases are created. The author of Passage 1 presents opposing sides of the issue of translation but does not appear to be supporting one side over the other. Therefore, we cannot say that he or she shares the attitude of the scholars mentioned in Passage 2.. Choice (B) is incorrect. In lines 64-65 the author of Passage 2 refers to scholars who believed that translation must be literal to be accurate. The author of Passage 2 does not seem to believe that only literal translations are accurate: he or she states that Literal translation sounds like no language at all and suggests that translators should aim at making an equation rather than a literal translation..

Choice (D) is incorrect. In lines 64-65 the author of Passage 2 refers to scholars who believed that translation must be literal to be accurate. The author of Passage 1 states that scholar Francis William Newman believed that a literal translation must be the most faithful translation and explains that in attempting a literal translation of Homers works, Newmans goal was to publish a translation that would contrast with the elegance of Alexander Popes eighteenthcentury translation. This implies that Popes translation is not a literal one. Pope does not seem to have shared the attitude that only literal translations are accurate.. Choice (E) is incorrect. In lines 64-65 the author of Passage 2 refers to scholars who believed that translation must be literal to be accurate. The author of Passage 1 explains that Matthew Arnold did not think literal translations are always accurate. He or she states that Arnold pointed out that a literal translation made for oddity and for uncouthness and believed that if a text is translated literally, then false emphases are created.15 Explanation for Correct Answer E. Choice (E) is correct. The author of Passage 2 writes that scholars who believed that translation must be literal to be accurate were angered by Pounds claim that his translation of The Seafarer was as nearly literal as any translation could be. The author states that it is obvious that Pounds version of the poem is not the most literal of translations and then goes on to point out inaccuracies in Pounds translation: Where the Anglo-Saxon has wrecan . . . Pound has reckon; there are . . . some mistakes, as when Pound misreads purh . . . as pruh. The author of Passage 2 suggests that the scholars wrath was understandable because Pound provoked the scholars with his claim about his translation, a claim that could be disputed.. Choice (A) is incorrect. The author of Passage 2 writes that scholars who believed that translation must be literal to be accurate were angered by Pounds claim that his translation of The Seafarer was as nearly literal as any translation could be. The author states that it is obvious that Pounds version of the poem is not the most literal of translations and then goes on to mention inaccuracies in Pounds translation (Where the Anglo-Saxon has wrecan . . . Pound has reckon; Pound misreads purh . . . as pruh). By mentioning instances in which Pounds translation is not accurate, the author of Passage 2 suggests that the scholars wrath was understandablenot irrationalbecause Pound provoked the scholars with a claim that could be disputed. Further, the author does not suggest that the scholars were concerned with Pounds intentions; they were concerned with his actual translation.. Choice (B) is incorrect. The author of Passage 2 writes that scholars who believed that translation must be literal to be accurate were angered by Pounds claim that his translation of The Seafarer was as nearly literal as any translation could be. The author states that it is obvious that Pounds version of the poem is not the most literal of translations and then goes on to mention inaccuracies in Pounds translation (Where the Anglo-Saxon has wrecan . . . Pound has reckon; Pound misreads purh . . . as pruh). The author of Passage 2 does not indicate whether the angered scholars were also translators, but he or she does not suggest that their wrath was meaningless; rather, he or she suggests that the scholars wrath was reasonable and understandable because Pound provoked the scholars with a claim that could be disputed.. Choice (C) is incorrect. The author of Passage 2 writes that scholars who believed that translation must be literal to be accurate were angered by Pounds claim that his translation of The Seafarer was as nearly literal as any translation

could be. The author states that it is obvious that Pounds version of the poem is not the most literal of translations and then goes on to mention inaccuracies in Pounds translation (Where the Anglo-Saxon has wrecan . . . Pound has reckon; Pound misreads purh . . . as pruh). Aside from indicating that certain scholars took notice of Pounds method of translation, the author of Passage 2 does not discuss the attention Pounds method received. Further, although he or she suggests that the scholars wrath was understandable, the author does not indicate that this anger was inevitable, or unavoidable.. Choice (D) is incorrect. The author of Passage 2 writes that scholars who believed that translation must be literal to be accurate were angered by Pounds claim that his translation of The Seafarer was as nearly literal as any translation could be. The author states that it is obvious that Pounds version of the poem is not the most literal of translations and then goes on to point out inaccuracies in Pounds translation (Where the Anglo-Saxon has wrecan . . . Pound has reckon; Pound misreads purh . . . as pruh). The author of Passage 2 does not suggest that the scholars wrath was not genuine; the scholars anger seems to have been sincere. However, there is no indication that the scholars felt personally insulted by Pounds claim concerning his own translation. 16 Explanation for Correct Answer C. Choice (C) is correct. The author of Passage 2 explains that Pound claimed his translation of The Seafarer was as nearly literal as any translation could be. The author then goes on to state that it is obvious that Pounds version of the poem was not a literal translation, and provides definitions of Anglo-Saxon words that Pound translated into modern English: Where the Anglo-Saxon has wrecan (to make, compose), Pound has reckon. Where the Anglo-Saxon has sumeres weard (guardian of summer), Pound has summerward. The author of Passage 2 refers to these definitions to demonstrate that Pound rendered the sounds of certain words from one language to another but did not always translate the words meaningsaccurately; the author shows that Pounds claim regarding the accuracy of his translation is questionable.. Choice (A) is incorrect. The author of Passage 2 explains that Pound claimed his translation of The Seafarer was as nearly literal as any translation could be. The author then goes on to state that it is obvious that Pounds version of the poem was not a literal translation, and provides definitions of Anglo-Saxon words that Pound translated into modern English: Where the Anglo-Saxon has wrecan (to make, compose), Pound has reckon. Where the Anglo-Saxon has sumeres weard (guardian of summer), Pound has summerward. The author of Passage 2 does not refer to the definitions of the words wrecan and sumeres weard to suggest that these words have more than one meaning; on the contrary, he or she seems to be suggesting that these Anglo-Saxon words each have one primary meaning. The author of Passage 2 indicates that when Pound translated certain Anglo-Saxon words into modern English, he captured the words sounds but did not accurately translate their meanings.. Choice (B) is incorrect. The author of Passage 2 explains that Pound claimed his translation of The Seafarer was as nearly literal as any translation could be. The author then goes on to state that it is obvious that Pounds version of the poem was not a literal translation, and provides definitions of Anglo-Saxon words that Pound translated into modern English: Where the Anglo-Saxon has wrecan (to make, compose), Pound has reckon. Where the Anglo-Saxon has sumeres weard (guardian of summer), Pound has summerward. The author of Passage 2 is not discussing developmental changes that have led to modern English words; he or she is discussing errors Pound made when he

translated Anglo-Saxon words into modern English, making the point that Pound captured the sounds of certain Anglo-Saxon words but did not accurately translate their meanings.. Choice (D) is incorrect. The author of Passage 2 explains that Pound claimed his translation of The Seafarer was as nearly literal as any translation could be. The author then goes on to state that it is obvious that Pounds version of the poem was not a literal translation, and provides definitions of Anglo-Saxon words that Pound translated into modern English: Where the Anglo-Saxon has wrecan (to make, compose), Pound has reckon. Where the Anglo-Saxon has sumeres weard (guardian of summer), Pound has summerward. The author of Passage 2 refers to an ancient language (Anglo-Saxon), but nowhere in the passage does he or she mention an ancient person or discuss the thematic power of any such person. The author of Passage 2 provides definitions to make the point that when Pound translated The Seafarer into modern English, he captured the sounds of certain Anglo-Saxon words but did not accurately translate their meanings.. Choice (E) is incorrect. The author of Passage 2 explains that Pound claimed his translation of The Seafarer was as nearly literal as any translation could be. The author then goes on to state that it is obvious that Pounds version of the poem was not a literal translation, and provides definitions of Anglo-Saxon words that Pound translated into modern English: Where the Anglo-Saxon has wrecan (to make, compose), Pound has reckon. Where the Anglo-Saxon has sumeres weard (guardian of summer), Pound has summerward. The author of Passage 2 mentions that Pound reproduced the sounds of Anglo-Saxon words, but he or she is not interested in challenging any assumptions about the sounds of ancient words. Rather, the author is concerned with the meanings of words from an ancient language; he or she provides definitions of certain AngloSaxon words to make the point that some of Pounds translations were inaccurate.17 Explanation for Correct Answer C. Choice (C) is correct. The author of Passage 2 indicates that instead of finding words that have the same meaning as words in another language, a translator should find words that bring over the sense and spirit of the original so that they are understood; he or she states that literal translation sounds like no language at all. The author of Passage 1 refers to the French phrase Il fait froida phrase one could use to describe a cold dayand notes that when translated literally, the phrase becomes It makes cold. This phrase is awkward and might not be readily understood by people familiar with the statement It is cold. The author of Passage 2 would likely cite the literal translation of Il fait froid as one that sounds like no language at all.. Choice (A) is incorrect. The author of Passage 2 states that literal translation sounds like no language at all and indicates that translators should find words that bring over the sense and spirit of the original so that they are understood. In contrast to Newmans translation of Homer, Popes translation is not literal and is described by the author of Passage 1 as elegant. Popes translation of Homer is not literal and there is no indication that his translation is difficult to understand or sounds meaningless, so it is unlikely that the author of Passage 2 would cite Popes work as an example of a translation that sounds like no language at all.. Choice (B) is incorrect. The author of Passage 2 states that literal translation sounds like no language at all and indicates that translators should find words that bring over the sense and spirit of the original so that they are understood. Arnolds review of Newmans book is not a translationit is a review or a critique

of Newmans translation of Homerand there is no indication that the review is difficult to understand or sounds meaningless in any way. Therefore, the author of Passage 2 would not cite it as an example of a translation that sounds like no language at all.. Choice (D) is incorrect. The author of Passage 2 states that literal translation sounds like no language at all and indicates that translators should find words that bring over the sense and spirit of the original so that they are understood. The author of Passage 1 provides Captain Burtons literal translation of the title Quitah alif laila wa laila as Book of the Thousand Nights and a Night, noting that this literal translation might cause a slight shock of surprise. He or she suggests that the title Book of the Thousand and One Nights, while not a literal translation, does capture the spirit of the original title. The title Book of the Thousand and One Nights is easily understood, so it is unlikely that the author of Passage 2 would cite it as an example of a translation that sounds like no language at all.. Choice (E) is incorrect. The author of Passage 2 states that literal translation sounds like no language at all and indicates that translators should find words that bring over the sense and spirit of the original so that they are understood. According to the author of Passage 1, throughout the Middle Ages in Europe translators were thought of as having read a work and then somehow evolving that work . . . from the known possibilities of their own languages. The author of Passage 1 suggests that these translators used language in a skillful way, and there is no indication that the translations they produced were difficult to understand or sounded meaningless; there is no reason to believe that the author of Passage 2 would cite translations from the Middle Ages as an examples of translations that sound like no language at all.18 Explanation for Correct Answer A. Choice (A) is correct. The author of Passage 2 discusses Ezra Pounds translation of The Seafarer, pointing out that although the translation is not exactly literal, it conveys the important meaning of the Anglo-Saxon poem. He or she goes on to say that Pound showed that the aim of translation should not be to translate something word for word but to find words that bring over the sense and spirit of the original so that they are understood. In this context, the word sense refers to the conveyed or intended meaning of something. The authors description of Pounds contribution as a translator emphasizes the importance of conveying the intended meaning of an original text in a way that an audience will understand.. Choice (B) is incorrect. Awareness is a realization, perception, or knowledge of something. The author of Passage 2 discusses Ezra Pounds translation of The Seafarer, pointing out that although the translation is not exactly literal, it conveys the important meaning of the Anglo-Saxon poem. He or she goes on to say that Pound showed that the aim of translation should not be to translate something word for word but to find words that bring over the sense and spirit of the original so that they are understood. The author of Passage 2 does not indicate that a translators aim should be to bring over the awareness of an original text; an awareness of a text would simply be the knowledge that that text exists. As it is used in line 82, the term sense refers to meaning; the author of Passage 2 suggests that a translation should capture the meaning of the original text.. Choice (C) is incorrect. Practicality is utility or usefulness. The author of Passage 2 discusses Ezra Pounds translation of The Seafarer, pointing out that although the translation is not exactly literal, it conveys the important meaning

of the Anglo-Saxon poem. He or she goes on to say that Pound showed that the aim of translation should not be to translate something word for word but to find words that bring over the sense and spirit of the original so that they are understood. The author of Passage 2 does not indicate that a translators aim should be to bring over the practicality of an original text or that a translator must consider the overall usefulness of a text when translating it. As it is used in line 82, the term sense refers to meaning, not practicality; the author of Passage 2 suggests that a translation should capture the meaning of the original text.. Choice (D) is incorrect. Intelligence is mental acuteness. The author of Passage 2 discusses Ezra Pounds translation of The Seafarer, pointing out that although the translation is not exactly literal, it conveys the important meaning of the Anglo-Saxon poem. He or she goes on to say that Pound showed that the aim of translation should not be to translate something word for word but to find words that bring over the sense and spirit of the original so that they are understood. It is somewhat illogical to suggest that a translators aim should be to bring over the intelligence of an original text; a text might demonstrate an authors mental acuteness, but the text itself cannot have intelligence. As it is used in line 82, the term sense refers to meaning, not intelligence; the author of Passage 2 suggests that a translation should capture the meaning of the original text.. Choice (E) is incorrect. Appreciation is admiration, approval, or gratitude. The author of Passage 2 discusses Ezra Pounds translation of The Seafarer, pointing out that although the translation is not exactly literal, it conveys the important meaning of the Anglo-Saxon poem. He or she goes on to say that Pound showed that the aim of translation should not be to translate something word for word but to find words that bring over the sense and spirit of the original so that they are understood. The author of Passage 2 does not indicate that a translators aim should be to bring over the appreciation of an original text, or to somehow carry over admiration or approval of that text; there is no suggestion that it is important for a translation to be admired. As it is used in line 82, the term sense refers to meaning, not appreciation; the author of Passage 2 suggests that a translation should capture the meaning of the original text.19 Explanation for Correct Answer A. Choice (A) is correct. The author of Passage 1 states that Arnold believed a good translation should convey the impression of [the] qualities found in an original text; Arnold cited qualities such as clarity, nobility, and simplicity found in the works of Homer. The author of Passage 2 suggests that Pounds translation of The Seafarer bring[s] over the sense and spirit of the original poem. Both Arnold and Pound seem to have assumed that a good translation captures the distinctive naturethe qualities, sense, and spiritof the original text.. Choice (B) is incorrect. There is no indication in the passages that either Arnold or Pound assumed that a good translation captures the poets thoughts about the creative process; neither passage discusses the creative process of a poet creating an original work or of a poet translating a text. Instead, the passages suggest that both Arnold and Pound assumed that a good translation captures the distinctive nature of the original text. Passage 1 indicates that Arnold believed in the importance of conveying the qualities found in an original text, and Passage 2 indicates that Pounds translation of The Seafarer bring[s] over the sense and spirit of the original poem.. Choice (C) is incorrect. There is no indication in Passage 1 that Arnold assumed that a good translation captures the values and ideals of the original poets era;

Arnold focused on the importance of conveying the qualities (such as clarity and nobility) found in an original text, not the values and ideals of the era in which the text was created. Likewise, there is no indication in Passage 2 that Pound assumed that a translation should capture such values and ideals; the author of Passage 2 suggests that Pounds translation of The Seafarer bring[s] over the sense and spirit of the original poem, not that it reflects the era in which the original poem was written. Arnold and Pound seem to have been concerned with capturing the distinctive naturethe qualities, sense, and spirit of an original text.. Choice (D) is incorrect. The author of Passage 2 states that when Pound translated The Seafarer, he aimed to reproduce the feel of the original by reproducing Anglo-Saxon sounds; this suggests that Pound may have felt that a good translation captures the sound and rhythm of a texts original language. However, there is no indication in Passage 1 that Arnold shared Pounds assumption. The author of Passage 1 states that Arnold believed in the importance of conveying qualities, such as clarity and nobility, that can be found in an original text, but he or she does not address Arnolds thoughts concerning the actual sounds and rhythms of the original language of a text..

Choice (E) is incorrect. There is no indication in the passages that either Arnold or Pound assumed that a good translation captures the subtleties of the poets reasoning; neither passage discusses subtleties in the reasoning of a poet creating an original work or of a poet translating a text. Instead, the passages suggest that both Arnold and Pound assumed that a good translation captures the distinctive nature of the original text. Passage 1 indicates that Arnold believed in the importance of conveying the qualities found in an original text, and Passage 2 indicates that Pounds translation of The Seafarer bring[s] over the sense and spirit of the original poem. Section #10: View Explanations 1 Explanation for Correct Answer C. Choice (C) is correct. It avoids the wordiness of the original by deleting the phrase As a place that is and by replacing the unidiomatic is flowing with flows.. Choice (A) involves wordiness. The phrase As a place that is is unnecessary and should be deleted, and the verb phrase is flowing should be replaced with the simple present-tense verb flows.. Choice (B) involves improper modification. It does not make sense to say that the flow of the Cahaba River is A place that is home to 131 species of fish.. Choice (D) is awkward and wordy. The entire phrase should be replaced by the concise Home to 131 species of fish, the Cahaba River flows.. Choice (E) results in a sentence fragment. There is no main verb to carry out the action of the sentence, only the participle flowing. 2 Explanation for Correct Answer D.

Choice (D) is correct. It avoids the verb form error of the original by providing the appropriate noun forms the trapping and removal to serve as the subjects of the verb affected.. Choice (A) involves a verb form error. The infinitive phrase To trap and remove does not appropriately serve as a subject of the verb affected.. Choice (B) results in illogical phrasing. It is trapping and removal of large alligators, and not the alligators themselves, that greatly affected the ecosystems of some southeast swamplands.. Choice (C) results in a lack of subject. There is no subject to complete the action of the verb affected.. Choice (E) results in illogical phrasing. It is trapping and removal of large alligators, and not the alligators themselves, that greatly affected the ecosystems of some southeast swamplands.3 Explanation for Correct Answer C. Choice (C) is correct. It avoids the sentence-fragment error of the original by providing the verb phrase is known to carry out the action of the sentence.. Choice (A) involves a sentence fragment. There is no main verb to carry out the action of the sentence, only the past participle known.. Choice (B) results in a comma splice. Two independent clauses (Iceland is known for its colorful tales and they poetically depict the dangerous, complex lives of the countrys earliest settlers) are improperly joined by only a comma.. Choice (D) results in a sentence fragment. There is no main verb to carry out the action of the sentence, only the verb depict, which is part of the modifying phrase beginning with which poetically depict.. Choice (E) results in a sentence fragment. There is no main verb to carry out the action of the sentence.4 Explanation for Correct Answer E. Choice (E) is correct. It avoids the lack of parallelism of the original by providing two verb phrases (replace the lightbulb and position the shelves) that are parallel with the preceding verb phrase connect the icemaker.. Choice (A) involves a lack of parallelism. The gerund phrases replacing the lightbulb and positioning the shelves are not parallel with the preceding phrase connect the icemaker.. Choice (B) results in wordiness and a lack of parallelism. The conjunctions both and as well as are not necessary and should be deleted and replaced with and, respectively. In addition, the gerund phrases replacing the lightbulb and positioning the shelves are not parallel with the preceding phrase connect the icemaker.. Choice (C) results in a lack of parallelism. The prepositional phrase with replacing the lightbulb and the adverb and verb phrase then positioning the shelves are not parallel with the preceding verb phrase connect the icemaker.. Choice (D) results in improper coordination. A conjunction is needed to link the first verb connect to the second verb replace.5

Explanation for Correct Answer E. Choice (E) is correct. It avoids the modification error of the original by placing the subject (the family) immediately after the modifying phrase (Saying nothing further).. Choice (A) involves improper modification. It does not make sense to say that the table was Saying nothing further.. Choice (B) involves illogical phrasing. It does not make sense to say that the family cleared the table By saying nothing further. The preposition by should be deleted.. Choice (C) results in a verb-tense error. The present-tense verb clears is not consistent with the preceding past-tense verb phrase was said.. Choice (D) results in a comma splice. Two independent clauses (The family said nothing further and the table was cleared in preparation for the evening meal) are improperly joined by only a comma.6 Explanation for Correct Answer C. Choice (C) is correct. It avoids the lack of parallelism of the original by appropriately completing the correlative construction, beginning with not only to emulate, with the parallel but also to recognize.. Choice (A) results in a lack of parallelism. The sentence intends to link two verbs together using the correlative construction not onlybut also However, here the conjunction and is improperly used in place of but also. In addition, the verb recognizing is not parallel with the preceding infinitive to emulate.. Choice (B) results in a lack of parallelism. The sentence intends to link two verbs together using the correlative construction not onlybut also However, here the conjunction while is improperly used in place of but also. In addition, the verb recognizing is not parallel with the preceding infinitive to emulate.. Choice (D) results in a lack of parallelism. The noun phrase his recognition, which follows but also, is not parallel with the infinitive verb phrase to emulate.. Choice (E) results in a lack of parallelism. The sentence intends to link two verbs together using the correlative construction not onlybut also However, here the conjunction and is improperly used in place of but. 7 Explanation for Correct Answer A. Choice (A) is correct. It avoids the errors of the other options by providing the appropriate modifying phrase enjoying a life of leisure and drawing on natures bounty.. Choice (B) involves a vague pronoun error. It is not clear to what the plural pronoun they is meant to refer.. Choice (C) involves a flaw in sentence structure. It is not clear what the phrase to draw on natures bounty for their sustenance is meant to modify.. Choice (D) includes a vague pronoun and results in a comma splice. It is not clear to what the plural pronoun they is meant to refer. In addition, two independent clauses (In the early eighteenth century, English poets and painters imagined

the countryside filled with shepherds who enjoyed a life of leisure and then they drew on natures bounty for their sustenance) are improperly joined by only a comma.. Choice (E) involves a flaw in sentence structure. The plural pronoun they cannot function as a modifier of the noun shepherds. The pronoun who should be used instead.8 Explanation for Correct Answer C. Choice (C) is correct. It avoids the pronoun-agreement error of the original by replacing the plural phrase they are with the singular verb is.. Choice (A) involves a pronoun-agreement error. The plural pronoun they cannot be used to refer to the singular the broad-banded copperhead.. Choice (B) involves a pronoun-agreement error. The plural pronoun they cannot be used to refer to the singular the broad-banded copperhead.. Choice (D) involves a pronoun-agreement error. The plural possessive pronoun their cannot be used to refer to the singular the broad-banded copperhead.. Choice (E) results in a comma splice. Two independent clauses (Smallest of the copperhead snakes, the broad-banded copperhead, if disturbed, vibrates its tail rapidly and in addition, it secretes its substance that smells like cucumbers) are improperly joined by only a comma.9 Explanation for Correct Answer A. Choice (A) is correct. It avoids the errors of the other options by providing the appropriate construction The reasonis that and by providing a consistent verb tense.. Choice (B) results in an ambiguous-pronoun error and a verb-tense error. It is unclear whether the pronoun it refers to the reason or humor. In addition, the past-tense verb exposed and the future conditional verb phrase could defuse are inconsistent with the preceding present-tense verb make.. Choice (C) results in redundancy and a subject-verb agreement error. The sentence begins with The reason so the conjunction because is unnecessary and should be replaced with that. In addition, the plural verbs expose and defuse do not agree with the singular subject humor.. Choice (D) is redundant and unidiomatic. The sentence begins with The reason, so the phrase because of is unnecessary and should be replaced with that. In addition, the participles exposing and diffusing should be replaced with the verbs expose and difuse.. Choice (E) results in illogical phrasing. It does not make sense to say that The reasonis that of exposing wrongs and defusing anger and resentment. 10 Explanation for Correct Answer B. Choice (B) is correct. It avoids the illogical statement of the original by comparing poets to the poet Edna St. Vincent Millay instead of to her poetry.. Choice (A) results in an illogical statement. It does not make sense to compare poets to Edna St. Vincent Millays poetry..

Choice (C) results in an illogical statement. It does not make sense to compare poets to the poetry of Edna St. Vincent Millay.. Choice (D) results in an illogical statement. It does not make sense to compare poets to something in the poems of Edna St. Vincent Millay.. Choice (E) involves a verb-tense error. The future conditional would do is not consistent with the preceding present-tense verb combine. 11 Explanation for Correct Answer D. Choice (D) is correct. It avoids the error of the original by providing the appropriate modifying phrase with the most venturesome playing chase and executing complicated turns on the ice.. Choice (A). Choice (B) results in a comma splice. Two independent clauses (In winter, when the ponds froze over, the villagers went skating and the most venturesome played chase and executed complicated turns on the ice) are improperly joined by only a comma.. Choice (C) involves a coordination error. The phrase in that illogically implies that the villagers went skating because the most venturesome played chase. The phrase in that should be replaced with a conjunction that more appropriately establishes the intended relationship between the villagers skating and the most venturesome playing chase.. Choice (E) results in a comma splice. Two independent clauses (In winter, when the ponds froze over, the villagers went skating and accordingly the most venturesome played chase and executed complicated turns on the ice) are improperly joined by only a comma.12 Explanation for Correct Answer E. Choice (E) is correct. It avoids the vague-pronoun error of the original by providing the plural noun people to which the pronoun they can refer. In addition, it provides a necessary comma after the conjunction therefore.. Choice (A) involves a vague-pronoun error. There is nothing in the sentence to which the plural pronoun they can logically refer. In addition, a comma is needed after the conjunction therefore.. Choice (B) involves a vague-pronoun error. There is nothing in the sentence to which the plural pronoun they can logically refer.. Choice (C) results in redundancy. Both since and therefore indicate the causeeffect relationship between a persons need to feel a connection with the past and their preserving and recycling of old objects, so one of the conjunctions should be deleted.. Choice (D) results in a coordination error and awkward phrasing. The conjunction and is unnecessary and should be deleted, and a semicolon should precede therefore. In addition, the phrase a need to reconnect with and recover the past is a feeling for many people is unidiomatic.13 Explanation for Correct Answer C. Choice (C) is correct. It avoids the subject-verb agreement error and noun agreement error of the original by providing the singular verb is and the

singular noun barrier to agree with the singular subject Andes mountain chain.. Choice (A) involves a subject-verb agreement error and a noun agreement error. The plural verb are and the plural noun barriers do not agree with the singular subject Andes mountain chain.. Choice (B) involves a subject-verb agreement error and a noun agreement error. The plural verb are and the plural noun barriers do not agree with the singular subject Andes mountain chain.. Choice (D) results in a noun agreement error and a subject-verb agreement error. The plural noun barriers and the plural verb are do not agree with the singular noun Andes mountain chain.. Choice (E) involves a verb-tense error and a flawed comparative structure. The past-tense verb was is not consistent with the preceding present-tense verb includes. In addition, without the word more, this sentence does not properly express the intended comparison between The Andes Mountain chain and the Rocky Mountains.14 Explanation for Correct Answer B. Choice (B) is correct. It avoids the redundancy of the original by deleting the phrase is the reason why.. Choice (A) involves redundancy. The word Because establishes the cause-effect relationship between European filmmakings shutting down and the rise to prominence of the film industry in the United States; therefore, the phrase is the reason why is not necessary.. Choice (C) results in a comma splice. Two independent clauses (European filmmaking all but shut down during the First World War and the film industry in the United States rose to prominence) are improperly joined by only a comma.. Choice (D) results in improper coordination. The conjunction so inappropriately joins a subordinate clause (With European filmmaking close to shutting down during the First World War) with an independent clause (the film industry in the United States roes to prominence)..

Choice (E) is awkward and wordy. The concise phrase Because European filmmaking all but shut down during the First World War should be used instead.

You might also like